Download as pdf or txt
Download as pdf or txt
You are on page 1of 124

Lakshya JEE AIR (2025)

Magnetic Effect of Current

MAGNETIC EFFECT OF CURRENT


The branch of physics which deals with the magnetism due to electric current or moving
charge (i.e. electric current is equivalent to the charges or electrons in motion) is called
electromagnetism.

ORESTED'S DISCOVERY
The relation between electricity and magnetism was discovered by Orested in 1820.
Orested showed that the electric current through the conducting wire deflects the magnetic
needle held below the wire.
S N

North

• When the direction of current in conductor is reversed then deflection of magnetic needle is
also reversed
• On increasing the current in conductor or bringing the needle closer to the conductor the
deflection of magnetic needle increases.
Oersted discovered a magnetic field around a conductor carrying electric current. Other related
facts are as follows:
– +

S I

N
Oersted's experiment. Current in
the wire deflects the compass needle

(a) A magnet at rest produces a magnetic field around it while an electric charge at rest produce an
electric field around it.
(b) A current carrying conductor has a magnetic field and not an electric field around it. On the
other hand, a charge moving with a uniform velocity has an electric as well as a magnetic field
around it.
(c) An electric field cannot be produced without a charge whereas a magnetic field can be
produced without a magnet.

PHYSICS WALLAH 1
Magnetic Effect of Current

(d) No poles are produced in a coil carrying current but such a coil shows north and south
polarities.
(e) All oscillating or an accelerated charge produces E.M. waves also in additions to electric and
magnetic fields.

• Current Element
A very small element ab of length d of a thin conductor carrying current I called current element.
Current element is a vector quantity whose magnitude is equal to the product of current and
length of small element having the direction of the flow of current.
dB
P

r
I
I

a
d b →
d
• Biot – Savart's Law
With the help of experimental results, Biot and Savart arrived at a mathematical expression
that gives the magnetic field at some point in space in terms of the current that produces the
field. That expression is based on the following experimental observations for the magnetic
field dB at a point P associated with a length element d of a wire carrying a steady current I.
1
dB  I, dB  d, dB  sin and dB 
r2
Id sin   0 Id sin 
 dB  dB =
r 2
4 r2
Vector form of Biot-Savart's law
 0 Id sin 
dB = nˆ =unit vector perpendicular to the plane of ( Id ) and ( r )
4 r2
 0 Id  r
dB = [ Id × r = (Id) (r)sin n̂ ]
4 r3

GOLDEN KEY POINTS


 0 Id  r
• According to dB = , direction of magnetic field vector ( dB ) is always perpendicular
4 r3
to the plane of vectors ( Id ) and ( r ), where plane of ( Id ) and ( r ) is the plane of wire.
• Magnetic field on the axis of current carrying conductor is always zero ( = 0° or  = 180°)
• Magnetic field on the perimeter of circular loop or coil is always minimum.

MAGNETIC FIELD LINES (By Michal Faraday)


In order to visualize a magnetic field graphically, Michal faraday introduced the concept of
field lines.
Field lines of magnetic field are imaginary lines which represents direction of magnetic field
continuously.
PHYSICS WALLAH 2
Magnetic Effect of Current

GOLDEN KEY POINTS


• Magnetic field lines are closed curves.
• Tangent drawn at any point on field line represents direction of the field at that point.
• Field lines never intersects to each other.
• At any place crowded lines represent stronger field while distant lines represents weaker field.
• In any region, if field lines are equidistant and straight the field is uniform otherwise not.
Non-uniform Field Uniform Field

Magnitude is Direction is Both magnitude Both magnitude


not constant not constant and direction are and direction are
not constant constant

• Magnetic field lines emanate from or enters in the surface of a magnetic material at any angle.
• Magnetic field lines exist inside every magnetised material.
• Magnetic field lines can be mapped by using iron dust or using compass needle.

RIGHT HAND THUMB RULE


This rule gives the pattern of magnetic field lines due to current carrying wire.
(i) Straight current (ii) Circular current
Thumb → In the direction of current Curling fingers → In the direction of current,
Curling fingers → Gives field line pattern Thumb → Gives field line pattern
Case I : wire in the plane of the paper Case I : wire in the plane of the paper
I
Magnetic
field lines ACW CW

B B

Towards observer or Away from the observer


perpendicular or perpendicular
I out-wards inwards

Case II : Wire is ⊥ to the plane of the paper. Case II : Wire is ⊥ to the plane of the paper

ACW CW
S N N S

B I
ACW concentric & CW concentric &
circular field lines circular field lines Towards observer Away from the observer

GOLDEN KEY POINTS


• When current is straight, field is circular
• When current is circular, field is straight (along axis)
• When wire is in the plane of paper, the field is perpendicular to the plane of the paper.
• When wire is perpendicular to the plane of paper, the field is in the plane of the paper.
PHYSICS WALLAH 3
Magnetic Effect of Current

APPLICATION OF BIOT-SAVART LAW :


• Magnetic field surrounding a thin straight current carrying conductor
A

id
 r

a  
O

P

B
AB is a straight conductor carrying current i from B to A. At a point P,
whose perpendicular distance from AB is OP =a, the direction of field is perpendicular to the
plane of paper, inwards (represented by a cross)
= a tan  dl = a sec2 d...(i)

 = 90°–  & r = asec

• By Biot-Savart’s law
 0 id sin 
dB =  (due to a current element id at point P)
4 r2
 0 id sin 
B=  dB =  4 r2
(due to wire AB)

 0i
B=
4  cos d 
Taking limits of integration as – 2 – 2 to 1
1
 0i  0i  0i
sin1 + sin 2 
4 a 
B= cosd  = [sin ]−12 = (inwards)
− 2
4 a 4 a

Example
Magnetic field due to infinite length wire at point 'P'
Solution
0 I 0 I
BP = [sin90° + sin90°] BP =
4 d 2 d

I 2 = 90°
90°
M P
1 = 90°

d
−

PHYSICS WALLAH 4
Magnetic Effect of Current

Example
Magnetic field due to semi infinite length wire at point 'P'
Solution
0 I 0 I
BP = [sin + sin90°] BP = [sin + 1]
4 d 4 d

90° 2=90°
M P
1=

L d

Example
Magnetic field due to special semi infinite length wire at point 'P'
Solution
0 I 0 I
BP = [sin0° + sin90°] BP =
4 d 4 d

I
90° 2=90°

M 1=° P
d

Example
Magnetic field due to special finite length wire at point 'P'
N

90° 2=°
M P
d
1=0°
Solution
0 I 0 I
BP = [sin0° + sin]; BP = sin
4 d 4 d

Example
If point ‘P’ lies outside the line of wire then magnetic field at point ‘P’:
d
−  P
2
− 

I 1

Solution
0 I  I
BP = sin ( 90 − 1 ) − sin ( 90 −  2 )  = 0 (cos 1 − cos  2 )
4 d 4 d

PHYSICS WALLAH 5
Magnetic Effect of Current

• Magnetic field due to a loop of current


Magnetic field lines due to a loop of wire are shown in the figure
B
i

i
i

The direction of magnetic field on the axis of current loop can be determined by right hand
thumb rule. If fingers of right hand are curled in the direction of current, the stretched thumb is
in the direction of magnetic field.

• Calculation of magnetic field


Consider a current loop placed in y-z plane carrying current i in anticlockwise sense as seen
from positive x-axis. Due to a small current element id shown in the figure, the magnetic
 0 id sin 90 0
field at P is given by dB = .
4 r2
z-axis

id
i y-axis
R

r= R2+x2
O
  dB
x
i P
x-axis

The angle between id and r is 900 because id is along y-axis, while r lies in x-z plane.
The direction of dB is perpendicular to r as shown. The vector dB can be resolved into two
components, dB cos along z-axis and dB sin along x-axis.
For any two diametrically opposite current elements, the components along x-axis add up,
while the other two components cancel out. Therefore, the field at P is due to x-component of
field only. Hence, we have
 0 id  0 id R  0 iR
B=  dB sin  =  4 r 2
sin  = 
4 r 2

r
B=
4 r 3  d   d = 2 R

B=
 0 i  2 R 2
4 r 3
=
0 i  2 R 2
4 ( R + x
2 2
)
3/ 2 ( r= R2 + x2 )
z-axis
dBcos
 dB

P
x-axis
dBsin

 0i
(a) At the centre, x=0, Bcentre=
2R
−3/ 2
 0i  x2   0i  3x2 
(b) At points very close to centre, x<<R  B= 1 + 2  = 1 − 
2  R  2R  2R2 
PHYSICS WALLAH 6
Magnetic Effect of Current

 0 I 2 R 2
(c) At points far off from the centre, x>>R  B =
4 x3
 0 2M
(d) The result in point (c) is also expressed as B =
4 x 3
Where M = I × R2, is called magnetic dipole moment.

Example
Find the magnetic field at the centre of a current carrying conductor bent in the form of an arc
subtending angle  at its centre. Radius of the arc is R.
Solution
Let the arc lie in x-y plane with its centre at the origin. y-axis

Consider a small current element id as shown. id i


The field due to this element at the centre is
 0 id sin 90 0 R
dB=
4 R2
( id and R are perpendicular ) 
d
R

 0 iRd  0 i 
Now d = Rd   dB = dB = d
4 4 R
2 x-axis
R O

The direction of field is outward perpendicular to plane of paper



 0i  0i  0i
 d  = 4 R   

Total magnetic field B=  dB  B =
4 R 0
B=
4 R

0

Example
Find the magnetic field at the centre of a current carrying conductor bent in the form of an arc
subtending angle 1 and 2 at the centre.
Solution b

Magnetic field at the centre of arc abc and adc wire of circuit loop
1
a
I1
2
 0 I11 0 I 2 2 Babc I 1 1
Babc = and Badc =  = d
4 r 4 r Badc I 2 2 I2
c

arc length 1
  angle =  = 1

radius 2 2

I1 R2
 V = I1R1 = I2R2  =
I2 R1
I1 
 = 2
( R =  R )
I2 1 A

Babc   
 = 2

1

Badc  1  2 
B1 1
 =
B 2 1

PHYSICS WALLAH 7
Magnetic Effect of Current

HELMHOLT'Z COILS ARRANGEMENT


This arrangement is used to produce uniform magnetic field of short range. It consists:-
R
(N, I, R) (N, I, R)
ACW ACW
O1 O2
M

(uniform magnetic field


of short range BHC)
P S Q

O1 M O
R/ 2 R/ 2 2

• Two identical co–axial coils (N, I, R same)


• Placed at distance (center to center) equal to radius ('R') of coils
• Planes of both coils are parallel to each other.
• Current direction is same in both coils (observed from same side) otherwise this arrangement is
not called "Helmholtz coil arrangement".

Example
A pair of stationary and infinitely long bent wires is placed in the x-y plane as shown in fig.
The wires carry currents of 10 ampere each as shown. The segments L and M are along the x-axis.
The segments P and Q are parallel to the y-axis such that OS = OR = 0.02 m. Find the
magnitude and direction of the magnetic induction at the origin O.
I Q

R O S

L M

P I

Solution
As point O is along the length of segments L and M so the field at O due to these segments will be
0 I ˆ  I
zero. Further, as the point O is near one end of a long wire, BR = BP + BQ = ( k ) + 0 ( kˆ )
4 d 4 d
[as RO = SO = d]
  2I  2  10 ˆ Wb
so, BR = 0   ( kˆ ) Substituting the given data, B R = 10–7 × ( k ) = 10–4 2 ( kˆ )
4  d  0.02 m
–4
B = 10 T and in (+z) direction.

Example
Calculate the field at the centre of a semi-circular wire of radius R in situations depicted in
figure (i), (ii) and (iii) if the straight wire is of infinite length.
I a I a
b I
R
R R
a c b I O I
O
I O I b
I I
c c
R R
(i)
(ii) (iii)

Solution
The magnetic field due to a straight current carrying wire of infinite length, for a point at a
0 I
distance R from one of its ends is zero if the point is along its length and if the point is on
4 R
PHYSICS WALLAH 8
Magnetic Effect of Current

0 I
a line perpendicular to its length while at the centre of a semicircular coil is so net magnetic
4R
→ → → →
field at the centre of semicircular wire is B R = B a + B b + B c
0 I 0 I
(i) B R = 0 +  +0= ( into the page)
4 R 4R
0 I 0 I 0 I 0 I
(ii) B R = + + = [ + 2] (out of the page)
4 R 4 R 4 R 4 R
0 I 0 I 0 I 0 I
(iii) B R = + + = [ – 2] (in to the page)
4 R 4 R 4 R 4 R

Example
Calculate the magnetic induction at the point O, if the current carrying wire is in the shape
shown in figure. The radius of the curved part of the wire is a and linear parts are assumed to
be very long and parallel.
b
c I d

a e

Solution
Magnetic induction at the point O due to circular portion of the wire
µ0 I   0i 3 3
B1 = =   (out of the page) ( = )
4 R 4 a 2 2
Magnetic induction at O due to wire cd will be zero since O lies on the line cd itself when
extended backward.
Magnetic induction at O due to infinitely long straight wire ae is
µ0 i  µ0 i     µ0 i
 B2 = [sin 1 + sin  2 ] where r = a, 1 = 0, 2 =  B2=  sin 0  + sin    =
4 r 2 4 a   2  4a
Because both the fields are in same direction i.e. perpendicular to plane of paper and directed
 i  3 
upwards, hence the resultant magnetic induction at O is B = B1 + B2 = 0  + 1 
4 a  2 
Example
In the frame work of wires shown in figure, a current i is allowed to flow. Calculate the magnetic
induction at the centre O. If angle  is equal to 90°, then what will be the value of magnetic
induction at O ?
B i
A R2
C

D
O
i R1
E

Solution
 0i
Magnetic induction at O due to the segment BC is B1 = 
4  R2
 0i
Similarly, the magnetic induction at O due to circular segment AED is B2 = (2  −  ) 
4 R1
Magnetic field due to segments AB and CD is zero, because point 'O' lies on axis of these parts.

PHYSICS WALLAH 9
Magnetic Effect of Current

 0i   2 −  
Hence resultant magnetic induction at O is B = B1 + B2 =  + ,
4   R2 R1 

  0i   3   0i  1 3 
If  = 90° = , then B =  +  =  + 
2 4   2 R2 2 R1  8  R2 R1 

Example
Two concentric circular coils X and Y of radii 16 cm and 10 cm respectively lie in the same
vertical plane containing the north-south direction. Coil X has 20 turns and carries a current of
16 A; coil Y has 25 turns and carries a current of 18 A. The sense of the current in X is
anticlockwise, and in Y clockwise, for an observer looking at the coils facing the west.
What is the magnitude and direction of the magnetic field at their common centre
(i) Due to coil X alone ? (ii) Due to coil Y alone ? (iii) Due to both the coils ?
Solution
According to the figure the magnitude of the magnetic field at the centre of coil X is
0 Ix N x 4   10 –7 16  20
Bx = = × = 4 × 10–4 T N
2 rx 2 0.16
Ix Coil X
Coil Y
Since the current in coil X is anticlockwise, the direction of Bx W E
By Bx
is towards the east as shown in figure. Iy

The magnitude of magnetic field at the centre of the coil Y


S
 0 Y N Y 4   10 −7 18  25
is given by BY = = × = 9 × 10–4 T
2 rY 2 2

 since the current in coil Y is clockwise, the direction of field BY is towards the west (see
fig.). Since the two fields are collinear and oppositely directed. The magnitude of the resultant
field = difference between the two fields and its direction is that of the bigger field. Hence the
net magnetic field at the common centre is 5  × 10– 4 T and is directed towards the west.

Example
A long wire bent as shown in the figure carries current I. If the radius of the semi-circular
portion is "a" then find the magnetic induction at the centre C.
Z I

a Y
O
C
X I I

Solution
0 I
Due to semi circular part B1 =
4a
( −iˆ )
0 I  I  I
Due to parallel parts of currents B2 = 2  ( − kˆ ) , Bnet = BC = B1 + B2 = 0 ( −iˆ) + 0 ( − kˆ )
4 a 4a 2 a
0 I
Magnitude of resultant field B = B12 + B22 = 2 + 4
4 a
PHYSICS WALLAH 10
Magnetic Effect of Current

Example
A piece of wire carrying a current of 6 A is bent in the form of a circular arc of radius 10.0 cm,
and it subtends an angle of 120° at the centre. Find the magnetic field due to this piece of wire
at the centre.
Solution
I
120°

µ0 I  2
Magnetic field at centre of arc B = ,  = 120° = rad
4 R 3
0 I 2 0 I 4   10 −7  6  100
B=  = = T = 12.57 µT
4 R 3 6R 6  10

Example
An infinitely long conductor as shown in fig. carrying a current I with a semicircular loop on
X-Y plane and two straight parts, one parallel to x-axis and another coinciding with Z-axis.
What is the magnetic field induction at the centre C of the semi-circular loop.
Y
I

I C r

O X
I
Z
Solution
The magnetic field induction at C due to current through straight part of the conductor parallel
to X-axis is
0  I   0 I → 0 I ˆ
B1 =  sin + sin 0  = acting along + Z direction. i.e. B 1 = k
4 ( r / 2 )  2  2 r 2 r
The magnetic field induction at C due to current through the semi-circular loop in X-Y plane is
0 I 0 I 0 I ˆ
B2 = ( ) = acting along + Z-direction i.e. B2 = k
4 ( r / 2 ) 2 r 2r
The magnetic field induction at C due to current through the straight part of the conductor
coinciding with Z-axis is
 I     I  I
B3 = 0  sin + sin 0  = 0 acting along (–X)-axis i.e. B3 = 0 ( −iˆ )
4 r / 2  2  2 r 2 r
Total magnetic field induction at C is
0 I ˆ  I  I  I
B = B1 + B2 + B3 = k + 0 kˆ – 0 iˆ = 0  (1 +  ) kˆ − iˆ 
2 r 2 2 r 2 r  

Example
A conductor carrying a current i is bended as shown in figure. Find the magnitude of magnetic
field at the origin
Solution
y
i 2
1 i
r
3 x
O i
z

 0i 1   0i  ˆ
Field at O due to part 1 B1 =
4 r
( −iˆ ) Field at O due to part 2 B2 = 
4  2r 
( )
 −k 

PHYSICS WALLAH 11
Magnetic Effect of Current

  Wire 3 passes through origin when it is extended backwards B3 = 0

 0 i  iˆ kˆ 
B0 = B1 + B2 + B3 = −  + 
4r   2

Example
A conductor of length 0.04 m is tangentially connected to a circular loop of radius 0.03 m perpendicular
to its plane. Find the magnetic field induction at the centre of the loop if 4 ampere current is
passed through the conductor as shown in fig.
0.03m
4A 

0.04m

m
05
Solution 0.

Magnetic field induction at the centre of the loop due to the straight current-carrying conductor,
0 I 4   10 −7  4  0.04 
B= sin 1 + sin  2  =  sin 0 +  = 1.07 × 10–5T
4 r 4   0.03  0.05 
Magnetic fields due to the two halves of the loop are equal in magnitude and opposite in
direction. So, the magnetic field induction due to the loop at the centre of the loop is zero. So,
the magnetic field induction at the centre of the loop is 1.07 × 10–5T.

Example
Figure shows a right-angled isosceles triangle PQR having its base equal to a.
A current of I ampere is passing downwards along a thin straight wire cutting the plane of the paper
normally as shown at Q. Likewise a similar wire carries an equal current passing normally
upwards at R. Find the magnitude and direction of the magnetic induction at P. Assume the
wires to be infinitely long.
P

90° r
r

Q 45° R
a

Solution
a
Let r = PQ = PR and a2 = r2 + r2 = 2r2 or r =
2

0 I 2 0 I 0I
Magnetic induction at P due to conductor at Q is B1 = = = (along PR)
2 r 2 a 2 a
0I
Magnetic induction at P due to conductor at R is B2 = (along PQ)
2 a
2 2
 0 I   0 I  2 0 I 0 I
Now, resultant of these two is B = B + B 2 2
=   +  = =
a
1 2
 2 a   2 a  2 a

The direction of B is towards the mid-point of the line QR.

PHYSICS WALLAH 12
Magnetic Effect of Current

Do Yourself – 1
(i) A charged particle of mass 5 mg and charge q = +2C has velocity v = 2iˆ − 3jˆ + 4kˆ .
Find out the magnetic force on the charged particle and its acceleration at this instant
due to magnetic field B = 3ˆj − 2kˆ . v and B are in m/s and Wb/m2 respectively.

(ii) A charged particle of charge 2C thrown vertically upwards with velocity 10 m/s. Find
the magnetic force on this charge due to earth’s magnetic field. Given vertical
component of the earth = 3T and angle of dip = 37º.
(iii) Find resultant magnetic field at ‘C’ in the figure shown.
S R

C a

P i Q
3a

(iv) Figure shows a square loop made from a uniform wire. Find the magnetic field at the
centre of the quare if a battery is connected between the points B and D as shown in the
figure

(v) In the figure shown there are two parallel long wires (placed in the plane of paper) are
carrying currents 2 and  consider points A, C, D on the line perpendicular to both the wires
and also in the plane of the paper. The distances are mentioned. Find (i) B at A, C, D
1 2


A 2 C D

a a/2 a
a

(ii) Position of point on line A C D where B is O.


(vi) Shown in the figure is a conductor carrying a current I. The magnetic field intensity at
the point O(common centre of all the three arcs) is :

r 

O
5µ 0 I µ 0 I 11µ 0 I
(A) (B) (C) (D) zero
24 r 24 r 24  r

PHYSICS WALLAH 13
Magnetic Effect of Current

AMPERE'S CIRCUITAL LAW


Ampere's circuital law state that line integral of the magnetic field around any closed path in
free space or vacuum is equal to 0 times of net current or total current which crossing through
the area bounded by the closed path. Mathematically B.d =  0 I
I4

circulation
ACW
I3
I5
I1 I2
Positive
 I=(I1− I2+I3) Negative

This law independent of size and shape of the closed path.


Any current outside the closed path is not included in writing the right hand side of law
Note :
• This law suitable for infinite long and symmetrical current distribution.
• Radius of cross section of thick cylindrical conductor and current density must be given to
apply this law.
MAGNETOMOTIVE FORCE (M.M.F.)

B.d =  0 I , where B =  0 H ,  0 H .d =  0 I  H .d = I

The line integral of magnetizing field around any closed path is equal to net current crossing
through the area bounded by the closed path, also called 'magneto motive force'. Magneto
motive force (M.M.F.) =  H .d

APPLICATION OF AMPERE'S CIRCUITAL LAW


• Magnetic field due to infinite long thin current carrying straight conductor
Consider a circle of radius 'r'. Let XY be the small element of length d. B and d are in the
same direction because direction of along the tangent of the circle. By
I
A.C.L.  B .d = 0  I ,
r B
O y

 Bd cos  =  0 I (where  = 0°) ACW x


d
I

B d cos 0 =  0 I  B  d =  0 I (where d = 2 r )

0 I
B (2r) = 0I  B=
2 r
• Magnetic field due to infinite long solid cylindrical conductor I

• For a point inside the cylinder r < R, Current from area R2 is = I R
2
I Ir
so current from area r2 is = ( r 2 ) = 1 2 3
R 2
R2
By Ampere circuital law for circular path 1 of radius r
I r2  0 Ir axis
Bin (2r) = 0I' = 0  Bin =  Bin  r Cross-sectional
2 R 2
2
R view

• For a point on the axis of the cylinder (r = 0); Baxis = 0 I


1 2 3
PHYSICS WALLAH 14
R
Magnetic Effect of Current
axis
Cross-sectional
• For a point on the surface of cylinder (r = R) view

By Ampere circuital law for circular path 2 of radius R I


1 2 3
0 I
Bs (2  R) = 0I  Bs = (it is maximum) R
2 R
• For a point outside the cylinder (r > R) :- B
Bmax
By Ampere circuital law for circular path 3 of radius r r 1
 Bout  r
B
0 I 1
Bout (2  r) = 0I  Bout =  Bout  r<R
2 r r r=0 r=R r>R

Magnetic field outside the cylindrical conductor does not depend upon nature
(thick/thin or solid/hollow) of the conductor as well as its radius of cross section.

• Magnetic field due to infinite long hollow cylindrical conductor


• For a point at a distance r such that r < a < b
b a
• For a point at a distance r such that a<r<b
1 2 3
 r 2 − a2 
B2(2r) =0 I'  B2(2r) = 0 I  2 2 
b −a 
Axis
Side View
0 I  r − a 
2 2

B2 =   I
2 r  b 2 − a 2  a 1 2 3

0 I Cross sectional
• For a point at a distance r such that r > b > a, B3 (2r) = 0I  B3 = view
2 r
• For a point at the axis of cylinder r = 0 Baxis = 0

Magnetic field at specific positions for thin hollow cylindrical conductor


I
At point 1 B1 = 0 O 1 2 3

0 I R
At point 2 B2 = (maximum) [outer surface] and
2 R
B2 = 0 (minimum) [inner surface] B

0 I Bout  1r
At point 3 B3 = (for the point on axis Baxis =0)
2 r
r
r=0 r=R
Magnetic field due to an infinite plane sheet of current
dBsin

dB
-z P  P

dB dBcos
x   dBcos

y r

x-axis
x x dBsin

PHYSICS WALLAH 15
Magnetic Effect of Current

An infinite sheet of current lies in x-z plane, carrying current along-z axis. The field at any
point P on y is along a line parallel to x-z plane. We can take a rectangular amperian loop as
shown. If you traverse the loop in clockwise direction, inward current will be positive.
By Ampere circuital law,  B.d = 0enlosed ....(i) a
B
PQRS
P Q
b
Let  represents current per unit length. x-axis

The current enclosed is given by enclosed=a b


S R
B
Now,  B.d =  B.d +  B.d +  B.d +  B.d
PQRS PQ QR RS SR

Now,  B.d =  B.d = 0 as B ⊥ d


QR SP

Also,  B.d +  B.d = 2  B  a as B d( )  2B × a =  a  B = 2


0
0

PQ RS

Example
A long straight solid conductor of radius 5 cm carries a current of 3A, which is uniformly
distributed over its circular cross-section. Find the magnetic field induction at a distance 4 cm
from the axis of the conductor. Relative permeability of the conductor = 1000.
Solution
Imagine a circular path of radius r whose centre lies on the axis of solid conductor such that the
point P lies on it.
I Ir 2
the current threading this closed path I'=  r = 2

R 2 R2 I= 3A

Magnetic field B acts tangential to the amperian circular path at P and is same
in
r
magnitude at every point on circular path. P B

 B.d
R= 5
Using Ampere circuital law = µ0µrI' c m

I
 Ir 2    Ir
 B (2r) =  0  r  2   B = 0 r 2
R  2 R

4   10 −7  1000  3  0.04
 B= = 9.6 × 10–3 T.
2   ( 0.05 )
2

Example
A current I flows along a thin walled tube of radius R with a long longitudinal slit of width
b (<<R). What is the magnetic field induction at a distance r (< R) ?
Solution
Using principle of superposition,
field due to strip in place of slit + field due to tube with slit B = 0
0 I
so B = field due to strip in place of slit=  b
2 r (2 R − b )

PHYSICS WALLAH 16
Magnetic Effect of Current

MAGNETIC FIELD DUE TO SOLENOID


It is a coil which has length and used to produce uniform magnetic field of long range. It
consists a conducting wire which is tightly wound over a cylindrical frame in the form of helix.
All the adjacent turns are electrically insulated to each other. The magnetic field at a point on
the axis of a solenoid can be obtained by superposition of field due to large number of identical
circular turns having their centers on the axis of solenoid.

Magnetic field due to a long solenoid


A solenoid is a tightly wound helical coil of wire. If length of solenoid is large, as compared to
its radius, then in the central region of the solenoid, a reasonably uniform magnetic field is
present. Figure shows a part of long solenoid with number of turns/length n. We can find the
field by using Ampere circuital law.
Consider a rectangular loop ABCD. For this loop

 B .d =  0 ienc
ABCD

Now  B.d =  B.d +  B.d +  B.d +  B.d = Ba


A
a
D
ABCD AB BC CD DA

This is because.  B.d =  B.d = 0, B ⊥ d


b
AB CD
B C
And,  B.d = 0 ( B outside the solenoid is negligible
DA
(Magnetic field lines)
Now, ienc = (n × a) × i s   × a = 0(n × a × i)   = 0ni

Finite length solenoid :


Its length and diameter are comparable.

2 r
B
1 (uniform)
E P M E'


End-I End-II

By the concept of BSL magnetic field at the axial point 'P' obtained as :
 0 nI
BP = (cos 1 − cos  2 ) Angle 1 and 2 both measured in same sense from the axis of the
2
solenoid to end vectors.

Infinite length solenoid :


Its length very large as compared to its diameter i.e. ends of solenoid tends to infinity.
(a) Magnetic field at axial point which is well inside the solenoid
 0 nI  0 nI
1  0° and 2  180°  B  [cos0° – cos180°]  [(1) – (–1)]  0 nI
2 2
(b) Magnetic field at both axial end points of solenoid
 0 nI  0 nI  0 nI
1 = 90° and 2  180°  B  [cos90° – cos180°]  [(0) – (–1)]
2 2 2
PHYSICS WALLAH 17
Magnetic Effect of Current

Example
The length of solenoid is 0.1m, and its diameter is very small. A wire is wound over it in two
layers. The number of turns in inner layer is 50 and that of outer layer is 40. The strength of
current flowing in two layers in opposite direction is 3A. Then find magnetic induction at the
middle of the solenoid.
Solution
Direction of magnetic field due to both layers is opposite, as direction of current is opposite so
N1 N2
Bnet = B1 – B2 = 0n1I1 – 0n2I2 = 0 I – 0 I (I1= I2 = I)

0 I 4   10 –7  3
= (N1 – N2) = (50 – 40) = 12 × 10–5 T
0 1
Example
A closely wound, solenoid 80 cm. long has 5 layers of winding of 400 turns each. The
diameter of the solenoid is 1.8 cm. If the current carried is 8.0A. Estimate the magnetic field
(a) Inside the solenoid (b) Axial end points of the solenoid
Solution
(a) Magnetic field inside the solenoid
N 4   10 −7  2000  8
Bin = 0nI = 0 I, (N=400× 5= 2000) = = 8 × 10–3 T
(80  10 −2 )

 0 nI 8   10 −3
(b) Magnetic field at axial end points of solenoid Bends = = = 4 × 10–3 T
2 2
Example
A straight long solenoid is produced magnetic field 'B' at its centre. If it cut into two equal
parts and same number of turns wound on one part in double layer. Find magnetic field
produced by new solenoid at its centre.
Solution
Magnetic field produced by a long solenoid is B = 0nI, where n = N/

 Same number of turns wound over half length

 N    0 NI 
 Magnetic field produced by new solenoid is B' = 0  I = 2  = 2B
 /2  
Example
Find out magnetic field at axial point ‘P’ of solenoid shown in figure (where turn density ‘n’
and current through it is I)

30° 60°
P

Solution
end end
30° 60°
P

Magnetic field at point ‘P’ due to finite length solenoid


PHYSICS WALLAH 18
Magnetic Effect of Current

 0 nI
BP = [cos 1 – cos 2], where 1 = 30° (CW),
2
 0 nI
2 = (180°–60°) = 120° (CW) = [cos 30°–cos 120°]
2
 0 nI  3  1    nI
=  −  −   = 0 ( 3 + 1)
2  2  2  4

Example
A uniform magnetic field exists in a region. A current carrying wire is placed in x-y plane as
shown. Find the force acting on part AB of the wire.
y

B=B0k
a

a a

i
a a B
A
x

Solution
The conductor consists of 5 straight sections viz AC, CD, DE, EF and FB as shown.
As the field is uniform, force on the sections is given by
( ) (
F = i iˆ  B . Thus, F = i aiˆ  B kˆ = − B iajˆ
AC 0 ) 0

( ) ( )
FCD = i ajˆ  B0 kˆ = B0iaiˆ , FDE = i aiˆ  B0 kˆ = − B0iajˆ , FEF = i − ajˆ  B0 kˆ = − B0iaiˆ ( )
FFB = i ( ajˆ  B kˆ ) = − B iajˆ Net force, F = F
0 0 AC + FCD + FDE + FEF + FFB = − 3 B0iajˆ

CURRENT CARRYING CONDUCTOR IN MAGNETIC FIELD


When a current carrying conductor placed in magnetic field, a magnetic force exerts on each
free electron which are present inside the conductor. The resultant of these forces on all the
free electrons is called magnetic force on conductor.
• Magnetic force on current element
Through experiments Ampere established that when current element I d is placed in magnetic

field B , it experiences a magnetic force dFm = I ( d  B )


Id
→  (Cw) →
dFm x B
(external)
• Current element in a magnetic field does not experience any force if the current in it is
parallel or anti–parallel with the field  = 0° or 180° dFm = 0 (min.)
• Current element in a magnetic field experiences maximum force if the current in it is
perpendicular with the field  = 90° dFm = BId (max.)
• Magnetic force on current element is always perpendicular to the current element vector
and magnetic field vector.

dFm ⊥ I d and dFm ⊥ B (always)
PHYSICS WALLAH 19
Magnetic Effect of Current

• Total magnetic force on straight current carrying conductor in uniform magnetic field given
as
f
f 
Fm =  dFm   d  = I × B , Fm = I ( L  B )
i i 
I
i f
L N
L
f

Where L =  d , vector sum of all length elements from initial to final point, which is in
i

accordance with the law of vector addition and | L | = length of the conductor.
• Total magnetic force on arbitrary shape current carrying conductor in uniform magnetic
field is
f
f 
 m   d  × B , Fm = I ( L  B ) (L = ab)
dF = I
i i 
I
Initial Final
point a point
L b
f
Where L = i
d , vector sum of all length elements from initial to final point or displacement

between free ends of an arbitrary conductor from initial to final point.

GOLDEN KEY POINT


• A current carrying closed loop (or coil) of any shape placed in uniform magnetic field then
no net magnetic force act on it (Torque may or may not be zero)

i=f
f →
L =  i
d = 0 or d =0

So net magnetic force acting on a current carrying closed loop Fm = 0 (always)


• When a current carrying closed loop (or coil) of any shape placed in non uniform magnetic
field then net magnetic force is always acts on it (Torque may or may not be zero)
R

R R
i i

Example
A wire bent as shown in fig carries a current i and is placed in a uniform field of magnetic
induction that emerges from the plane of the figure. Calculate the force acting on the wire.
Solution
The total force on the whole wire is
Fm = I| L |B = I(R + 2R + R)B = 4RIB

PHYSICS WALLAH 20
Magnetic Effect of Current

Example
A square of side 2.0 m is placed in a uniform magnetic field v in a direction perpendicular to
the plane of the square inwards. Equal current i = 3.0 A is flowing in the directions shown in
figure. Find the magnitude of magnetic force on the loop.

x x x Bx
C D
x x x x

x x x x

xA x x E x
Solution
Net force on the loop = 3 ( FAD ) 
  Force on wire ACD = Force on AD = Force on AED

 Fnet = 3(i) (AD) (B) = (3) (3.0) (2 2 )(2.0) N = 36 2 N. Direction of this force is towards EC.

Example
A metal rod of mass 10 gm and length 25 cm is suspended on two springs as shown in figure.
The springs are extended by 4 cm. When a 20 ampere current passes through the rod it rises by
1 cm. Determine the magnetic field assuming acceleration due to gravity to be 10 m/s2.
x
x x k
k
F=bil x
T T
x x
Mg I x
Solution
Let tension in each spring is = T0
Initially the rod will be in equilibrium if 2T0 = Mg then T0 = kx0 ...(i)
Now when the current I is passed through the rod it will experience a force
F = BIL vertically up; so in this situation for its equilibrium,
2T + BIL = Mg with T = kx...(ii) (x = 4 – 1 = 3cm)
T Mg − BIL x BIL
So from eq. (i) and eq.(ii) =  = 1−
T0 Mg x0 Mg
Mg ( x0 − x ) 10  10 −3  10  3  10 −2
B= = = 1.5 × 10–2T
ILx0 20  25  10 −2  4  10 −2

Example
Two conducting rails are connected to a source of emf. and form an incline as shown in fig. A
bar of mass 50 g slides without friction down the incline through a vertical magnetic field B. If
the length of the bar is 50 cm and a current of 2.5 A is provided by the battery, for what value
of B will the bar slide at a constant velocity ? [g = 10 m/s2]
I
v= I
co B
ns
t

PHYSICS WALLAH 21
Magnetic Effect of Current

Solution
Fm
co R
Force on current carrying wire F = BIL s
 m
gs
Fm in
The rod will move down the plane with constant velocity only if 

mg
F cos  = mg sin   BIL cos  = mg sin   = 

mg 50  10 −3  10 3
or, B = tan  = −2
 = 0.3 T
IL 2.5  50  10 4

Example
A wire PQ of mass 10g is at rest on two parallel metal rails. The separation between the rails is
4.9 cm. A magnetic field of 0.80 tesla is applied perpendicular to the plane of the rails, directed
downwards. The resistance of the circuit is slowly decreased. When the resistance decreases to
below 20 ohm, the wire PQ begins to slide on the rails. Calculate the coefficient of friction
between the wire and the rails.
P
6V
4.9cm

Solution
Wire PQ begins to slide when magnetic force is just equal to the force of friction, i.e.,
E 6
µ mg = i B sin  ( = 90°) so i = = = 0.3 A
R 20

i B ( 0.3 ) ( 4.9  10 −2 ) ( 0.8 )


so µ = = = 0.12
mg (10  10 −3 ) ( 9.8 )

Example
A wire abcdef with each side of length '' bent as shown in figure and carrying a current I is

placed in a uniform magnetic field B parallel to +y direction. What is the force experienced by
the wire.
Z
c
 
b a
d Y
e B
f
X 

Solution
Magnetic force on wire abcdef in uniform magnetic field is Fm = I ( L  B ) ,

L is displacement between free ends of the conductor from initial to


final point. L = () iˆ and B =(B) ĵ ;

Fm = I ( L  B ) = BIL (iˆ  ˆj ) = BI ( kˆ ) = BI, along +z direction

PHYSICS WALLAH 22
Magnetic Effect of Current

Do Yourself – 2 :
(i) A solenoid of length 0.4 m and diameter 0.6 m consists of a single layer of 1000 turns
of fine wire carrying a current of 5.0 × 10–3 ampere. Find the magnetic field on the axis
V −s
at the middle and at the ends of the solenoid. (Given 0 = 4 × 10–7 ).
A−m
(ii) A thin solenoid of length 0.4 m and having 500 turns of wire carries a current 1A; then
find the magnetic field on the axis inside the solenoid.

(iii) If B and E denote induction of magnetic field and energy density at the mid-point of a
long solenoid carrying a current i, then which of the following graph is/are correct –
B E E E

(A) (B) (C) (D)

i i B B

(iv) A solenoid is connected to a source of constant emf for a long time. A soft iron piece is
inserted into it. Then –
(A) self inductance of the solenoid gets increased
(B) flux linked with the solenoid increases hence steady state current gets decrease
(C) energy stored in the solenoid gets increased
(D) magnetic moment of the solenoid increased

MAGNETIC FORCE BETWEEN TWO PARALLEL CURRENT CARRYING CONDUCTORS


Like currents unlike currents
1 2 1 2

I1 I1 Repulsion

B2 B1 dF12 B2 B1 dF21
dF12 dF21

d d

The net magnetic force acts on a current carrying conductor due to its own field is zero. So
consider two infinite long parallel conductors separated by distance 'd' carrying currents I1 and I2.
Magnetic field at each point on conductor (ii) due to current I1 is
 I
B1 = 0 1 [uniform field for conductor (2)]
2d
Magnetic field at each point on conductor (i) due to current I2 is
 I
B2 = 0 2 [Uniform field for conductor (1)]
2d
consider a small element of length 'd' on each conductor. These elements are right angle to the
external magnetic field, so magnetic force experienced by elements of each conductor given as
 0 I2 
dF12 = B2 I1 d =   I1 d ...(i) (Where I1 d ⊥ B2)
 2d 
  0 I1 
dF21 = B1 I2 d =   I2 d ... (ii) (Where I2 d ⊥ B1)
 2d 
PHYSICS WALLAH 23
Magnetic Effect of Current

Where dF12 is magnetic force on element of conductor (i), due field of conductor (i) and dF21 is
magnetic force on element of conductor (ii), due to field of conductor (i).
dF12 dF21  II
Magnetic force per unit length of each conductor is = = 0 1 2
d d 2d
 0 I1 I 2 2 I1 I 2
f= N/m (in S.I.) f= dyne/cm (In C.G.S.)
2d d
Definition of ampere :
Magnetic force/unit length for both infinite length conductor gives as 1 2

1A 1A
 0 I1 I 2 (4   10 −7 ) (1) (1) –7
f= = = 2 × 10 N/m
2d 2  (1)
1m

'Ampere' is the current which, when passed through each of two parallel infinite long straight
conductors placed in free space at a distance of 1 m from each other, produces between them a
force of 2 × 10–7 N/m
L

I1 I2

d
(source) N
 0 I1 I 2
• Force scale f = is applicable when at least one conductor must be of infinite length so it
2d
behaves like source of uniform magnetic field for other conductor.
  0 I1 I 2 
Magnetic force on conductor 'LN' is FLN = f ×  FLN =  
 2d 
• Equilibrium of free wire
Case I : Upper wire is free : Consider a long horizontal wire which is rigidly fixed another
wire is placed directly above and parallel to fixed wire.
I2 fm
(Stable (free) Finite length (m,)
Equilibrium) g
h

(fixed) I1 Infinite length


– +
(Source)

 0 I1 I 2
Magnetic force per unit length of free wire fm = , and it is repulsive in nature because
2 h
currents are unlike.
Free wire may remains suspended if the magnetic force per unit length is equal to weight of its
unit length
 0 I1 I 2 m
At balanced condition fm = W'. Weight per unit length of free wire = = g (stable
2 h
equilibrium condition)
If free wire is slightly plucked and released then it will executes S.H.M. in vertical plane.
h
The time period of motion is T = 2 
g
PHYSICS WALLAH 24
Magnetic Effect of Current

Case II : Lower wire is free : Consider a long horizontal wire which is rigidly fixed. Another
wire is placed directly below and parallel to the fixed wire.

I1 (Source) Infinite length


– +
(fixed)
d (depht)

I2 fm
(Unstable (free) Finite length (m,)
Equilibrium)
g

 0 I1 I 2
Magnetic force per unit length of free wire is fm = , and it is attractive in nature because
2 d
currents are like.
Free wire may remains suspended if the magnetic force per unit length is equal to weight of its
unit length
At balanced condition fm = W'
 0 I1 I 2 m
Weight per unit length of free wire = g (unstable equilibrium condition)
2 d
Example
Two horizontal parallel straight conductors, each 20 cm long, are arranged one vertically above
the other and carry equal currents in opposite directions. The lower conductor is fixed while
the other is free to move in guides remaining parallel to the lower. If the upper conductor
weights 1.20 g, what is the approximate current that will maintain the conductors at a distance
0.75 cm apart.
Solution
In equilibrium magnetic force Fm will balance weight mg.
 0i 2
So mg = Fm  mg =
2 d
Fm
P i Q

mg
d

R i S
−3 −2
2mgd 2   1.2  10  9.8  0.75  10
i= = = 2205 = 47 A
0 4   10 −7  20  10 −2

MAGNETIC DIPOLE MOMENT


A magnetic dipole consists of a pair of magnetic poles of equal and opposite strength separated
by small distance. Ex. Magnetic needle, bar magnet, solenoid, coil or loop.
• Magnetic moment of Bar magnet
Neutral point A
Magnetic -m +m
axis

S  N Cross sectional
0 view

PHYSICS WALLAH 25
Magnetic Effect of Current

The magnetic moment of a bar magnet is defined as a vector quantity having magnitude equal
to the product of pole strength (m) with effective length () and directed along the axis of the
magnet from south pole to north pole.
It is an axial vector
M= m 
2
S.I. unit : - A.m

GOLDEN KEY POINTS


• Attractive property: A bar magnet attracts certain magnetic substances (e.g. Iron dust). The
attracting power of the bar magnet is maximum at two points near the ends called poles. So the
attracting power of a bar magnet at its poles called 'pole strength'
• The 'pole strength' of north and South Pole of a bar magnet is conventionally represented by
+m and –m respectively.
• The 'pole strength' is a scalar quantity with S.I. unit A–m.
• The 'pole strength' of bar magnet is directly proportional to its area of cross section. m  A
• The attracting power of a bar magnet at its centre point is zero, so it is called 'neutral point'.
• Magnetic poles are always exists in pairs i.e. mono pole does not exist in magnetism. So Gauss
law in magnetism given as  B  ds =0
• Effective length or magnetic length: – It is distance between two poles along the axis of a bar
magnet. As pole are not exactly at the ends, the effective length () is less than the geometrical

length (0) of the bar magnet.  ~− 0  91 0

• Inverse square law (Coulomb law) : The magnetic force between two isolated magnetic
poles of strength m1 and m2 lying at a distance 'r' is directly proportional to the product of pole
strength and inversely proportional to the square of distance between their centers. The
magnetic force between the poles can be attractive or repulsive according to the nature of the
poles.
Fm  m1m2 µ0
(S.I.)
mm 4
Fm=k 12 2 where k
Fm  12 r 1 (C.G.S.)
r
Inverse square law of Coulomb in magnetism is applicable only for two long bar magnets
because isolated poles cannot exist.
• If a magnet is cut into two equal parts along the length then pole strength is reduced to half and
m M
length remains unchanged. New magnetic dipole moment M'=m'() =  = .
2 2
The new magnetic dipole moment of each part becomes half of original value.
m' = (m/ 2), × '= 
 S N m' = m '=/ 2
-m +m
M S N S N
S N S N
 M
M= m × M'= m× =
M' = (m/ 2) × '= M/ 2 2 2

• If a magnet is cut into two equal parts transverse to the length then pole strength remains
  M
unchanged and length is reduced to half. New magnetic dipole moment M ' = m   = .
2 2
The new magnetic dipole moment of each part becomes half of original value.
PHYSICS WALLAH 26
Magnetic Effect of Current

• The magnetic dipole moment of a magnet is equal to product of pole strength and distance
between poles. M = m



N S
N S

N S

• As magnetic moment is a vector, in case of two magnets having magnetic moments M 1 and M2
with angle  between them, the resulting magnetic moment.
1/ 2  M 2 sin  
M =  M 12 + M 22 + 2 M 1M 2 cos   with tan  =  
 M 1 + M 2 cos  
Example
The force between two magnetic poles in air is 9.604 mN. If one pole is 10 times stronger than
the other, calculate the pole strength of each if distance between two poles is 0.1 m?
Solution
µ0 m1m2 −3 10 −7  m  10 m
Force between poles F = or 9.604  10 = or m2 = 96.04 N2T–2
4 r 2 0.1  0.1
 m = 9.8 N/T
So strength of other pole is 9.8 × 10 = 98 N/T

Example
A steel wire of length L has a magnetic moment M. It is then bent into a semicircular arc. What
is the new magnetic moment?
Solution
M
If m is the pole strength then M = m.L  m =
L
L
If it is bent into a semicircular arc then L= r  r =

M L 2M
So new magnetic moment M ' = m  2 r =  2 =
L  
Example
Two identical bar magnets each of length L and pole strength m are placed at right angles to
each other with the north pole of one touching the south pole of other. Evaluate the magnetic
moment of the system.
Solution
M1 =M2 = mL S
M2
 
 M R = M 12 + M 22 + 2 M 1 M 2 cos = 2 mL M1
2 MR
NS N M1
M sin 90 M2
and tan  = = 1 i.e. = tan–11 = 45°
M + M cos 90

PHYSICS WALLAH 27
Magnetic Effect of Current

MAGNETIC MOMENT OF CURRENT CARRYING COIL (LOOP)


Current carrying coil (or loop) behaves like magnetic dipole. The face of coil in which current
appears to flow anti clock wise acts as north pole while face of coil in which current appears to
flow clock wise acts as south pole.

• A loop of geometrical area 'A', carries a current 'I' then magnetic moment of coil M = I A
• A coil of turns 'N', geometrical area 'A', carries a current 'I' then magnetic moment M = N IA
Magnetic moment of current carrying coil is an axial vector M = NIA where A is a area vector
perpendicular to the plane of the coil and along its axis. SI UNIT : A-m2 or J/T
ACW CW

M M

Direction of M find out by right hand thumb rule


• Curling fingers  In the direction of current • Thumb  Gives the direction of M
For a current carrying coil, its magnetic moment and magnetic field vectors both are parallel
axial vectors.

Example
Find the magnitude of magnetic moment of the current carrying loop ABCDEFA. Each side of
the loop is 10 cm long and current in the loop is i = 2.0 A
C D

B E

A F

Solution
By assuming two equal and opposite currents in BE, two current carrying loops (ABEFA and
BCDEB) are formed. Their magnetic moments are equal in magnitude but perpendicular to
each other.
C D

B E
A F

Hence, Mnet = M2+M2 = 2M


where M = iA = (2.0)(0.1)(0.1) = 0.02 A–m2
 Mnet = ( 2 )(0.02) A-m2 = 0.028 A–m2

PHYSICS WALLAH 28
Magnetic Effect of Current

Example
The wire loop PQRSP formed by joining two semicircular wires of radii R 1 and R2 carries a
current I as shown in fig. What is the magnetic induction at the centre O and magnetic moment
of the loop in cases (A) and (B) ?
(A) (B)
I
R2 R2

O
R1 S R R1 I Q P
S R O Q P

Solution
As the point O is along the length of the straight wires, so the field at O due to them will be
zero and hence.
 0  I I  0 1 1 
(A) B =  +  i.e., I  − 
4   R2 R1  4   R1 R2 
1 1  1
& M = NIS =1 × I  R22  + R12  = I  R22 − R12  
2 2  2
(B) Following as in case (A), in this situation,
0 1 1  1
B= I  +   and, M = I  R2 + R1  
2 2

4   R1 R2  2

MAGNETIC DIPOLE IN MAGNETIC FIELD


Torque on magnetic dipole
N
Fm=mB
CW
+m

B=(uniform)

S

CW
Fm=mB-m

(a) Bar magnet


 = force × perpendicular distance between force couple
 = (mB) (sin) , where M = m
 = 90°   = MB (maximum)
 = MBsin 
 = 0° or 180° =0 (minimum) Vector form = M B
(b) Coil or Loop I M CW
= M B  = NI ( A  B ) 
 = 90°   = BINA (maximum) O
B(uniform)
 = BINAsin
 = 0° or 180°  = 0 (minimum)
Plane of coil

GOLDEN KEY POINTS


• Torque on dipole is an axial vector and it is directed along axis of rotation of dipole.
• Tendency of torque on dipole is try to align the M in the direction of B or tries to makes the
axis of dipole parallel to B or makes the plane of coil (or loop) perpendicular to B .
Fnet=0 (no translatory motion)
• Dipole in uniform magnetic field
 may or may not be zero (decides by )
Fnet 0 (translatory motion)
• Dipole in non uniform magnetic field
 may or may not be zero (decides by )
PHYSICS WALLAH 29
Magnetic Effect of Current

• When a current carrying coil (or loop) is placed in longitudinal magnetic field then maximum
torque acts on it.  = 90° ( M ⊥ B )
 max = MB = BINA
• When a current carrying coil (or loop) is placed in transverse magnetic field the no torque acts
on it.
( →
 = 0° M B )
or  = 180° ( M anti B )  min = 0

Do Yourself – 3 :
(i) Find the resultant magnetic force and torque on the loop.

(ii) A straight rod of mass m and length L is suspended from the identical springs as shown
in the figure. The spring stretched a distance x0 due to the weight of the wire. The
circuit has total resistance R. When the magnetic field perpendicular to the plane of
paper is switched on, springs are observed to extend further by the same distance. The
magnetic field strength is.........
E

2 mgR mgR mgR mgR


(A) (B) (C) (D)
LE EL 2 LE E

(iii) A metal wire PQ of mass 10gm lies at rest on two horizontal metal rails separated by
4.9 cm. A vertically downward magnetic field of magnitude 0.8 Tesla exists in the
space. The resistance of circuit Rh is slowly decreased and it is found that when the
resistance goes below 20 , the wire PQ starts sliding on the rails. Then coefficient of
friction between wire and rail [Take g = 9.8 m/s2] is-
P
6V Rh

(A) 0.10 (B) 0.12 (C) 0.16 (D) 0.24


(iv) A straight conductor of mass m and carrying a current i is hinged at one end and placed
in a plane perpendicular to the magnetic field B as shown in figure. At any moment if
the conductor is let free, then the angular acceleration of the conductor will be (neglect
gravity) –
× × × ×
Hinged
end
× × × B×
× ×i × ×
× × × ×
L
3iB 2 iB iB 3i
(A) (B) (C) (D)
2m 3 m 2m 2 mB

PHYSICS WALLAH 30
Magnetic Effect of Current

WORK DONE IN ROTATING A MAGNETIC DIPOLE


Work done in rotating a dipole in a uniform magnetic field through small angle 'd'
dW = .d = MBsind
So work done in rotating a dipole from angular position 1 to 2 with respect to the Magnetic
2 2
field direction W = 1
dW = 1
MB sin d  =MB(cos1 – cos2)

• If magnetic dipole is rotated from field direction i.e. 1 = 0° to position 2=


then work done is W = MB (1 – cos) = 2MB sin2 /2
in one rotation = 0° or 360° W = 0 in 1/4 rotation  = 90°  W = MB
in half rotation = 180°  W = 2MB in 3/4 rotation  = 270°  W = MB
• Work done to rotate a dipole in a magnetic field is stored in the form of potential energy of
magnetic dipole.

POTENTIAL ENERGY OF MAGNETIC DIPOLE


The potential energy of dipole defined as work done in rotating the dipole from a direction
perpendicular to the given direction. U = W – W90°  U = MB (1 – cos) – MB = MB cos In
vector form U = − M . B
GOLDEN KEY POINTS
• When M and B are parallel ( = 0°), the dipole has minimum potential energy and it is in
stable equilibrium. U = – MB (minimum)
• When M and B are anti parallel ( = 180°), the dipole has maximum potential energy and it is
in unstable equilibrium. U = MB (maximum)
• When M and B are perpendicular to each other ( = 90°), the dipole has potential energy U=0
and in this situation maximum torque acts on it hence no equilibrium.

Example
A circular coil of 25 turns and radius 6.0 cm, carrying a current of
10 A, is suspended vertically in a uniform magnetic field of magnitude
1.2 T. The field lines run horizontally in the plane of the coil.
Calculate the force and torque on coil due to the magnetic field. In I →
B
I
which direction should a balancing torque be applied to prevent the
coil from turning ?
Solution
Magnetic force Fm= I (d B )
For coil or close loop d =0 so Fm = 0

The torque  on a coil of any shape having N turns and


current  in a magnetic field B is given by  = NIABsin
 = 25 × 10 ×  × 6 × 6 × 10–4 × 1.2 × sin90° = 3.39 N
The direction of  is vertically upwards. To prevent the coil from turning, an equal and opposite
torque must be applied.

PHYSICS WALLAH 31
Magnetic Effect of Current

Example
A uniform magnetic field of 5000 gauss is established along the positive z-direction. A rectangular
loop of side 20 cm and 5 cm carries a current of 10 A is suspended in this magnetic field. What
is the torque on the loop in the different cases shown in the following figures? What is the
force in each case? Which case corresponds to stable equilibrium?
Z Z Z

→ → →
B I B B
I
Y Y Y
O O I O

X (a) X (b) X (c)


Z Z Z

→ → →
B B B

O O Y O Y
30° Y
I
X (d) (e) (f)
Solution
(a) Torque on loop, =BIA sin
Here, =90°; B=5000 gauss=5000× 10–4 tesla = 0.5 tesla
 = 10 ampere, A=20 × 5cm2=100 × 10–4=10–2 m2
Now,  = 0.5 × 10× 10–2=5 × 10–2 Nm It is directed along –y-axis
(b) Same as (a).
(c) =5 × 10–2 Nm along –x-direction
(d)  =5×10–2 N m at an angle of 240° with +x direction.
(e)  is zero. [Angle between plane of loop and direction of magnetic field is 90°]
(f)  is zero.
Resultant force is zero in each case. Case (e) corresponds to stable equilibrium.

Example
A circular coil of 100 turns and having a radius of 0.05 m carries a current of 0.1 A. Calculate
the work required to turn the coil in an external field of 1.5 T through 180° about an axis
perpendicular to the magnetic field? The plane of coil is initially at right angles to magnetic
field.
Solution
Work done W = MB (cos1–cos2) =NAB (cos1–cos2)
 W = Nr B (cos1– cos2) = 100 × 0.1 × 3.14 × (0.05)2 × 1.5 (cos 0° –cos) = 0.2355J
2

Example
A bar magnet of magnetic moment 1.5 JT–1 lies aligned with the direction of a uniform
magnetic field of 0.22 T.
(a) What is the amount of work required to turn the magnet so as to align its magnetic moment.
(i) Normal to the field direction? (ii) Opposite to the field direction?
(b) What is the torque on the magnet in case (i) and (ii)?
Solution
Here, M = 1.5 JT–1, B = 0.22 T.
(a) P.E. with magnetic moment aligned to field = – MB
P.E. with magnetic moment normal to field = 0
P.E. with magnetic moment antiparallel to field = + MB
PHYSICS WALLAH 32
Magnetic Effect of Current

(i) Work done = increase in P.E. = 0 – (–MB) = MB = 1.5 × 0.22 = 0.33 J.


(ii) Work done = increase in P.E. = MB – (–MB) = 2MB = 2 × 1.5 × 0.22 = 0.66 J.
(b) We have  = MBsin
(i)  = MB sin = 1.5 × 0.22 × 1 = 0.33 J. (=90°  sin = 1)
This torque will tend to align M with B.
(ii)  = MB sin = 1.5 × 0.22 × 0=0 ( =180°  sin=0)

Example
A short bar magnet of magnetic moment 0.32 J/T is placed in uniform field of 0.15 T. If the bar
is free to rotate in plane of field then which orientation would correspond to its (i) stable and
(ii) unstable equilibrium? What is potential energy of magnet in each case?
Solution
(i) If M is parallel to B then =0°. So potential energy U = Umin = – MB
Umin = –MB = –0.32 × 0.15 J= –4.8 × 10–2 J (stable equilibrium)
(ii) If M is antiparallel to B then  = ° So potential energy
U = Umax = + MB = + 0.32 × 0.15 = 4.8 × 10–2 J (unstable equilibrium.)

ATOMIC MAGNETISM
An atomic orbital electron, which doing bounded uniform circular motion around nucleus. A
current constitutes with this orbital motion and hence orbit behaves like current carrying loop.
Due to this magnetism produces at nucleus position. This phenomenon called as 'atomic
magnetism.
Bohr's postulates : v

mv 2 kze 2  h  ACW
(i) = (ii) L = mvr = n   , where n = 1, 2,3 ....... +Ze
r r 2
 2  r  e

Fe
Basic elements of atomic magnetism : I(current)
e ev e
(a) Orbital current :- I = ef = = =
T 2 r 2
(b) Magnetic induction at nucleus position :- As circular orbit behaves like current
0 I
carrying loop, so magnetic induction at nucleus position BN =
2r
 0 ef  0e  0 ev  0 e
BN = = = =
2r 2Tr 4 r 2
4 r

(c) Magnetic moment of circular orbit :- Magnetic dipole moment of circular orbit
er 2 e r 2
M = IA where A is area of circular orbit. M = ef ( r )=
evr
2
= =
T 2 2
• Relation between magnetic moment and angular momentum of orbital electron
I(current)

e
M

PHYSICS WALLAH 33
Magnetic Effect of Current

evr m Le
Magnetic moment M =  = (angular momentum = mvr)
2 m 2m
− eL
Vector form M =
2m
For orbital electron its M and L both are antiparallel axial vectors.

BOHR MAGNETON (B)


According of Bohr's theory, angular momentum of orbital electron is given by
nh
L= , where n = 1, 2, 3 ........ and h is plank's constant.
2
eL eh
Magnetic moment of orbital electron is given by M = = n
2m 4m
eh
• If n = 1 then M = , which is Bohr magneton denoted by B
4m
 • Definition of B :
Bohr magneton can be defined as the magnetic moment of orbital electron which revolves
in first orbit of an atom.
eh 1.6  10 −19  6.6  10 −34
 • B = = − 31
= 0.923 × 10–23 A.m.2
4m 4  3.14  9.1  10

• Basic elements of atomic magnetism for first orbit of H-atom (n=1, z = 1)


(a) Accurate form :- (v = 2.18 × 106 m/sec, f = 6.6 × 1015 cy/sec. r = 0.529Å)
  • Orbital current I = 0.96 mA
  • Magnetic induction at nucleus position BN = 12.8 T
  • Magnetic moment of orbital electron M = 0.923 × 10–23 A.m2
(b) Simple form :- (v  2 × 106 m/sec, f  6 × 1015cy/sec, r  0.5Å)
  • Orbital current I  1mA
  • Magnetic induction at nucleus position BN  4T
 • Magnetic moment of orbital electron M = µB A.m2

A NONCONDUCTING CHARGED BODY IS ROTATED WITH SOME ANGULAR SPEED.


In this case the ratio of magnetic moment and angular momentum is constant which is equal to
q
2m
here q = charge and m = the mass of the body.
Example
In case of a ring, of mass m, radius R and charge q distributed on it circumference.
Angular momentum L = I = (mR2)() ... (i)
2
Magnetic moment M = iA = (qf) (R )
   R 2
M = (q)   (R2) = q ...(ii)
 2  2 ++ + + + +q
++ +
+
 M q + +
f= From Eqs. (i) and (ii) = +
+
R
+
+
2 L 2m ++
++ + + + + + + +

PHYSICS WALLAH 34
Magnetic Effect of Current

Although this expression is derived for simple case of a ring, it holds good for other bodies
qL q(I )
also. For example, for a disc or a sphere. M= M= , where L = I 
2m 2m
Rigid body Ring Disc Solid sphere Spherical shell
2
mR 2 2
Moment of inertia (I) mR2 mR2 mR2
2 5 3
qI  q R 2 q R 2 q R 2 q R 2
Magnetic moment=
2m 2 4 5 3

FORCE ON A CHARGED PARTICLE IN A MAGNETIC FIELD


Force experienced by a current element d in magnetic field B is given by
dF = Id × B ....(i)
Now if the current element d is due to the motion of charge particles, each particle having a
charge q moving with velocity through a cross-section A, d = ( nqAv ) d = ( nqdV ) v [with
volume dV=A d]
From eqn (i) we can write dF = ndV q ( v  B )
ndV=the total number of charged particles in volume dV
(n=number of charged particles per unit volume), force on a charged particle

→ → →
1 dF
F = = q ( v B)
n dV

GOLDEN KEY POINT


• The force F is always perpendicular to both the velocity v and the field B
• A charged particle at rest in a steady magnetic field does not experience any force.
If the charged particle is at rest then v = 0 , so v  B = 0
• A moving charged particle does not experience any force in a magnetic field if its motion is
parallel or antiparallel to the field.
q= 0° F F
q v
B
B B
v q O  O 90° v
v q
= 180° q
F= 0 F= vBsin Fmax=qvB
(A) (B) (C)

• If the particle is moving perpendicular to the field. In this situation all the three vectors F , v
and B are mutually perpendicular to each other. Then sin  = max = 1, i.e.,  = 90°,
The force will be maximum Fmax = q v B
• Work done by force due to magnetic field in motion of a charged particle is always zero.
When a charged particle move in a magnetic field, then force acts on it is always perpendicular
to displacement, so the work done, W =  F .ds =  Fds cos 90 = 0 (as  = 90°),
 1 
And as by work-energy theorem W =  KE, the kinetic energy  = mv 2  remains unchanged
 2 
and hence speed of charged particle v remains constant.
However, in this situation the force changes the direction of motion, so the direction of
velocity v of the charged particle changes continuously.

PHYSICS WALLAH 35
Magnetic Effect of Current

MOTION OF A CHARGED PARTICLE IN A MAGNETIC FIELD


Motion of a charged particle when it is moving collinear with the field magnetic field is not
affected by the field (i.e. if motion is just along or opposite to magnetic field) ( F = 0) Only
the following two cases are possible :
• Case  :
When the charged particle is moving perpendicular to the field.
The angle between B and v is =90°. So the force will be maximum (= qvB) and always
perpendicular to motion (and also field); Hence the charged particle will move along a circular
path (with its plane perpendicular to the field). Centripetal force is provided by the force qvB,
mv 2 mv
So = qvB  r =
r qB
v qB
Angular frequency of circular motion, called cyclotron or gyro-frequency. = =
r m
2 m
and the time period, T= = 2 i.e., time period (or frequency) is independent of speed
 qB
of particle and radius of the orbit. Time period depends only on the field B and the nature of
the particle, i.e., specific charge (q/m) of the particle.
This principle has been used in a large number of devices such as cyclotron (a particle
accelerator), bubble-chamber (a particle detector) or mass-spectrometer etc.

• Case  :
The charged particle is moving at an angle  to the field : (  0°, 90° or 180°). Resolving the
velocity of the particle along and perpendicular to the field. The particle moves with constant
velocity v cos  along the field ( no force acts on a charged particle when it moves parallel to
the field).
And at the same time it is also moving with velocity v sin perpendicular to the field due to
which it will describe a circle (in a plane perpendicular to the field)
m ( v sin  )
Radius of the circular path r = and
qB
2 r 2m
Time period T= =
v sin  qB
So the resultant path will be a helix with its axis parallel to the field as shown in fig.
v sin  Helical Path

r
 B
v cos 

2 m
The pitch p of the helix = linear distance travelled in one rotation p=T (vcos) = ( v cos )
qB

PHYSICS WALLAH 36
Magnetic Effect of Current

Example
An electron emitted by a heated cathode and accelerated through a potential difference of
2.0 kV enters a region with uniform magnetic field of 0.15 T. Determine the radius of the
trajectory of the electron if the field is –
(a) Transverse to its initial velocity(b) Makes an angle of 30° with the initial velocity
[Given : me = 9 × 10–31 kg]
Solution
1 2eV 2  1.6  10 −19  2  10 3 8
mv2 = eV  v = = − 31
= × 107 m/s
2 m 9  10 3
mv 9  10 −31  (8 / 3)  10 7
(a) Radius r1 = = −19
= 10–3 m = 1mm
qB 1.6  10  0.15
1
(b) Radius r2 = v = r1sin =1× sin30° =1 × = 0.5 mm
2
MOTION OF CHARGED PARTICLE IN COMBINED ELECTRIC AND MAGNETIC
FIELDS :
Let a moving charged particle is subjected simultaneously to both electric field E and
magnetic field B .
The moving charged particle will experience electric force Fe = qE and magnetic force
→ →
Fm = q ( v  B ) .

Net force on the charge particle F = q ( E + v  B ) "Lorentz-force"

Depending on the direction of v , E and B various situation are possible and the motion in
general is quite complex.
Case  : v , E and B all the three are collinear :
E
v v'
q q
B
As the particle is moving parallel or antiparallel to the field. The magnetic force on it will be
F qE
zero and only electric force will act So, acceleration of the particle a = =
m m
Hence, the particle will pass through the field following a straight line path (parallel to the field)
with change in its speed.
In this situation speed, velocity, momentum and kinetic energy all will change without change
in direction of motion as shown in figure above.
Case  : v , E and B are mutually perpendicular :
E

q Fe q
v v
Fm

B
If in this situation direction and magnitude of E and B are such that

PHYSICS WALLAH 37
Magnetic Effect of Current

F
Resultant force F = Fe + Fm = 0  a = =0
m
Then as shown in fig., the particle will pass through
E
the field with same velocity  Fe=Fm i.e. qE = qvB  v =
B
Example
A beam of protons is deflected sideways. Could this deflection be caused by
(i) a magnetic field (ii) an electric field? If either possible, what would be the difference?
Solution
Yes, the moving charged particle (e.g. proton, -particles etc.) may be deflected sideway either
by an electric or by a magnetic field.
(i) The force exerted by a magnetic field on the moving charged particle is always perpendicular
to direction of motion, so that no work is done on the particle by this magnetic force. That is
the magnetic field simply deflects the particle and does not increase its kinetic energy.
(ii) The force exerted by electric field on the charged particle at rest or in motion is always along
the direction of field and the kinetic energy of the particle changes.
Example
A neutron, a proton, an electron an -particle enter a region of constant C
× × × ×
magnetic field with equal velocities. The magnetic field is along the B
× × × ×
inwards normal to the plane of the paper. The tracks of the particles are A× × × ×
× × × ×
D
shown in fig. Relate the tracks to the particles.
Solution
Force on a charged particle in magnetic field F = q ( v  B )
For neutron q=0, F=0 hence it will pass undeflected i.e., tracks C corresponds to neutron.
If the particle is negatively charged, i.e. electron. F = − e(v  B )
It will experience a force to the right; so track D corresponds to electron.
If the charge on particle is positive. It will experience a force to the left; so both tracks A and B
corresponds to positively charged particles (i.e., protons and -particles). When motion of
charged particle perpendicular to the magnetic field the path is a circle with radius
mv m m  4m  m m m m
R= i.e. r  and as   =  while   =     
qB q  q    2e   q p e  q   q  p
So r > rp track B to -particle and A corresponds to proton.

Example
An electron does not suffer any deflection while passing through a region. Are you sure that
there is no magnetic field? Is the reverse definite?
Solution
If electron passing through a certain region does not suffer any deflection, then we are not sure
that there is no magnetic field in that region. This is due to that electron suffers no force when
it moves parallel or antiparallel to magnetic field. Thus the magnetic field may exist parallel or
antiparallel to the direction of motion of electron.
The reverse is not true since an electron can also be deflected by the electric field.
PHYSICS WALLAH 38
Magnetic Effect of Current

Example
In a chamber, a uniform magnetic field of 8 × 10–4 T is maintained. An electron with a speed of
4.0 × 106m/s enters the chamber in a direction normal to the field.
(a) Describe the path of the electron.
(b) What is the frequency of revolution of the electron?
(c) What happens to the path of the electron if it progressively loses its energy due to
collisions with the atoms or molecules of the environment?
Solution
mv 9.1  10 −31  4  10 6
(a) The path of the electron is a circle of radius r = = − 19 −4
= 2.8 × 10–2m
Be 1.6  10  8  10
The sense of rotation of the electron in its orbit can be determined from the direction of the
centripetal force. F = – e ( v × B ). So, if we look along the direction of B , the electron
revolves clockwise.

(b) the frequency of revolution of the electron in its circular orbit


eb 1.6  10 −19  8.0  10 −4
f= = Hz = 22.4 MHz
2 m 2   9.1  10 −31
(c) Due to collision with the atomic consistent of the environment, the electron progressively
loses its speed. If the velocity vector of the electron remains in the same plane of the initial
circular orbit after collisions, the radius of the circular orbit will decrease in proportion to
the decreasing speed. However, in general, the velocity of the electron will not remain in
the plane of the initial orbit after collision. In that case, the component of velocity normal
to B will determine the radius of the orbit, while the component of velocity parallel to B
remains constant. Thus, the path of the electron, between two collisions is, in general,
helical. But an important fact must be noted: the frequency of orbital revolution remains
the same, whatever be the speed of the electron.

Example
A beam of protons with velocity 4 × 105 m/s enters a uniform magnetic field of 0.3 tesla at an
angle of 60° to the magnetic field. Find the radius of the helical path taken by the proton beam.
Also find the pitch of helix. Mass of proton=1.67× 10–27kg.
Solution
mv sin 
Radius of helix r = (component of velocity ⊥ to field is vsin)
qB

3
(1.67  10 −27 )(4  10 5 )
= 2 = 2 × 10–2m = 1.2cm
(1.6  10 −19 )0.3 3
2 r
Again, pitch p = vcos × T (where T= )
v sin 
v cos   2 r cos 60   2   (1.2  10 −2 )
 p= = = 4.35× 10–2m = 4.35cm
v sin  sin 60 

PHYSICS WALLAH 39
Magnetic Effect of Current

Example
The region between x = 0 and x = L is filled with uniform, steady magnetic field B0 kˆ . A particle of
mass m, positive charge q and velocity v0 iˆ travels along X-axis and enters the region of magnetic
field. Neglect the gravity throughout the question.
(a) Find the value of L if the particle emerges from the region of magnetic field with its final
velocity at an angle 30° to its initial velocity.
(b) Find the final velocity of the particle and the time spent by it in the magnetic field, if the
magnetic field now extends up to 2.1 L.
Solution
(a) The particle is moving with velocity v0 iˆ , perpendicular to
Magnetic field B0 kˆ . Hence the particle will move along x=0
x=L
mv0
a circular arc OA of radius r = A
qB0 D
30°
Let the particle leave the magnetic field at A. r
30°
AD L r mv0
From CDA, sin60°= =  L = rsin30° = L = C
CA r 2 2 qB0
y
(b) As the magnetic field extends up to 2.1 L i.e., L > 2r, v0i

so the particle completes half cycle before leaving the magnetic


field, as shown in figure.
The magnetic field is always perpendicular to velocity vector, v0i

therefore the magnitude of velocity will remain the same.


Final velocity = v0 ( −iˆ ) =– v0 iˆ
r m
Time spent in magnetic field= =
v0 qB0
Example
A uniform magnetic field with a slit system as shown in fig. is to be used as a momentum filter
for high energy charged particles. With a field of B tesla it is found that the filter transmits
-particle each of energy 5.3 MeV. The magnetic field is increased to 2.3 B tesla and deuterons are
passed into the filter. What is the energy of each deuteron transmitted by the filter ?

Source Detector

Solution
r 2q 2B2
In case of circular motion of a charged particle in a magnetic field Ek =
2m
So according to the given problem
r 2 (2e) 2 B 2 r 2 (e) 2 (2.3B) 2 (E k ) D (2.3) 2  4
(Ek) = and (Ek)D = i.e. =
2(4m) 2(2m) (E k )  22  2
5.29
 (Ek)D = (5.3) × = 14.02 MeV
2
PHYSICS WALLAH 40
Magnetic Effect of Current

Do Yourself – 4 :
(i) In the figure shown find the resultant magnetic force and torque about ‘C’, and ‘P’.

B

P Q
C R
(ii) Prove that magnetic force per unit length on each of the infinitely long wire due to each
other is 012/2d. Here it is attractive also.

1 2

d
(iii) A wire is bent in the form of an equilateral triangle PQR of side 20 cm and carries a
current of 2.5 A. It is placed in a magnetic field B of magnitude 2.0 T directed
perpendicularly to the plane of the loop. Find the forces on the three sides of the
triangle.
(iv) A charged particle is accelerated through a potential difference of 24 kV and acquires a
speed of 2×106 m/s. It is then injected perpendicularly into a magnetic field of strength
0.2 T. Find the radius of the circle described by it.

Example
m
A charged sphere of mass m and charge q starts sliding from rest on a q

vertical fixed circular track of radius R from the position as shown in figure. B
There exists a uniform and constant horizontal magnetic field of induction
B. Find the maximum force exerted by the track on the sphere.
Solution
R O

Fm N R
mgsin 
mg

Magnetic force on sphere


Fm = qvB (directed radially outward)
mv 2
 N – mg sin  – qvB =
R
mv 2
N= + mg sin  + qvB
R
2mgR
Hence, at  = /2 we get Nmax = + mg + qB 2gR = 3mg + qB 2gR
R

PHYSICS WALLAH 41
Magnetic Effect of Current

Example
An electron gun G emits electrons of energy 2keV travelling in the positive x-direction. The
electrons are required to hit the spot S, where GS = 0.1m, and the line GS makes an angle of
60° with the x-axis, as shown in the figure. A uniform magnetic field B parallel to GS exists in
the region outside the electron gun. Find the minimum value of B needed to make the electron
hit S.
B S

60°
x
G
Solution
2EK 1
The velocity of the electrons emitted by electron gun along x-axis, is v =  E = mv2
m 2
The velocity of the electron can be resolved into two components v cos  and v sin , parallel
and perpendicular to the magnetic field respectively. Due to component v cos  electron will
move in the direction of magnetic field with constant speed v cos  but due to component v sin
, it will move on a circular path in the plane ⊥ to magnetic field. Hence electron will move on
a spiral path. As electrons are required to hit the spot S, hence distance travelled by electron in
one time period along the direction of magnetic field must be just equal to GS. The electrons
may also hit the spot S after two or more time periods but minimum value of B is required
As per above discussion,
GS = Distance travelled along the direction of magnetic field in one time period
2 m 2E K 2 m
= (v cos ) × T = v cos  × = × cos  ×
qB m qB

2E 1
B= × cos  × 2m ×
m q  (GS)

2  2  1.6  10 −16 cos 60   2  3.14  9.1  10 −31


=  = 4.68 × 10–3 T
9.1  10 −31 1.6  10 −19  0.1

MOVING COIL GALVANOMETER


A galvanometer is used to detect the current and has moderate resistance.

Principle. When a current carrying coil is placed in a magnetic field, it experiences a torque
given by  = NiAB sin  where  is the angle between normal to plane of coil and direction of
magnetic field. In actual arrangement the coil is suspended between the cylindrical pole pieces
of a strong magnet.

PHYSICS WALLAH 42
Magnetic Effect of Current

The cylindrical pole pieces give the field radial such that sin =1 (always). So torque  = NiAB
If C is torsional rigidity (i.e., restoring couple per unit twist of the suspension wire), then for
deflection  of coil =C. In equilibrium we have external couple = Restoring couple i.e.
NAB
C  = NiAB or  = i i.e.,   i
C
In words the deflection produced is directly proportional to current in the coil.
 NAB
The quantity = is called the current sensitivity of the galvanometer. Obviously for
i C
greater sensitivity of galvanometer the number of turns N, area of coil A and magnetic field B
produced by pole pieces should be larger and torsional rigidity C should be smaller. That is
why the suspension wire is used of phosphor bronze for which torsional rigidity C is smaller.

CONVERSION OF GALVANOMETER INTO AMMETER


An ammeter is a low resistance galvanometer; used to measure current directly in amperes and
is always connected in series with the circuit. To convert a galvanometer into ammeter, a low
resistance, called shunt, is connected in parallel to the galvanometer as shown in figure.

Let ig be the current in galvanometer for its full scale deflection and G the resistance of
galvanometer. Let i is the range of ammeter and is the current in shunt S. Then potential
difference across a and b is
Vab = ig G = iSS. ...(i)
At junction a, i=iS + ig i.e., iS=i–ig
S
Therefore from (i) igG = (i–ig)S or ig(S+G) = iS i.e., ig = i ...(ii)
S +G
This is the working equation for conversion of galvanometer into ammeter. Here ig < i.
ig G i 
From (ii) shunt required S= If ig << i, S=  g  G
i − ig  i 
1 1 1 SG
The resistance of ammeter RA so formed is given by = +  RA = ....(iii)
RA G S S +G
Note: Equation (ii) may also be used to increase the range of given ammeter. Here G will be
resistance of given ammeter, S shunt applied, ig its initial range and i the new range desired.

CONVERSION OF GALVANOMETER INTO VOLTMETER


A voltmeter is a high resistance galvanometer and is connected between two points across
which potential difference, is to be measured i.e., voltmeter is connected in parallel with the
circuit. To convert a galvanometer into voltmeter, a high resistance R in series is connected to
the galva

PHYSICS WALLAH 43
Magnetic Effect of Current

V V
If V is range of voltmeter, then ig = or resistance in series R = − G ...(i)
R+G ig

This is working equation for conversion of galvanometer into voltmeter.


The resistance of voltmeter so formed is RV = R+G ...(ii)
Note: Equation (i) may also be used to increase the range of voltmeter. If V0 is initial range
V0 V
and V is new range of voltmeter, then ig = =
G R+G

Do Yourself – 5 :
(i) A proton (p), -particle and deuteron (D) are moving in circular paths with same
kinetic energies in the same magnetic field. Find the ratio of their radii and time
periods. (Neglect interaction between particles).

(ii) A positive charge particle of charge q, mass m enters into a uniform


P
magnetic field with velocity v as shown in the figure. There is no B
magnetic field to the left of PQ. A
Find v

Q
(i) time spent,
(ii) distance travelled in the magnetic field
(iii) impulse of magnetic force.

(iii) In the figure shown the magnetic field on the left on ‘PQ’ is zero and on the right of
‘PQ’ it is uniform. Find the time spent in the magnetic field.
P

B

(iv) A proton, a dutron and an  particle with the same KE enter in a region of uniform
magnetic field, moving at right angles to B. what is the ratio of the radii of their circular
paths ?
(A) 1 : 2 :1 (B) 1 : 2 : 2 (C) 2 :1 : 1 (D) 2 : 2 :1

PHYSICS WALLAH 44
Magnetic Effect of Current

Answers for Do yourself


Do yourself – 1 :

(i) 0.8 ( −3iˆ + 2ˆj + 3kˆ ) m/s2

(ii) 2 × 10 × 4 × 10–6 = 8 × 10–5 N towards west.

 3  0i  0i  4 0 i
(iii) Bres = 2  + =
 2 a 3 
 2 a 3 a

(iv) 0
(v) (i) Bres = 0
(ii) It is clear from the above solution that B = 0 at point ‘D’.
(vi) (A)

Do yourself – 2 :

  10 −5
(i) T 2 × 10–6 T (ii) 5 × 10–4 T
13 ,
(iii) (ABD) (iv) (ACD)

Do yourself – 3 :

(i) Fres = 0 ,  = iR 2 B( − ˆj) (ii) (B)

(iii) (B) (iv) (C)

Do yourself – 4 :
(i) 2IBR2
(iii) 0
(iv) 12cm

Do yourself – 5 :
(i) 1:2:2
m mv
(ii) (i) t = (ii) r(2) = . 2 (iii) –2mv sin  î
qB qB

(iii) m/qB
(iv) (A)

PHYSICS WALLAH 45
Magnetic Effect of Current

SOME WORKED OUT EXAMPLES


Example#1
Current i = 2.5 A flows along the circle x2 + y2 = 9 cm2 (here x & y in cm) as shown. Magnetic
field at point (0, 0, 4 cm) is

 9   9 
(A) ( 36 10−7 T ) kˆ (B) ( 36   10 −7 T ) ( − kˆ ) (C)   10 −7 T  kˆ (D)   10 −7 T  ( − kˆ )
 5   5 
Solution Ans. (A)
Magnetic field on the axis of a circular loop
−4
  2 iR 2   2.5  3  10  9 
2 2

B =  0  = 10 −7  kˆ =   10 −5 T  kˆ = ( 36  10 −7 T ) kˆ
 4  ( R 2 + z 2 ) 125  10 −6  25 
3/ 2

Example#2
There are constant electric field E 0 ˆj & magnetic field y

Bkˆ present between plates P and P'. A particle of E0 B0


mass m is projected from plate P' along y axis Am v2
P
with velocity v1. After moving on the curved path,
v1
it passes through point A just grazing the plate P
m x
with velocity v2. The magnitude of impulse P'

(i.e. F t = p ) provided by magnetic force during


the motion of particle from origin to point A is :–

(A) m|v2–v1| (B) m v12 + v22 (C) mv1 (D) mv2


Solution Ans. (D)
Electric force is only responsible for the change in momentum along y–axis. Therefore impulse
provide by magnetic force is JB = mv2.

Example#3
Three identical charge particles A, B and C are projected perpendicular to the uniform
magnetic field with velocities v1, v2 and v3 (v1 < v2 < v3) respectively such that T1, T2 and T3 are
their respective time period of revolution and r1, r2 and r3 are respective radii of circular path
described. Then :-
r1 r2 r3 r1 r2 r3
(A)   (B) T1 < T2 < T3 (C)   (D) r1 = r2 = r3
T1 T2 T3 T1 T2 T3
Solution Ans. (C)
2m mv r
T= &r=  v
qB qB T

PHYSICS WALLAH 46
Magnetic Effect of Current

Example#4
An infinitely long straight wire is bent as shown in figure. The circular

cm
10
portion has a radius of 10 cm with its center O at a distance r from the O
straight part. The value of r such that the magnetic field at the center O of r

the circular portion is zero will be :-


10 20 1 5
(A) cm (B) cm (C) cm (D) cm
  5 
Solution Ans. (A)
0 I 0 I 10
Bcircular loop = − Bwire  = r= cm
2  10 2 r 

Example#5 y
Two cylindrical straight and very long non magnetic conductors A and B, A B
x
insulated from each other, carry a current I in the positive and the negative
z–direction respectively. The direction of magnetic field at origin is
(A) −iˆ (B) +iˆ (C) ĵ (D) – ĵ
Solution Ans. (C)
y

A B
BA BB
x

Example#6
The magnetic force between wires as shown in figure is :-

i L

I
 0 iI 2
 x+L  0 iI 2  2x + L 
(A) n  (B) n 
2  2x  2  2x 
 0 iI
 x+L
(C) n  (D) None of these
2  x 
Solution Ans. (C)
0 I i L
Magnetic field at dr, B = dr
2 r
r x
Force on small element at a distance r of wire of length L is
 I 
I
dF = i(dr)  0 
 2 r 
 0i I x+ L dr  0i I
 x+L
F=
2  x r
= n
2  x 

PHYSICS WALLAH 47
Magnetic Effect of Current

Example#7
A wire carrying a current of 4A is bent in the form of a parabola x 2 + y = 16 as shown in
figure, where x and y are in meter. The wire is placed in a uniform magnetic field B = 5 kˆ tesla.
The force acting on the wire is
y

(A) 80 ˆj N (B) – 80 ˆj N (C) −160 ˆj N (D) 160 ˆj N


Solution Ans. (C)
F = I (  B ) = 4 ( 8iˆ  5 kˆ ) = − 160 ˆj N

Example#8
A conducting coil is bent in the form of equilateral triangle of side 5 cm. Current flowing
through it is 0.2 A. The magnetic moment of the triangle is :-
(A) 3 × 10–2 A–m2 (B) 2.2 × 10–4 A–m2 (C) 2.2 × 10–2 A–m2 (D) 3 × 10–4 A–m2
Solution Ans. (B)
Magnetic moment of current carrying triangular loop M = IA
1 3  5  10 −2 
M = 0.2   5  10 −2  –4
 = 2.2 × 10 A-m
2
2 2
 

Example#9
A disc of radius r and carrying positive charge q is rotating with angular speed  in a uniform
magnetic field B about a fixed axis as shown in figure, such that angle made by axis of disc
with magnetic field is . Torque applied by axis on the disc is

Disc 
B

Fixed axis

q  r B sin 
2
q  r 2 B sin 
(A) , clockwise (B) , anticlockwise
2 4
q  r 2 B sin  q  r 2 B sin 
(C) , anticlockwise (D) , clockwise
2 4
Solution Ans. (D)
M q q mr 2 q  r 2 B sin 
= M =   = M B = ( clockwise )
L 2m 2m 2 4

q mr 2 q  r 2 B sin 
M =   = M B = ( clockwise )
2m 2 4

PHYSICS WALLAH 48
Magnetic Effect of Current

Example#10
A particle of mass m and charge q is thrown from origin at t = 0 with velocity 2i + 3 j + 4k
m
units in a region with uniform magnetic field 2i units. After time t = , an electric field E
qB
is switched on, such that particle moves on a straight line with constant speed. E may be
(A) –8 ˆj + 6kˆ units (B) − 6iˆ − 9 kˆ units (C) −12 ˆj + 9 kˆ units (D) 8 ˆj − 6 kˆ units
Solution Ans. (A)
n
At t = , v = 2iˆ − 3 ˆj − 4 kˆ ; For net force to be zero qv  B + qE = 0  E = − v  B = −8 ˆj + 6 kˆ
qB

Example#11
q
A particle of specific charge =   1010 Ckg–1 is projected from the origin along the positive
m
x–axis with a velocity of 105 ms–1 in a uniform magnetic field B = − 2  10 −3 kˆ tesla. Choose
correct alternative(s)
(A) The centre of the circle lies on the y–axis
(B) The time period of revolution is 10–7 s.
5
(C) The radius of the circular path is mm

1
(D) The velocity of the particle at t =  10 −7 s is 10 5 ˆj m / s
4
Solution Ans. (A,B,C,D)
F = q ( v  B ) , v = 10 5 iˆ , B = − 2  10 −3 kˆ  Force will be in y–direction y
Motion of particle will be in xy plane
2 m 2
Time period T = = −3
= 10 −7 s
qB   10  2  10
10

t= T
5 4
mv 10 5 5
Radius of path = = −3
=  10 −3 m = mm
qB   10  2  10
10
 
x
T 10 −7 t=0
At t = = s. Velocity of particle will be in +y direction.
4 4

Example#12
A circular current carrying loop of radius R is bent about its diameter by 90° and placed in a
magnetic field B = B0 (iˆ + ˆj ) as shown in figure. y

(A) The torque acting on the loop is zero


I R 2
(B) The magnetic moment of the loop is
2
( −iˆ − ˆj ) I

x
(C) The angular acceleration of the loop is non zero. I

I R 2 z
(D) The magnetic moment of the loop is
2
( − iˆ + ˆj )
Solution Ans. (A,B)
I R 2
M =
2
( −iˆ − ˆj ) ;  = M  B = 0
PHYSICS WALLAH 49
Magnetic Effect of Current

Example#13
A current–carrying ring is placed in a magnetic field. The direction of the field is perpendicular
to the plane of the ring–
(A) There is no net force on the ring.
(B) The ring will tend to expand.
(C) The ring will tend to contract.
(D) Either (B) or (C) depending on the directions of the current in the ring and the magnetic field.
Solution Ans. (A,D)
Net force = 0 and ring will tend to expand/contracts depending on I & B.

Example#14
In the given figure, B is magnetic field at P due to shown segment AB of an infinite current carrying
wire. A loop is taken as shown in figure. Which of the following statement(s) is/are correct.

 0i
(A)  Bd =  0i (B) B =
2 r
(C) Magnetic field at P will be tangential (D) None of these
Solution Ans. (C)
Magnetic field at P is tangential.

Example#15 to 17
Two charge particles each of mass ‘m’, carrying charge +q and connected with each other by a
massless inextensible string of length 2L are describing circular path in the plane of paper,
qB0 L
each with speed v = (where B0 is constant) about their centre of mass in the region in
m
which an uniform magnetic field B exists into the plane of paper as shown in figure. Neglect
any effect of electrical & gravitational forces.

15. The magnitude of the magnetic field such that no tension is developed in the string will b
B0
(A) (B) B0 (C) 2B0 (D) 0
2

16. If the actual magnitude of magnetic field is half to that of calculated in part (i) then tension in
the string will be
3 q 2 B02 L q 2 B02 L 2q 2 B02 L
(A) (B) zero (C) (D)
4 m 2m m

PHYSICS WALLAH 50
Magnetic Effect of Current

3 q 2 B02 L
17. Given that the string breaks when the tension is T = . Now if the magnetic field is
4 m
reduced to such a value that the string just breaks then find the maximum separation between
the two particles during their motion
(A) 16 L (B) 4L (C) 14L (D) 2L
Solution
15. Ans. (B)
T
mv 2 q + + 2q
T + qvB = qvB
L

 qB0 L  mv 2
T = 0;  q  B =  B = B0
 m  L

16. Ans. (C)


T
+ qvB0 mv 2 m ( qB0 L ) 2 q ( qB0 L ) B0 q 2 B02 L
T+ = ;T= −  =
qv(B0/2) 2 R m2R m 2 2m

17. Ans. (C)

2L v
V Maximum reparation = 2R + (2R – 2L)
R

mv 2
T + qvB =  B = B0/4  R = 4L  maximum separation =16L – 2L= 14L
R

Example#18 to 20
Curves in the graph shown give, as functions of radial distance r, the magnitude B of the magnetic
field inside and outside four long wires a, b, c and d, carrying currents that are uniformly
distributed across the cross sections of the wires. The wires are far from one another.
B
a

b c

d
r

18. Which wire has the greatest radius?


(A) a (B) b (C) c (D) d

19. Which wire has the greatest magnitude of the magnetic field on the surface?
(A) a (B) b (C) c (D) d

PHYSICS WALLAH 51
Magnetic Effect of Current

20. The current density in wire a is


(A) Greater than in wire c
(B) Less than in wire c
(C) Equal to that in wire c
(D) Not comparable to that in wire c due to lack of information
Solution
18. Ans. (C)
0 I R
Inside the cylinder: B = r ...(i) I
2 R 2
r
 I
Outside the cylinder  B = 0 ...(ii) r
2 r
1
Inside cylinder B  r and outside B 
r
So from surface of cylinder nature of magnetic field changes.
Hence it is clear from the graph that wire ‘c’ has greatest radius.

19. Ans. (A)


Magnitude of magnetic field is maximum at the surface of wire ‘a’.

20. Ans. (A)


0 I dB 0 I I
Inside the wire B ( r ) = r; = i.e. slope   current density.
2 R 2
dr 2 R 2
R 2
It can be seen that slope of curve for wire a is greater than wire c.

Example#21
Column-I gives some current distributions and a point P in the space around these current
distributions. Column-II gives some expressions of magnetic field strength. Match column-I to
corresponding field strength at point P given in column-II
Column – I Column – II
3 0 i
(A) A conducting loop shaped as regular hexagon of side x, (P)
32 x
Carrying current i. P is the centroid of hexagon

3 0 i
(B) A cylinder of inner radius x and outer radius 3x, carrying (Q)
x
Current i. Point P is at a distance 2x from the axis of
the cylinder
 0i
(C) Two coaxial hollow cylinders of radii x and 2x, each carrying (R)
2x
current i, but in opposite direction. P is a point at distance
1.5x from the axis of the cylinders
 0i
(D) Magnetic field at the centre of an n-sided regular (S)
3x
polygon, of circum circle of radius x, carrying current
i, n → , P is centroid of the polygon. (T) Zero
PHYSICS WALLAH 52
Magnetic Effect of Current

Solution Ans. (A) → (Q) ; (B) → (P) ; (C) → (S) ; (D) → (R)

P
 0i 3 0 i
For A : BP = 6   sin 30 + sin 30   =
4  ( x sin 60  ) x x
60°
x

3 
0  i  2x
3 0 i P
For B: BP =  8  =
2  ( 2 x ) 32 x x

3x

1.5
x
 0i  0i P
For C : BP = =
2  (1.5 x ) 3 x x

3x

 0i
For D : If n → , n sided polygon → circle so B =
2x

Example#22
A very small current carrying square loop (current I) of side 'L' is placed in y-z plane with
centre at origin of the coordinate system (shown in figure). In column–I the coordinate of the
points are given & in column–II magnitude of strength of magnetic field is given. Then
y
. P2(0, a, 0)

.P3(a, a, 0)

0
I
.
P1(a, 0, 0)
x

z
Column I Column II
 0 I L2
(A) At point O (0, 0, 0) (P)
2 a 3
2 2 0 I
(B) At point P1 (a, 0, 0) (here a > > L) (Q)
L
 0 5 I L2
(C) At point P2 (0, a, 0) (here (a > > L) (R)
16  a 3
 0 I L2
(D) At point P3 (a, a, 0) (here a > > L) (S)
4 a 3

 0 5 IL2
(T)
4 a 3
PHYSICS WALLAH 53
Magnetic Effect of Current

Sol. Ans. (A) → (Q); (B) → (P); (C) → (S); (D) → (R)

0 I     2 2 0 I
For A : BP = 4   sin + sin  =
1
4 ( L / 2 )  4 4 L

 
2  0  ( IL2 )
4   0 IL2
= 
2 kM
For B: BP = =
2
r3 a3 2 a 3

 0  ( 2 )
  IL  0 IL2
4 
=
kM
For C : BP = =
3
r3 a3 4 a 3

kM
For D : BP = 4 3
1 + 3 cos 2 
r

1
where cos  =
2

 0  2
  (IL )
 4   1   5IL
2

B P4 = 1+ 3  = 0
(a 2 ) 3 2 16 a 3

PHYSICS WALLAH 54
Magnetic Effect of Current

EXERCISE #1
Biot Savart's Law
1. Two long, straight wires, each carrying a current of 10 A, are placed along the X- and Y-axes
respectively. The currents point along the positive directions of the axes. Find the magnetic
field at the points
(a) (1 m, 2 m), (b) (–1 m, 2 m), (c) (– 1 m, – 2 m) and (d) (1 m, – 2 m).

2. A circular loop of radius 4.0 cm is placed in a horizontal plane and carries an electric current of
5.0 A in the clockwise direction as seen from above. Find the magnetic field
(a) At a point 3.0 cm above the centre of the loop.
(b) At a point 3.0 cm below the centre of the loop.

3. Two concentric circular coils X and Y of radii 16 cm and 10 cm, respectively, lie in the same
vertical plane containing the north to south direction. Coil X has 20 turns and carries a current
of 16A ; coil Y has 25 turns and carries a current of 18 A. The sense of the current in X is
anticlockwise, and clockwise in Y, for an observer looking at the coils facing west. Give the
magnitude and direction of the net magnetic field due to the coils at their centre. [NCERT]

4. For a circular coil of radius R and N turns carrying current I, the magnitude of the magnetic
 0 IR 2 N
field at a point on its axis at a distance x from its centre is given by, B =
2(x 2 + R 2 ) 3/ 2
(a) Show that this reduces to the familiar result for field at the centre of the coil.
(b) Consider two parallel co-axial circular coils of equal radius R, and number of turns N,
carrying equal currents in the same direction, and separated by a distance R. How that the field
on the axis around the mid-point between the coils is uniform over a distance that is small as
 0 NI
compared to R, and is given by, B = 0.72 , approximately.
R
[Such an arrangement to produce a nearly uniform magnetic field over a small region is known
as Helmholtz coils] [NCERT]

5. A current element  = xiˆ −  yjˆ carries 10A current. It is placed at origin. Calculate magnetic field
at point 'P' which is at position vector r = (iˆ + ˆj)m with respect to origin. (where x = y = 1mm)

6. A circular loop of radius r carries a current i. How should a long, straight wire carrying a current
4i be placed in the plane of the circle so that the magnetic field at the centre becomes zero ?

7. A pair of stationary and infinitely long bent wires are placed in the x-y plane as shown in
figure. The wires carry currents of i = 10A each as shown. The segments L and M are along the
x–axis. The segment P and Q are parallel to the y–axis such that OS = OR = 0.02 m. Find the
magnitude and direction of the magnetic induction at the origin O.
y

i
Q
 L R M  x
i P O S i
i

PHYSICS WALLAH 55
Magnetic Effect of Current

8. A long straight wire carries a current of 10 A directed along the negative y-axis as shown in
figure. A uniform magnetic field B0 of magnitude 10–6 T is directed parallel to the x-axis. What
is the resultant magnetic field at the following points?
z
y

(a) x = 0 , z = 2 m ; (b) x = 2 m, z = 0 ; (c) x = 0, z = – 0.5 m

5
9. Two circular coils A and B of radius cm and 5 cm respectively carry current 5 Amp and
2
5
Amp respectively. The plane of B is perpendicular to plane of A and their centres
2
coincide. Find the magnetic field at the centre.

Ampere's Law
10. Six wires of current I1 = 1A, I2 = 2A, I3 = 3A, I4 = 1A, I5 = 5A and I6 = 4A cut the page
perpendicularly at the points 1, 2, 3, 4, 5 and 6 respectively as shown in the figure. Find the
value of the integral  B  d1 around the closed path.
1 2
4 3 5
6

11. A thin but long, hollow, cylindrical tube of radius r carries a current i along its length. Find the
magnitude of the magnetic field at a distance r/3 from the surface
(a) Inside the tube (b) Outside the tube.

12. A long, cylindrical tube of inner and outer radii a and b carries a current with uniform current
density j over its cross-section. Find the magnitude of the magnetic field at a point
(a) Just inside the tube
(b) Just outside the tube

13. A toroid has a core (non-ferromagnetic) of inner radius 25 cm and outer radius 26 cm, around
which 3500 turns of a wire are wound. If the current in the wire is 11A, what is the magnetic
field
(a) outside the toroid,
(b) inside the core of the toroid, and
(c) in the empty space surrounded by the toroid. [NCERT]

PHYSICS WALLAH 56
Magnetic Effect of Current

14. A magnetic field of 100 G (1G = 10–4 T) is required which is uniform in a region of linear
dimension about 10 cm and area of cross-section about 10-3 m2. The maximum current carrying
capacity of a given coil of wire is 15A and the number of turns per unit length that can be
wound round a core is at most 1000 turns m–1. Suggest some appropriate design particular of a
solenoid for the required purpose. Assume the core is not ferromagnetic. [NCERT]

Motion of Charged Paritcle


15. A charged particle (charge q, mass m) has velocity v0 at origin in +x direction. In space there is a
uniform magnetic field B in -z direction. Find the y coordinate of particle when is crosses y axis.

16. A beam of protons with a velocity 4 × 105 m/s enters a uniform magnetic field of 0.3 T at an
angle of 60° to the magnetic field. Find the radius of the helical path taken by the proton beam.
Also find the pitch of the helix (which is the distance travelled by a proton in the beam parallel
to the magnetic field during one period of rotation).

17. An electron emitted by a heated cathode and accelerated through a potential difference of
2.0 kV, enters a region with uniform magnetic field of 0.15 T. Determine the trajectory of the
electron if the field
(a) is transverse to its initial velocity, (b) makes an angle of 30° with the initial velocity.
[NCERT]

18. In a chamber, a uniform magnetic field of 6.5 G (1 G = 10–4 T) is maintained. An electron is shot
into the field with a speed of 4.8 × 106 m s–l normal to the field. Explain why the path of the
electron is a circle. Determine the radius of the circular orbit. (e = 1.6 × 10–19C, me = 9.1 × l0–31 kg)

19. Answer the following questions :


(a) A magnetic field that varies in magnitude from point to point but has a constant direction
(east to west) is set up in a chamber. A charged particle enters the chamber and travels
undeflected along a straight path with constant speed. What can you say about the initial
velocity of the particle ?

(b) A charged particle enters an environment of a strong and non-uniform magnetic field
varying from point to point both in magnitude and direction and comes out of it following a
complicated trajectory. Would its final speed equal the initial speed if it suffered no
collisions with the environment?

(c) An electron travelling west to east enters a chamber having a uniform electrostatic field in
north to south direction. Specify the direction in which a uniform magnetic field should be
set up to prevent the electron from deflecting from its straight line path.

PHYSICS WALLAH 57
Magnetic Effect of Current

20. A magnetic field set up using Helmholtz coils is uniform in a small region and has a magnitude
of 0.75 T. In the same region, a uniform electrostatic field is maintained in a direction normal
to the common axis of the coils. A narrow beam of (single species) charged particles all
accelerated through 15 kV enters this region in a direction perpendicular to both the axis of the
coils and the electrostatic field. If the beam remains undeflected when the electrostatic field is
9.0 × 105Vm–1, make a simple guess as to what the beam contains. Why is the answer not
unique? [NCERT]

21. In question no. 20, obtain the frequency of revolution of the electron in its circular orbit. Does
the answer depend on the speed of the electron? Explain. [NCERT]

Ampere's Force & Torque


22. A straight horizontal conducting rod of length 0.45 m and mass 60g is suspended by two
vertical wires at its ends. A current of 5.0 A is set up in the rod through the wires. [NCERT]
(a) What magnetic field should be set up normal to the conductor in order that the tension in
the wires is zero?
(b) What will be the total tension in the wires if the direction of current is reversed keeping the
magnetic field same as before? [Ignore the mass of the wires). g = 9.8 ms–2.

23. A uniform magnetic field of 1.5 T exists in a cylindrical region of radius 10.0 cm, its direction
parallel to the axis along east to west. A wire carrying current of 7.0 A in the north to south
direction passes through this region. What is the magnitude and direction of the force on the
wire if, [NCERT]
(a) the wire intersects the axis.
(b) the wire is turned from N-S to northeast-northwest direction,
(c) the wire in the N-S direction is lowered from the axis by a distance of 6.0 cm ?

24. (a) A circular coil of 30 turns and radius 8.0 cm carrying a current of 6.0 A is suspended
vertically in a uniform horizontal magnetic field of magnitude 1.0 T. The field lines make
an angle of 60° with the normal of the coil. Calculate the magnitude of the counter torque
that must be applied to prevent the coil from turning. [NCERT]
(b) Would your answer change, if the circular coil in (a) were replaced by a planar coil of some
irregular shape that encloses the same area?
(All other particulars are also unaltered.)
PHYSICS WALLAH 58
Magnetic Effect of Current

25. A uniform magnetic field of 3000 G is established along the positive z-direction. A rectangular
loop of sides 10 cm and 5 cm carries a current of 12A. What is the torque on the loop in the
different cases shown in figure. What is the force on each case? Which case corresponds to
stable equilibrium? [NCERT]
z z z
B

I I
B B
I
y y y
x (a) x (b) x (c)

z
z z

B B B

y I y I y
30°
x (d) x (e) x (f)

26. A circular coil of 20 turns and radius 10 cm is placed in a uniform magnetic field of 0.10 T
normal to the plane of the coil. If the current in the coil is 5.0 A, what is the [NCERT]
(a) total, torque on the coil,
(b) total force on the coil,
(c) average force on each electron in the coil due to the magnetic field ?
(The coil is made of copper wire of cross-sectional area 10–5 m2, and the free electron density
in copper is given to be about 1029 m–3.)

27. Two moving coil meters. M1 and M2 have the following particulars: [NCERT]
R1 = 10  , N1 = 30,
A1 = 3.6 × 10–3 m2. B1 = 0.25 T
R2 = 14 , N2 = 42 A2 = 1.8 × 10–3 m2, B2 = 0.50 T
(The spring constants are identical for the two meters). Determine the ratio of
(a) current sensitivity and
(b) voltage sensitivity of M2 and M1.

PHYSICS WALLAH 59
Magnetic Effect of Current

EXERCISE #2

1. A long thin walled pipe of radius R carries a current I along its length. The current density is
uniform over the circumference of the pipe. The magnetic field at the center of the pipe due to
quarter portion of the pipe shown, is

2. Find the flux passing through the shaded rectangular plane which passes through the axis due
to the long straight current carrying conductor having a current i. Given length  = 1m, current

0 2 I2
I = 2A, Radius R = 5cm. If answer is n (in weber). Write value of n.
16 


R
i

3. A cylindrical conductor of radius R carries a current along its length. The current density J,
however, it is not uniform over the cross section of the conductor but is a function of the radius
according to J = br, where b is a constant. Find an expression for the magnetic field B.
i

(a) at r1 < R &


(b) at distance r2 > R, mesured from the axis

4. An electron gun G emits electron of energy 2kev traveling in the (+) ve x-direction. The electron
are required to hit the spot S where GS =0.1m & the line GS makes an angle of 60° with the
x-axis, as shown in the fig. A uniform magnetic field B parallel to GS exists in the region
outsides to electron gun. Find the minimum value of B needed to make the electron hit S.
S
B
B
60°
Gun X

PHYSICS WALLAH 60
Magnetic Effect of Current

5. A U-shaped wire of mass m and length l is immersed with its two ends in mercury (see figure).
The wire is in a homogeneous field of magnetic induction B. If a charge, that is, a current pulse
q=  i dt , is sent through the wire, the wire will jump up.

Calculate, from the height h that the wire reaches, the size of the charge or current pulse,
assuming that the time of the current pulse is very small in comparision with the time of flight.
Make use of the fact that impulse of force equals  F dt , which equals mv. Evaluate q for

B = 0.1 Wb/m2, m = 10gm,  = 20cm & h=3 meters. [g = 10 m/s2]

6. A particle of mass 1 × 10–26 kg and charge +1.6 × 10–19 C travelling with a velocity 1.28 × 106 m/s
in the +x direction enters a region in which a uniform electric field E and a uniform magnetic
field of induction B are present such that Ex = Ey = 0, Ez = –102.4 kV/m and Bx = Bz = 0,
By = 8× 10–2 weber/m2. The particle enters this region at the origin at time t = 0. Determine the
location (x, y and z coordinates) of the particle at t = 5 × 10–6 s. If the electric field is switched
off at this instant (with the magnetic field still present), what will be the position of the particle at
t = 7.45 × 10–6 s?

 y ˆ
7. A non-uniform magnetic field B = B0  1 +  (k) is present in region of space in between y = 0
 d
and y = d. The lines are shown in the diagram. A particle of mass ‘m’ and positive charge ‘q’ is
moving. Given an initial velocity v = v ˆi . Find the components of velocity of the particle
0

when it leaves the field.


y

d P(x,y)

v0 x

8. Two coils each of 100 turns are held such that one lies in the vertical plane with their centres
coinciding. The radius of the vertical coil is 20 cm and that of the horizontal coil is 30 cm.
How would you neutralize the magnetic field of the earth at their common centre? What is
the current to be passed through each coil? Horizontal component of earth's magnetic induction
= 3.49×10–5 T and angle of dip = 30º.

9. An infinite wire, placed along z-axis, has current I1 in positive z-direction. A conducting rod
placed in xy plane parallel to y-axis has current I2 in positive y-direction. The ends of the rod
subtend + 30° and –60° at the origin with positive x-direction. The rod is at a distance a from
the origin. Find net force on the rod.

PHYSICS WALLAH 61
Magnetic Effect of Current

10. A circular loop of radius R is bent along a diameter and given a shape as shown in the figure.
One of the semicircles (KNM) lies in the x − z plane and the other one (KLM) in the y − z plane
with their centers at the origin. Current I is flowing through each of the semicircles as shown in
figure.

L M
I

N y
K I
x
z
(i) A particle of charge q is released at the origin with a velocity v = −v0 î . Find the
instantaneous force f on the particle. Assume that space is gravity free.
(ii) If an external uniform magnetic field B ˆj is applied, determine the forces F1 and F2 on the
semicircles KLM and KNM due to this field and the net force F on the loop.

11. A current of 10A flows around a closed path in a circuit which is in the horizontal plane
as shown in the figure. The circuit consists of eight alternating arcs of radii r1 = 0.08 m and
r2 = 0.12 m. Each arc subtends the same angle at the centre.
D
r2 C

A
r1

(a) Find the magnetic field produced by this circuit at the centre.
(b) An infinitely long straight wire carrying a current of 10A is passing through the centre of
the above circuit vertically with the direction of the current being into the plane of the
circuit. What is the force acting on the wire at the centre due to the current in the circuit?
What is the force acting on the arc AC and the straight segment CD due to the current at
the centre?

12. A particle of charge +q and mass m moving under the influence of a uniform electric field E î
and a magnetic field B k̂ enters in I quadrant of a coordinate system at a point (0, a) with initial
velocity v î and leaves the quadrant at a point (2a, 0) with velocity −2v ˆj . Find
(a) Magnitude of electric field
(b) Rate of work done by the electric field at point (0, a)
(c) Rate of work done by both the fields at (2a, 0).

13. Electric charge q is uniformly distributed over a rod of length l. The rod is placed parallel to a
long wire carrying a current i. The separation between the rod and the wire is a. Find the force
needed to move the rod along its length with a uniform velocity v.

14. There exists a uniform magnetic and electric field of magnitude 1 T and 1 V/m respectively
along positive y-axis. A charged particle of mass 1 kg and charge 1 C moving with velocity
1 m/sec along x-axis is at origin at t = 0. If the coordinates of particle at time p seconds is
5
given as (X, Y, Z) in meter, then find the value of (XY + YZ + ZX) .
2

PHYSICS WALLAH 62
Magnetic Effect of Current

15. A proton beam passes without deviation through a region of space where there are uniform
transverse mutually perpendicular electric and magnetic field with E and B. Then the beam
strikes a grounded target. Find the force imparted by the beam on the target if the beam current
is equal to I.

16. The figure shows a conductor of weight 1.0 N and length L = 0.5 m placed on a rough inclined
plane making an angle 300 with the horizontal so that conductor is perpendicular to a uniform
horizontal magnetic field of induction B = 0.10 T. The coefficient of static friction between the
conductor and the plane is 0.1. A current of I = 10 A flows through the conductor inside the
plane of this paper as shown. What is the force needed to be the applied parallel to the inclined
plane to sustaining the conductor at rest?

17. An infinite uniform current carrying wire is kept along z-axis, carrying Y F
G E
current I0 in the direction of the positive z-axis. OABCDEFG represents
a circle (where all the points are equally spaced), whose centre at point O (4m, 0 m) D X
0 I0

C
(4m, 0 m) and radius 4 m as shown in the figure. If B.d = in A
B
DEF
k
S.I. unit, then find the value of k.

18. An infinite current carrying conductor, parallel to z-axis is situated at point P as shown in the
B → 0i
figure. Value of  B . d is given by  , then fill the value of  in OMR sheet ?
A
96
Y
P
(0, a)

O A (a, 0)
(0, 0) (a/ 3, 0) B X

19. A rod of length  and total charge ‘q’ which is uniformly distributed is rotating with angular velocity
 about an axis passing through the centre of rod and perpendicular to rod. Find the magnitude of
magnetic dipole moment (in Amp. m2) of rod. (Take : q = 4C, = 6 rad/s and = 2m.)


20. A uniform wooden bar of mass kg and radius of cross-
100 C G
P
section 10cm carries a light coil C of 100 turns. The bar is
smoothly pivoted at P. If the coil carries a current 2A and B
x
–2
subjected to an external magnetic field 10 T, the bar remains
in equilibrium. Find the distance x (in cm) of the C.M. of the rod from the pivot.

PHYSICS WALLAH 63
Magnetic Effect of Current

21. Q charge is uniformly distributed over the same surface of a right circular cone of semi-vertical
angle  and height h. The cone is uniformly rotated about its axis at angular velocity . Calculated
associated magnetic dipole moment.

22. A stationary, circular wall clock has a face with a radius of 15cm. Six turns of wire are wound
around its perimeter, the wire carries a current 2.0 A in the clockwise direction. The clock is
located, where there is a constant, uniform external magnetic field of 70 mT (but the clock still
keeps perfect time) at exactly 1:00 pm, the hour hand of the clock points in the direction of the
external magnetic field (a) After how many minutes will the minute hand point in the direction
of the torque on the winding due to the magnetic field? (b) What is the magnitude of this
torque.
23. A square current carrying loop made of thin wire and having a mass m =10g can rotate without
friction with respect to the vertical axis OO1, passing through the centre of the loop at right
angles to two opposite sides of the loop. The loop is placed in a homogeneous magnetic field
with an induction B = 10-1 T directed at right angles to the plane of the drawing. A current
I = 2A is flowing in the loop. Find the period of small oscillations that the loop performs about
its position of stable equilibrium.

24. 3 infinitely long thin wires each carrying current i in the same direction, are in the x-y plane of
a gravity free space. The central wire is along the y-axis while the other two are along x = ± d.
(i) Find the locus of the points for which the magnetic field B is zero.
(ii) If the central wire is displaced along the z-direction by a small amount & released, show
that it will execute simple harmonic motion. If the linear density of the wires is , find the
frequency of oscillation.

25. A rectangular loop PQRS made from a uniform wire has length a, width b and mass m. It is free
to rotate about the arm PQ, which remains hinged along a horizontal line taken as the
y-axis (see figure). Take the vertically upward direction as the z-axis. A uniform magnetic field
B = (3 ˆi + 4 k)
ˆ B exists in the region. The loop is held in the x-y plane and a current I is passed
0

through it. The loop is now released and is found to stay in the horizontal position in equilibrium.

(a) What is the direction of the current I in PQ?


(b) Find the magnetic force on the arm RS.
(c) Find the expression for I in terms of B0, a, b and m.
PHYSICS WALLAH 64
Magnetic Effect of Current

26. A rectangular loop of wire is oriented with the left corner at the origin, one edge along X-axis
and the other edge along Y-axis as shown in the figure. A magnetic field is into the page and
has a magnitude that is given by  = y where  is constant. Find the total magnetic force on
the loop if it carries current i.

27. A closed loop carrying a current i is placed so that its plane is perpendicular to the long straight
conductor which carries a current i0 as shown in the figure. The torque acting on the current
loop is µ0 N-m. Then find the value of . (Given i = 2 A, i0 = 2A, r0 = 2 m,  = 60°)
i
r0
i0
 i i

2r0

28. A straight segment OC (of length L meter) of a circuit carrying a current I amp is placed along
the x-axis. Two infinitely ling straight wires A and B, each extending form z = –  to + , are
fixed at y = – a metre and y = +a metre respectively, as shown in the figure. If the wires A and
B each carry a current I amp into plane of the paper. Obtain the expression for the force acting
on the segment OC. What will be the force OC if current in the wire B is reversed?

29. A non conducting non-magnetic rod having square cross-section of side L/4 is suspended from
a rigid support as shown in the figure. A light and small coil of 600 turns is wrapped tightly at
the left end of the rod where uniform magnetic field B exists in vertically downward direction.
Air of density  hits the half of the right part of the rod with velocity v as shown in the figure.
What should be the current (in mA) in the coil so that rod remains horizontal? [Assume air
particles come to rest after colliding with the rod] [Take :  = 1.05 kg/m3, v = 10 m/s, B = 1T,
L = 2m]
air v

L/4
O
B L/4
L L/2 L/2

PHYSICS WALLAH 65
Magnetic Effect of Current

EXERCISE #3

Biot Savart's Law


1. A horizontal overhead power line carries a current of 90 A in east to west direction. What is
the magnitude and direction of the magnetic field due to the current 1.5 m below the line?
(A) 1.2 × 10–5T, towards south (B) 1.2 × 10–5T, towards north
(C) 2.5 × 10–5T, towards south (D) 2.5 × 10–5T, towards north

2. A long, straight wire carrying a current of 1.0 A is placed horizontally in a uniform magnetic
field B = 1.0 × 10–5 T pointing vertically upward (figure). The magnitude of the resultant
magnetic field at the points P and Q, both situated at a distance of 2.0 cm from the wire in the
same horizontal plane are respectively
P
i
Q B
(A) zero, 20 µT (B) 20 T, zero (C) zero, zero (D) 20 T, 20 T

3. Two parallel wires carry equal currents of 10 A along the same direction and are separated by a
distance of 2.0 cm. Find the magnetic field at a point which is 2.0 cm away from each of these
wires.
(A) 3.4 × 10–4 T in a direction parallel to the plane of the wires and perpendicular to the wires
(B) 1.7 × 10–4 T in a direction parallel to the plane of the wires and parallel to the wires
(C) 1.7 × 10–4 T in a direction parallel to the plane of the wires and perpendicular to the wires
(D) 3.4 × 10–4 T in a direction parallel to the plane of the wires and parallel to the wires

4. Consider a 10 cm long piece of a wire which carries a current of 10 A. Find the magnitude of
the magnetic field due to the piece at a point which makes an equilateral triangle with the ends
of the piece.
(A) Zero (B) 5.5 µT (C) 7.5 µT (D) 11.5 µT

5. A conducting circular loop of radius a is connected to two long, straightwires. The straight
wires carry a current i as shown in figure. Find the magnetic field B at the centre of the loop.

i i

0I 0I 0I 0I


(A) zero (B) (C) (D) +
2a a a 2 a

6. A piece of wire carrying a current of 6.00 A is bent in the form of a circular arc of radius
10.0 cm, and it subtends an angle of 120° at the centre. Find the magnetic field B due to this
piece of wire at the centre.
(A) zero (B) 1.26 × 10–5 T (C) 5 × 10–5 T (D) 7.2 × 10–5 T
PHYSICS WALLAH 66
Magnetic Effect of Current

7. A charge of 3.14 × 10–6 C is distributed uniformly over a circular ring of radius 20.0 cm. The
ring rotates about its axis with an angular velocity of 60.0 rad/s. Find the ratio of the electric
field to the magnetic field at a point on the axis at a distance of 5.00 cm from the centre.
(A) zero (B) 0.5 × 1015 m/s (C) 1.88 × 1015 m/s (D) 5.88 × 1015 m/s

8. All straight wires are very long. Both AB and CD are area of the same circle, both subtending
right angles at the centre O. Then the magnetic field at O is
A A´
i
B O R
C i C´
D

B´ D´

 0i  0i  0i  0i
(A) (B) 2 (C) (D) (  + 1)
4 R 4 R 2 R 2 R

9. An equilateral triangular loop of wire of side carries a current i. The magnetic field produced
at the circumcenter of the loop is
 0 3 3i  0 9i  0 18i  0 6i
(A) (B) (C) (D)
   

10. Consider six wires coming into or out of the page, all with the same current. Rank the line
integral of the magnetic field (from most positive to most negative) taken counterclockwise
around each loop shown.
loop C
loop B

X
X
X

loop D
loop A
(A) B > C > D > A (B) B > C = D > A (C) B > A > C = D (D) C > B = D > A

11. Statement-1 : Ampere law can be used to find magnetic field due to finite length of a straight
current carrying wire.
Statement-2 : The magnetic field due to finite length of a straight current carrying wire is symmetric
about the wire.
(A) Statement-1 is true, statement-2 is true and statement-2 is correct explanation for
statement-1.
(B) Statement-1 is true, statement-2 is true and statement-2 is NOT the correct explanation for
statement-1.
(C) Statement-1 is true, statement-2 is false.
(D) Statement-1 is false, statement-2 is true.
PHYSICS WALLAH 67
Magnetic Effect of Current

12. A long, cylindrical wire of radius b carries a current i distributed uniformly over its cross-
section. Find the magnitude of the magnetic field at a point inside the wire at a distance a from
the axis.
 0 ib  0 ia 2  0 ia
(A) zero (B) (C) (D)
2 a 2
2 b 3
2 b 2

13. A copper wire of diameter 1.6 mm carries a current of 20 A. Find the maximum magnitude of
the magnetic field B due to this current.
(A) 5.0 mT (B) 10 mT (C) 15 mT (D) 15.5 mT

14. A closely wound solenoid 80 cm long has 5 layers of windings of 400 turns each. The diameter
of the solenoid is 1.8 cm. If the current carried is 8.0 A. estimate the magnitude of B inside the
solenoid near its centre.
(A) zero (B) 8 × 10–3 T (C) 15 × 10–3T (D)  × 10–3 T

15. A circular loop of radius 20 cm carries a current of 10 A. An electron crosses the plane of the
loop with a speed of 2.0 × 106 m/s. The direction of motion makes an angle of 30° with the
axis of the circle and passes through its centre. Find the magnitude of the magnetic force on the
electron at the instant it crosses the plane.
(A) zero (B)  × 10–19 N (C) 16 × 10–19 N (D) 32 × 10–19 N

Motion of Charged Particle


16. A charged particle enters a non-uniform uni-directional field such that initial velocity is
parallel to magnetic field, then the radius of curvature of its path is (in standard notation) :
(A) mV/qB (B) 0 (C)  (D) qB/mV

17. A charge particle moves in a uniform magnetic field such that initial velocity is perpendicular
to the magnetic field. No other force acts on the particle.
(A) the motion is uniform rectilinear
(B) the motion can be non uniform circular motion
(C) the motion can be uniform circular motion
(D) the motion must be uniform circular motion.

18. A tightly-wound, long solenoid carries a current of 2.00 A. An electron is found to execute a
uniform circular motion inside the solenoid with a frequency of 1.00 × 108 rev/s. Find the
number of turns per metre in the solenoid.
(A) 500 Turns/m (B) 1020 Turns/m (C) 1232 Turns/m (D) 1420 Turns/m

19. An electron is moving along positive x-axis. A uniform electric field exists towards negative
y-axis. What should be the direction of magnetic field of suitable magnitude so that net force
of electron is zero
(A) positive z- axis (B) negative z-axis (C) positive y-axis (D) negative y-axis

PHYSICS WALLAH 68
Magnetic Effect of Current

20. Two protons move parallel to each other, keeping distance r between them, both moving with
same velocity V . Then the ratio of the electric and magnetic force of interaction between them is
(A) c 2 V 2 (B) 2c 2 V 2 (C) c 2 2V 2 (D) None

21. An electron having kinetic energy T is moving in a circular orbit of radius R perpendicular to a
uniform magnetic induction B . If kinetic energy is doubled and magnetic induction tripled, the
radius will become
3R 3 2 4
(A) (B) R (C) R (D) R
2 2 9 3

22. Two particles A and B of masses mA and mB respectively and having the same charge are
moving in a plane. A uniform magnetic field exists perpendicular to this plane. The speeds of
the particles are vA and vB respectively and the trajectories are as shown in the figure. Then

(A) mAvA < mBvB (B) mAvA > mBvB


(C) mA < mB and vA < vB (D) mA = mB and vA = vB

23. A charged particle moves in a magnetic field B = 10 ˆi with initial velocity u = 5iˆ + 4ˆj . The
path of the particle will be
(A) straight line (B) circle (C) helical (D) none

24. An electron makes 3 × 105 revolutions per second in a circle of radius 0.5 angstrom. Find the
magnetic field B at the centre of the circle.
(A) 6 × 10–10 T (B) 12 × 10–10 T (C) 18 × 10–10 T (D) 24 × 10–10 T

25. Electrons moving with different speeds enter a uniform magnetic field in a direction perpendicular
to the field. They will move along circular paths.
(A) of same radius
(B) with larger radii for the faster electrons
(C) with smaller radii for the faster electrons
(D) either (B) or (C) depending on the magnitude of the magnetic field

26. In the previous question, time periods of rotation will be :


(A) same for all electrons
(B) greater for the faster electrons
(C) smaller for the faster electrons
(D) either (B) or (C) depending on the magnitude of the magnetic field
PHYSICS WALLAH 69
Magnetic Effect of Current

27. A particle of mass m and charge q moves with a constant velocity v along the positive
x-direction. It enters a region containing a uniform magnetic field B directed along the
negative z-direction, extending from x = a to x = b. The minimum value of v required so that
the particle can just enter the region x > b is
(A) q b B/m (B) q( b – a) B/m (C) q a B/m (D) q(b + a) B/2m

28. A particle having charge of 1 C, mass 1 kg and speed 1 m/s enters a uniform magnetic field,
having magnetic induction of 1 T, at an angle  = 30° between velocity vector and magnetic
induction. The pitch of its helical path is (in meters)
3 
(A) (B) 3 (C) (D) 
2 2
29. A particle with charge +Q and mass m enters a magnetic field of magnitude B, existing only to
the right of the boundary YZ. The direction of the motion of the particle is perpendicular to the
m
direction of B. Let T = 2 . The time spent by the particle in the field will be
QB

  + 2    − 2 
(A) T (B) 2T (C) T   (D) T  
 2   2 

30. In the previous question, if the particle has –Q charge, the time spend by the particle in the
field will be
  + 2    − 2 
(A) T (B) 2T (C) T   (D) T  
 2   2 

31. A block of mass m & charge q is released on a long smooth inclined plane magnetic field B is
constant, uniform, horizontal and parallel to surface as shown. Find the time from start when
block loses contact with the surface.

m cos  m cos ec  m cot 


(A) (B) (C) (D) none
qB qB qB

32. In a cyclotron, a charged particle


(A) Undergoes acceleration all the time.
(B) Speeds up between the dees because of the magnetic field.
(C) Speeds up in a dee.
(D) Slows down within a dee and speeds up between dees.
PHYSICS WALLAH 70
Magnetic Effect of Current

Ampere Force & Torque


33. In given figure, X and Y are two long straight parallel conductors each carrying a current of
2 A. The force on each conductor is F Newton. When the current in each is changed to 1 A and
reversed in direction, the force on each is now

(A) F/4 and unchanged in direction (B) F/2 and reversed in direction
(C) F/2 and unchanged in direction (D) F/4 and reversed in direction
34. Two long and parallel straight wires A and B carrying currents of 8.0 A and 5.0 A in the same
direction are separated by a distance of 4.0 cm. Estimate the force on a 10 cm section of wire
A.
(A) 2 × 10–5 N; attractive force normal to B towards A.
(B) 2 × 10–5 N; attractive force normal to A towards B.
(C) 5 × 10–5 N; attractive force normal to A towards B.
(D) 5 × 10–5 N; attractive force normal to B towards A.

35. A 3.0 cm wire carrying a current of 10 A is placed inside a solenoid perpendicular to its axis.
The magnetic field inside the solenoid is given to be 0.27 T. What is the magnitude of magnetic
force on the wire?
(A) zero (B) 3.5 × 10–2 N (C) 8.1 × 10–2 N (D) 12.1 × 10–2 N

36. What is the magnitude of magnetic force per unit length on a wire carrying a current of 8 A
and making an angle of 30° with the direction of a uniform magnetic field of 0.15 T?
(A) zero (B) 1 N m–1 (C) 3.2 N m–1 (D) 0.6 N m–1

37. The wires which connected the battery of an automobile to its starting motor carry a current of
300 A (for a short time). What is the force per unit length between the wires if they are 70 cm
long and 1.5cm apart? Is the force attractive or reupulsive ?
(A) 1.2 N m–1 ; repulsive (B) 1.2 N m–1 ; attractive
(C) 3.5 N m–1 ; repulsive (D) 3.5 N m–1; attractive

38. A solenoid 60 cm long and of radius 4.0 cm has 3 layers of windings of 300 turns each. A
2.0 cm long wire of mass 2.5 g lies inside the solenoid (near its centre) normal to its axis ; both
the wire and the axis of the solenoid are in the horizontal plane. The wire is connected through
two leads parallel to the axis of the solenoid to an external battery which supplies a current of
6.0 A in the wire. What value of current (with appropriate sense of circulation) in the windings
of the solenoid can support the weight of the wire? g = 9.8 ms–2.
(A) zero (B) 108 A (C) 200 A (D) 458 A

39. The magnetic field lines due to a bar magnet are correctly shown in

(A) (B) (C) (D)

PHYSICS WALLAH 71
Magnetic Effect of Current

40. A wire of mass 100 g carrying a current of 2A towards increasing x is in the form of
y = x2 (–2m  x  +2m). This wire is placed in a magnetic field B = − 0.02kˆ Tesla & gravity
free region. The acceleration of the wire (in m/s2) is :-
(A) −1.6 ˆj (B) −3.2 ˆj (C) 1.6 ˆj (D) 2.4 ˆj
41. A circular loop of radius R carries a current I. Another circular loop of radius r(<<R) carries a
current i and is placed at the centre of the larger loop. The planes of the two circles are at right
angle to each other. Find the torque acting on the smaller loop.
 0 iIr 2  0 iIr 2  0 iIr 2
(A) zero (B) (C) (D)
4R 2R R

42. A particle of charge q and mass m moves in a circular orbit of radius r with angular speed .
The ratio of the magnitude of its magnetic moment to that of its angular momentum depends
on
(A)  and q (B) , q and m (C) q and m (D)  and m

43. A rectangular coil PQ has 2n turns, an area 2a and carries a current 2I, (refer figure). The plane
of the coil is at 60° to a horizontal uniform magnetic field of flux density B. The torque on the
coil due to magnetic force is

(A) BnaI sin60° (B) 8BnaI cos60° (C) 4naI Bsin60° (D) none

44. The square loop ABCD, carrying a current I, is placed in a uniform magnetic field B,
as shown. The loop can rotate about the axis XX'. The plane of the loop makes an angle
 ( < 90°) with the direction of B. Through what angle will the loop rotate by itself before the
torque on it becomes zero
x
C
 Z
B
Y B
I D

A X'
(A)  (B) 90° –  (C) 90° +  (D) 180° – 

45. Figure shows a square current carrying loop ABCD of side 10 cm and current i = 10A. The
magnetic moment M of the loop is

( )
(A) (0.05) ˆi − 3kˆ A − m 2 ( )
(B) (0.05) ˆj + kˆ A − m 2

(C) (0.05) ( 3iˆ + kˆ ) A − m 2


(D) ( ˆi + kˆ ) A − m 2

PHYSICS WALLAH 72
Magnetic Effect of Current

46. A disc of radius r and carrying positive charge q is rotating with an 

angular speed  in a uniform magnetic field B about a fixed axis as shown Disc 
B
in figure, such that angle made by axis of disc with magnetic field is .
Torque applied by axis on the disc is Fixed axis
q  r B sin 
2
q  r B sin 
2

(A) , clockwise (B) , anticlockwise


2 4
q  r 2 B sin  q  r 2 B sin 
(C) , anticlockwise (D) , clockwise
2 4

47. A square coil of side 10 cm consists of 20 turns and carries a current of 12 A. The coil is
suspended vertically and the normal to the plane of the coil makes an angle of 30° with the
direction of a uniform horizontal magnetic field of magnitude 0.80 T. What is the magnitude of
torque experienced by the coil ?
(A) zero (B) 2.32 Nm (C) 1.56 Nm (D) 0.96 N m

MULTIPLICATE CORRECT TYPE

48. Consider three quantities x = E/B, y = 1 /  0  0 and z = . Here, l is the length of a wire, C
CR
is a capacitance and R is a resistance. All other symbols have standard meanings.
(A) x, y have the same dimensions
(B) y, z have the same dimensions
(C) z, x have the same dimensions
(D) none of the three pairs have the same dimensions.

49. The magnetic lines of force due to a straight current carrying wire will be:
(A) circular for finite length of wire (B) circular for semi-infinite wire
(C) circular for infinite wire (D) Parabolic for finite wire

50. A current I flows along the length of an infinitely long, straight, solid pipe. Then correct
statement(s)-
(A) The magnetic field is zero only on the axis of the pipe
(B) The magnetic field is different at different points inside the pipe
(C) The magnetic field is maximum on surface
(D) The magnetic field at all points inside the pipe is the same, but not zero

51. Figure shows cross-section view of a infinite cylindrical wire with a y

cylinderical cavity, current density is uniform ˆj = − j kˆ as shown in figure. 


 
0

(A) Field inside cavity is uniform   x
 →
a
(B) Field inside cavity is along a    

 
(C) Field inside cavity is perpendicular to a
(D) If an electron is projected with velocity v0 ˆj it will move undeviated before colliding with
cavity wall

PHYSICS WALLAH 73
Magnetic Effect of Current

52. A particle of charge q is projected with a momentum P = Piˆ in the given region of magnetic
field B = Bkˆ . It emerges from the magnetic field after deviating through an angle  = 30°.

x

v y

d x

(A) The value of P is 2qBd


(B) The value of P is qBd
P
(C) Maximum change in momentum takes place for d 
qB
P
(D) Maximum change in momentum takes place for d 
2qB

53. Two charged particles traverse identical helical paths in a completely opposite sense in a
uniform magnetic field B = B kˆ0

(A) They have equal z-components of momenta.


(B) They must have equal charges.
(C) They necessarily represent a particle-antiparticle pair.
 e   e 
(D) The charge to mass ratio satisfy   +  = 0:
 m 1  m  2

54. Current flows through uniform, square frames as shown. In which case is the magnetic field at
the centre of the frame not zero?

(A) (B) (C) (D)

55. A long straight wire carries a current along the x-axis. Consider the points A(0, 1, 0), B(0, 1,
1), C(1, 0, 1) and D(1, 1, 1). Which of the following pairs of points will have magnetic fields
of the same magnitude?
(A) A and B (B) A and C (C) B and C (D) B and D

56. Two identical charged particles enter a uniform magnetic field with same speed but at angles
30° and 60° with field. Let a, b and c be the ratio of their time periods, radii and pitches of the
helical paths than
(A) abc = 1 (B) abc > 1 (C) abc < 1 (D) a = bc

57. An electron is moving along the positive X-axis. You want to apply a magnetic field for a short
time so that the electron may reverse its direction and move parallel to the negative X-axis.
This can be done by applying the magnetic field along
(A) Y-axis (B) Z-axis (C) Y-axis only (D) Z-axis only
PHYSICS WALLAH 74
Magnetic Effect of Current

58. In a region of space, a uniform magnetic field B exists in the y-direction. A proton is fired from
the origin, with its initial velocity v making a small angle  with the y-direction in the yz
plane. In the subsequent motion of the proton,

(A) its x-coordinate can never be positive


(B) its x- and z-coordinates cannot both be zero at the same time
(C) its z-coordinate can never be negative
(D) its y-coordinate will be proportional to the square of its time of flight

MATRIX MATCH TYPE


59. Three wires are carrying same constant current i in different directions. Four loops enclosing
the wires in different manners are shown. The direction of d is shown in figure.
Loop-1

i i
Loop-2
i
Loop-3

Loop-4

Column I Column II
(A) Along closed loop–1 (P)  B.d = µ 0i

(B) Along closed loop–2 (Q)  B.d = −µ 0 i

(C) Along closed loop–3 (R)  B.d =0


(D) Along closed loop–4 (S) Net work done by the magnetic force to
move a unit charge along the loop is zero
60. Column I (Magnetic moment of) Column II
q r 2
(A) a uniformly charged ring rotating uniformly about its axis (P)
5
q r 2
(B) a charged particle rotating uniformly about a point (Q)
4
q r 2
(C) a uniformly charged disk rotating uniformly about its axis (R)
3
q r 2
(D) a uniformly charged spherical shell rotating (S)
2
uniformly about one of its diameter

(E) a uniformly charged sphere rotating (T) qr2


uniformly about one of its diameter

PHYSICS WALLAH 75
Magnetic Effect of Current

EXERCISE #4
1. A ring like metallic conductor of resistance R and radius a, caries a constant current I. The
ratio of the angular momentum L of the conduction electrons (about the axis of the ring) and
the magnetic field B at the centre of the ring satisfy [where e and m represent the magnitudes
of the electronic charge and mass]
B e2 B B e B m
(A)  (B)  e.m (C)  (D) 
L m L L m L e

2. Two long parallel wires are at a distance 2d apart. They carry steady equal currents flowing out
of the plane of the paper, as shown. The variation of the magnetic field B along the XX’ is
given by

(A) (B) (C) (D)

3. Which of the following field patterns is correct for two long straight equal parallel current
carrying wires ?
i i
2a

(A) (B) (C) (D)

4. An infinitely long conductor PQR is bent to form a right angle as shown. A current I flows
through PQR. The magnetic field due to this current at the point M is H1. Now, another
infinitely long straight conductor QS is connected at Q so that the current in PQ remaining
unchanged. The magnetic field at M is now H2. The ratio H1/H2 is given by

(A) 1/2 (B) 1 (C) 2/3 (D) 2

5. The magnetic field at the centre of a current carrying ring is B0 and at a small axial distance
B − B0
x (<< R) ; R = radius of the ring, is B. Then, =
B0
2 2 3
x 3 x  5 x 
(A)   (B)   (C)   (D) None of these
R 2R 2R

PHYSICS WALLAH 76
Magnetic Effect of Current

6. Equal antiparallel currents are directed in two parallel wires so that one is out of the page and
the other is into the page as shown. Compare the magnitude of the magnetic field B2 at any
arbitrary point equidistant from the wires to the magnitude of the field B1 at that point from
one wire alone :

(A) B2 > B1 for all equidistant points


(B) B2 < B1 for all equidistant points
(C) B2 > B1 for closer equidistant points only
(D) B2 < B1 for closer equidistant points only

7. A long straight wire along the z-axis carries a current I in the negative z-direction. The magnetic
vector field B at a point having coordinates (x, y) in the z = 0 plane is
 0 I (yiˆ − xj)
ˆ  0 I (xiˆ + yj)
ˆ  0 I (xjˆ − yi)
ˆ  0 I (xiˆ − yj)
ˆ
(A) (B) (C) (D)
2  (x 2 + y 2 ) 2  (x 2 + y 2 ) 2  (x 2 + y 2 ) 2  (x 2 + y 2 )

8. Three rings, each having equal radius R, are placed mutually perpendicular to each other and
each having its centre at the origin of co-ordinate system. If current I is flowing thriugh each
ring then the magnitude of the magnetic field at the common centre is
0I
(A) 3
2R
(B) zero (C) ( 2 −1 ) 2RI
0
(D) ( 3− 2 ) 2RI
0

9. Two concentric coils X and Y of radii 16 cm and 10 cm lie in the same vertical plane
containing N-S direction. X has 20 turns and carries 16 A. Y has 25 turns & carries 18A. X has
current in anticlockwise direction and Y has current in clockwise direction for an observer,
looking at the coils facing the west. The magnitude of net magnetic field at their common
centre is
(A) 5 × 10–4 T towards west (B) 13 × 10–4 T towards east
(C) 13 × 10–4 T towards west (D) 5 × 10–4 T towards east

10. Infinite number of straight wires each carrying current I are equally placed as shown in the
figure. Adjacent wires have current in opposite direction. Net magnetic field at point P is

 0 I ln 2  0 I ln 4  0 I ln 4 ˆ
(A) k̂ (B) k̂ (C) ( − k) (D) Zero
4 3a 4 3a 4 3a

PHYSICS WALLAH 77
Magnetic Effect of Current

11. Two mutually perpendicular conductors carrying currents I1 and I2 lie in one plane. Locus of
the point at which the magnetic induction is zero, is a
(A) circle with centre as the point of intersection of the conductor.
(B) parabola with vertex as the point of intersection of the conductors
(C) straight line passing through the point of intersection of the conductors.
(D) rectangular hyperbola

12. Find the magnetic field at P due to the arrangement shown

 0i  1  2 0 i
(A) 1 −  (B) 
2 d  2 2 d
 0i  0i  1 
(C)  (D) 1 + 
2 d 2 d  2

13. A coil having N turns is wound tightly in the form of a spiral with inner and outer radii a and b
respectively. When a current I passes through the coil, the magnetic field at the centre is
 0 NI 2 0 NI  0 NI b 0IN b
(A) (B) (C) ln (D) ln
b a 2(b − a) a 2(b − a) a

14. In the diagram shown, a wire carries current I. What is the value of the  B·ds (as in Ampere's
law) on the helical loop shown in the figure? The integration in done in the sense shown. The loop
has N turns and part of helical loop on which arrows are drawn is outside the plane of paper.
I

(A) – µ0(NI) (B) µ0(I) (C) µ0(NI) (D) Zero

15. Two small spherical conductors with initial charge +Q and –Q are connected with a very long
straight wire with a switch. When switch is closed
P
r Key
+ –
+Q –Q
C
Statement-1: Ampere's law in the form  B.d = 0 Ienclosed is applicable for the curve C
encircling the wire.
Statement-2: From symmetry considerations magnetic field is tangent at each point of circle C.
(A) Statement-1 is true, statement-2 is true and statement-2 is correct explanation for statement-1.
(B) Statement-1 is true, statement-2 is true and statement-2 is NOT the correct explanation for
statement-1.
(C) Statement-1 is true, statement-2 is false.
(D) Statement-1 is false, statement-2 is true.
PHYSICS WALLAH 78
Magnetic Effect of Current

16. A coaxial cable is made up of two conductors. The inner conductor is solid and is of radius R1
& the outer conductor is hollow of inner radius R2 and outer radius R3. The space between the
conductors is filled with air. The inner and outer conductors are carrying currents of equal
magnitudes and in opposite directions. Then the variation of magnetic field with distance from
the axis is best plotted as:

R1

R2

R3

B B B B
(A) (B) (C) (D)
r r r r
R 1 R 2 R3 R 1 R 2 R3 R 1 R 2 R3 R 1 R 2 R3

17. A small segment of length a is cut along z-axis from a infinite sheet having a surface current
density J (current per unit width). The magnetic field at P is :
P y
h x

a
J

 h ˆ  0 Jh 0 J  a  0 j  h 
(A) 2 0 J  1 − i (B) î (C)  − 1  ˆi (D) −  − 1  ˆi
 a  2a 2  h  2  a 

18. A hollow cylinder having infinite length and carrying uniform current per unit length  along
the circumference as shown. Magnetic field inside the cylinder is

0
(A) (B) 0 (C) 20 (D) none
2

19. Two long conductors are arranged as shown above to form overlapping
cylinders, each of radius r, whose centers are separated by a distance
d. Current of density J flows into the plane of the page along the
shaded part of one conductor and an equal current flows out of the
plane of the page along the shaded portion of the other, as shown.
What are the magnitude and direction of the magnetic field at point A?
(A) (0/2)dJ, in the +y-direction (B) (0/2)d2/r, in the +y-direction
(C) (0/2)4d2J/r, in the –y-direction (D) (0/2)Jr2/d, in the –y-direction
PHYSICS WALLAH 79
Magnetic Effect of Current

20. An ionized gas contains both positive and negative ions. If it is subjected simultaneously to an
electric field along the +x direction and a magnetic field along the +z direction, then
(A) positive ions deflect towards +y direction and negative ions towards -y direction
(B) all ions deflect towards +y direction.
(C) all ions deflect towards -y direction
(D) positive ions deflect towards -y direction and negative ions towards +y direction.

21. At t = 0 a charge q is at the origin and moving in the y-direction


with velocity v = v ˆj . The charge moves in a magnetic field
that is for y > 0 out of page and given by B1 zˆ and for y < 0
into the page and given − B2 zˆ . The charge's subsequent trajectory
is shown in the sketch. From this information, we can deduce
that
(A) q > 0 and | B1 | < | B2 | (B) q < 0 and | B1 | < | B2 |
(C) q > 0 and | B1 | > | B2 | (D) q < 0 and | B1 | > | B2 |

22. A particle of charge q and mass m starts moving from the origin under the action of an electric
field E = E ˆi and B = B ˆi with velocity v = v ˆj . The speed of the particle will become 2v0
0 0 0

after a time
2mv 0 2Bq 3 Bq 3 mv 0
(A) t = (B) t = (C) t = (D) t =
qE mv 0 mv 0 qE

23. Four particles A, B, C and D of masses mA, mB, mC and mD respectively, follow the paths
shown in the figure, in a uniform magnetic field. Each particle moving with same speed. QA,
QB, QC and QD are the specific charge of particles A,B,C and D respectively
Constant
magnetic field

R
A B
2R C
R
D
(A) QA < QB < QC < QD
(B) QD < QB < QC < QA
(C) Charge on the particle B and particle D is of different nature
(D) Work done by magnetic force on the particle C is minimum as compared to other particle

24. ( )
A electron experiences a force 4.0 ˆi + 3.0 ˆj × 10–13 N in a uniform magnetic field when its

( )
velocity is 1.5 ˆi − 2.0 ˆj  10 7 ms–1. When the velocity is redirected and becomes 2.5 kˆ 10 7 ms–1,

the magnetic force of the electron is zero. The magnetic field vector B is :
(A) – 0.075 ˆi + 0.1 ˆj (B) 0.1iˆ + 0.075 ˆj (C) 0.075 ˆi − 0.1 ˆj + kˆ (D) 0.075 ˆi − 0.1 ˆj

PHYSICS WALLAH 80
Magnetic Effect of Current

25. A non-planar circular loop consists of two semi-circles one of which lies in yz-plane & the
other is in xz-plane as shown. The magnetic force experienced by positive charge of value Q
moving with velocity v along x direction when it is at the origin is :
y

I
x

Qv 0 I Qv 0 I Qv 0 I


(A) (B) (C) (D) 0
4R 2R 2 2R

26. A charged particle is released from rest in a region of uniform electric and magnetic fields,
which are parallel to each other. The locus of the particle will be a
(A) helix of constant pitch (B) straight line
(C) helix of varying pitch (D) cycloid

27. A particle of specific charge (charge/mass)  starts moving from the origin under the action of
an electric field E = E ˆi and magnetic field B = B kˆ . Its velocity at (x0, y0, 0) is (4iˆ − 3ˆj) . The
0 0

value of x0 is:
13  E 0 16  B 0 25 5
(A) (B) (C) (D)
2 B0 E0 2 E 0 2B 0

28. A particle of specific charge (q/m) is projected from the origin of coordinates with initial
velocity [ui – vj]. Uniform electric magnetic fields exist in the region along the +y direction, of
magnitude E and B. The particle will definitely return to the origin once if
(A) [vB/2E] is an integer (B) (u2 + v2)1/2 [B/E] is an integer
(C) [vB/E] in an integer (D) [uB/E] is an integer

29. An electron moving with a velocity V1 = 2 ˆi m/s at a point in a magnetic field experiences a

force F1 = − 2 ˆjN . If the electron is moving with a velocity V2 = 2 ˆj m/s at the same point, it

experiences a force F2 = + 2 ˆi N . The force the electron would experience if it were moving

with a velocity V3 = 2kˆ m/s at the same point is

(A) zero ˆ
(B) 2kN
ˆ
(C) − 2kN (D) information is insufficient

30. A metal ring of radius r = 0.5 m with its plane normal to a uniform magnetic field B of
induction 0.2 T carries a current I = 100 A. The tension in Newton developed in the ring is:
(A) 100 (B) 50 (C) 25 (D) 10

PHYSICS WALLAH 81
Magnetic Effect of Current

31. In a certain region uniform electric field E and magnetic field x


B are present in the opposite direction. At the instant t = 0, a
particle of mass m carrying a charge q is given velocity vo at an
angle , with the y axis, in the yz plane. The time after which B
z
E 
the speed of the particle would be minimum is equal to
mv 0 mv 0 sin  y v0
(A) (B)
qE qE
mv 0 cos  2 m
(C) (D)
qE qB

32. Two long wires which are perpendicular to each other carry currents as shown in the figure.
They are free to move. Consider only magnetic interaction

(A) The two wires will come closer translationly.


(B) The two wires will move away translationly.
(C) The two wires will rotate such that the currents become uni-directional and then come
closer due to attraction
(D) The two wires will rotate such that the currents become anti-parallel and then move far
away due to repulsion.

33. A uniform magnetic field B = 3iˆ + 4 ˆj + kˆ( ) Tesla exist in a region of space. A semicircular
wire of radius 1 m carrying a current of 1A having its centre at (2,2,0) m is placed on the X-Y
plane as shown. The force on the semicircular wire will be
y
1m
(2m, 2m)

45°
x
O

(A)
1
2
( ˆi − ˆj + kˆ ) N (B) (
2 ˆi − ˆj + kˆ N ) (C)
1
2
( ˆi + ˆj − kˆ ) N (D) (
2 ˆi + ˆj − kˆ N )
34. The magnetic force between wires as shown in figure is :

i L

I
 0 iI 2
x+   0 iI 2
 2x +   0 iI  x + 
(A) n  (B) n  (C) n  (D) None of these
2  2x  2  2x  2  x 
PHYSICS WALLAH 82
Magnetic Effect of Current

35. A very long wire carrying current I is fixed along x-axis. Another parallel finite wire carrying a
current in the opposite direction is kept at a distance d above the wire in xy plane. The second
wire is free to move parallel to itself. The options available for its small displacements are in
(i) +ve x direction (ii) +ve y direction (iii) +ve z direction
Taking gravity in negative y direction, the nature of equilibrium of second wire is
(A) stable for movement in x direction, unstable for movement in y direction, neutral for movement
in z direction
(B) stable for movement in y direction, unstable for movement in z direction, neutral for movement
in x direction
(C) stable for movement in z direction, unstable for movement in y direction, neutral for movement
in x direction
(D) stable for movement in y direction, unstable for movement in x direction, neutral for movement
in z direction

36. A conductor of length  is placed perpendicular to a horizontal uniform magnetic field B.


Suddenly a certain amount of charge is passed through it, when it is found to jump to a height
h. The amount of charge than passes through the conductor is
m gh 2m gh m 2gh
(A) (B) (C) (D) None of these
B 3B B

37. A semi circular current carrying wire having radius R is placed in x-y plane with its centre at
Bo x ˆ
origin ‘O’. There is non-uniform magnetic field B = k (here Bo is +ve constant) is existing in
2R
the region. The magnetic force acting on semi circular wire will be along

(A) – x-axis (B) + y-axis (C) – y-axis (D) + x-axis


38. A current carrying circular coil of single turn of mass kg & radius 25cm is hanging by two
10
ideal strings. A constant magnetic field 1T is set up in the horizontal direction and 1A current
flows through coil as shown in figure. Then the ratio of tension (T1/T2) in the string will be :
z

T2
T1
B t

O 45°

–y

(A) 2 : 1 (B) 5 : 3 (C) 4 : 1 (D) 1 : 2


PHYSICS WALLAH 83
Magnetic Effect of Current

39. A uniform current carrying ring of mass m and radius R is connected by a massless string as
shown. A uniform magnetic field B0 exist in the region to keep the ring in horizontal position,
then the current in the ring is ( = length of string)

B0

mg mg mg mg
(A) (B) (C) (D)
RB 0 RB 0 3RB 0 R 2 B 0

40. A conducting ring of mass 2 kg and radius 0.5 m is placed on a smooth horizontal plane. The
ring carries a current i = 4A. A horizontal magnetic field B = 10T is switched on at time t = 0
as shown in figure. The initial angular acceleration of the ring will be

(A) 40  rad/s2 (B) 20  rad/s2 (C) 5  rad/s2 (D) 15  rad/s2

41. In the figure shown a coil of single turn is wound on a sphere of radius R and mass m. The
plane of the coil is parallel to the plane and lies in the equatorial plane of the sphere. Current in
the coil is i. The value of B if the sphere is in equilibrium is

mg cos  mg mg tan  mg sin 


(A) (B) (C) (D)
iR iR iR iR

MULTIPLE CORRECT TYPE

42. ˆ
A charged particle is projected with initial velocity in x-y plane in a magnetic field B = 10kT
from the origin. The particle moves in a circle and just touches a straight line y = 5 (m) at
x = 5 3 (m). Then (mass of particle = 5 × 10–5 kg, charge = 1 C)

(A) The particle is projected at an angle 60° with x–axis


(B) The radius of the circle is 10 m
(C) The speed of the particle is 2m/s
(D) The speed of the particle is 5 3 m/s

PHYSICS WALLAH 84
Magnetic Effect of Current

43. A particle of charge +q and mass m moving under the influence of a uniform electric field E iˆ
and uniform magnetic field Bkˆ follows a trajectory from P to Q as shown in figure. The
velocities at P and Q are viˆ and −2vjˆ , Which of the following statement(s) is/are correct ?

y
P v E
B
a

Q
x
2a 2v

3  mv 2 
(A) E =  
4  qa 

3  mv 3 
(B) Rate of work done by the electric field at P is  
4 a 
(C) Rate of work done by the electric field at P is zero
(D) Rate of work done by both the fields at Q is zero

44. Two long thin, parallel conductors carrying equal currents in the same direction are fixed parallel
to the x-axis, one passing through y = a and the other through y = –a. The resultant magnetic
field due to the two conductors at any point is B. Which of the following are correct?
Z

–a
a Y
O
i
i
X

(A) B = 0 for all points on the x-axis


(B) At all points on the y-axis, excluding the origin, B has only a z-component.
(C) At all points on the z-axis, excluding the origin, B has only a y-component.
(D) B cannot have an x-component.

45. A particle of charge 'q' and mass 'm' enters normally (at point P) in a region of magnetic field
with speed v. It comes out normally from Q after time T as shown in figure. The magnetic
field B is present only in the region of radius R and is uniform. Initial and final velocities are
along radial direction and they are perpendicular to each other. For this to happen, which of
the following expression(s) is/are correct–
v
Q

B
v
P
R

mv R m
(A) B= (B) T= (C) T= (D) None of these
qR 2V 2qB

PHYSICS WALLAH 85
Magnetic Effect of Current

46. A particle of mass m and charge q moving with velocity v enters a region of uniform magnetic
field of induction B . Then
(A) its path in the region of the field is always circular
(B) its path in the region of the field is circular if v .B =0
(C) its path in the region of the field is a straight line if v  B = 0
(D) distance travelled by the particle in time T does not depend on the angle between and

COMPREHENSION TYPE
Paragraph for Question Nos. 47 to 49
Curves in the graph shown give, as functions of radial distance r, the magnitude B of the
magnetic field inside and outside four long wires a, b, c and d, carrying currents that are
uniformly distributed across the cross sections of the wires. Overlapping portions of the plots
are indicated by double labels.
B

a
b
c a,
a,b b,d c
c,d
r
47. Which wire has the greatest radius?
(A) a (B) b (C) c (D) d

48. Which wire has the greatest magnitude of the magnetic field on the surface?
(A) a (B) b (C) c (D) d

49. The current density in wire a is


(A) greater than in wire c
(B) less than in wire c
(C) equal to that in wire c
(D) not comparable to that in wire c due to lack of information

Paragraph for Question Nos. 50 to 53


A velocity filter uses the properties of electric and magnetic
Detector
fields to select charged particles that are moving with a specific E
velocity. Charged particles with varying speeds are directed into
B
the filter as shown in figure. The filter consists of an electric field (into d
page)
E and a magnetic field B, each of constant magnitude, directed v
perpendicular to each other as shown. The particles that move
q
straight through the filter with their direction unaltered by the
fields have the specific filter speed, v0. Those with speeds to v0 may experience sufficiently
little deflection that they also enter the detector.
The charged particle will experience a force due to the electric field given by the
relationship F = qE , where q is the charge of the particle and E is the electric field. The
moving particle will also experience a force due to the magnetic field. This force acts to
PHYSICS WALLAH 86
Magnetic Effect of Current

oppose the force due to the electric field. The strength of the force due to the magnetic field is
given by the relationship F = q ( v  B ) , where q is the charge of the particle v , is the speed of
the particle, and B is the magnetic field strength. When the forces due to the two fields are
equal and opposite, the net force on the particle will be zero, and the particle will pass through
the filter with its path unaltered. The electric and magnetic field strengths can be adjusted to
choose the specific velocity to be filtered. The effects of gravity can be neglected.
50. The electric and magnetic fields in the filter of figure are adjusted to detect particles with positive
charge q of a certain speed, v0. Which of the following expressions is equal to this speed ?
(A) B/(q2E) (B) E/(q2B) (C) B/E (D) E/B

51. Which of the following is true about the velocity filter shown in figure ?
(A) It would not work with negatively charged particles
(B) The wider the detector entrance, the more narrow the range of speed detected
(C) The greater the distance d, the more narrow the range of speeds detected
(D) The detector may not detect a charged particle with the desired filter speed if its charge is
too high

52. Which of the following statements is true regarding a charged particle that is moving through
the filter at a speed that is less than the filter speed ?
(A) It experiences a greater force due to the magnetic field than due to the electric field
(B) It experiences a greater force due to the electric field than due to the magnetic field
(C) It experiences equal force due to both fields but greater acceleration due to the electric field
(D) It experiences equal force due to both fields but greater acceleration due to the magnetic field

53. Particles of identical mass and charge are sent through the filter at varying speeds, and the
magnitude of acceleration of each particle is recorded as it first begins to be deflected. If the
filter is set to detect particles of speed v0, which one of the following is correct graph between
acceleration and velocity of particle:

(A) a (B) a (C) a (D) a


v0 v0 v0 v0
v v v v

Paragraph for Question Nos. 54 & 55


Each atom in a paramagnetic material has a permanent magnetic moment associated with the
net angular momentum of electrons. In a diamagnetic material with no applied magnetic field
the individual atoms have no net magnetic moment; the magnetic effects of the electrons in
each individual atom cancel. An external magnetic field changes the motion of the electron and
induces a net magnetic moment in each atom.

54. Consider an electron (charge e) of mass m orbiting a nucleus in orbit of radius r with speed v.
An external field is applied parallel to orbital magnetic moment of electron. Taking change in
speed to be small as compared to orbital speed and orbital radius to remain constant, find
change in magnitude of magnetic moment :-
e2 r 2B e2 r 2B 4e 2 r 2 B 2e 2 r 2 B
(A) (B) (C) (D)
4m 2m m m

PHYSICS WALLAH 87
Magnetic Effect of Current

55. Mark the CORRECT statement :


(A) If magnetic field is applied parallel to orbital magnetic moment (M) of electron then M is
decreased.
(B) If magnetic field is applied parallel to orbital magnetic moment (M) of electron then M is
increased.
(C) Change in magnetic moment dM vector is always perpendicular to B
(D) Change in magnetic moment dM vector is always parallel to B

Paragraph for Question Nos. 56 to 58


In a rigid wire frame ABCDEFA, as shown in the figure, a current I0 flows through the wire
from F to A as shown in the figure. Radii of the three quarters AB, CD and EF are r 1=31/4 m,
r2=3–1/4m and r3=1 m respectively with common centre at O. All the wires are light except for
the portion ED which have a mass of ‘m’ kg.
Z
B

Y
D O F A
E

56. The magnetic moment of the loop is


I 0  ˆ 1 ˆ ˆ  I 0  ˆ 1 ˆ ˆ 
(A) 3i+ j+ k (B) − 3i+ j+ k
4  3  4  3 
I 0  1 ˆ ˆ I 0  1 ˆ ˆ
(C)  − 3 ˆi − j+ k (D)  − 3 ˆi + j+ k
4  3  4  3 

57. If this loop is kept in a magnetic field of magnitude 2 T which is directed along positive y-axis,
angular acceleration of the loop is [ where  = (r3–r2)]

8 I 0 ˆ 6 I 0 ˆ 8 I 0 ˆ 6 I 0 ˆ
(A)  i + 3kˆ  (B)  i + 3kˆ  (C)  i − 3kˆ  (D)  i − 3kˆ 
m 2 m 2 m 2 m 2

58. If circular wire AB, CD and EF are replaced by straight wires AB, CD and EF, as shown by the
dotted lines and keeping the same sense of current, the magnetic field due to the straight wires
at the center O is

 0 I  ˆi   0 I  ˆi 
(A) −  1/ 4 + 3 ˆj + kˆ  (B) −  1/ 4 + 3 ˆj + kˆ 
1/ 4 1/ 4

2  3  4  3 

 0 I  ˆi   0 I  ˆi 
(C) −  1/ 4 + 3 ˆj + kˆ  (D) −  1/ 4 + 3 ˆj + kˆ 
1/ 4 1/ 4

2  3  2 2  3 

PHYSICS WALLAH 88
Magnetic Effect of Current

Paragraph for Question Nos. 59 to 61


Inside a homogenous long straight cylindrical wire of
 Axis of
circular cross-sectional area with radius R. There is a R/2 cavity
J
circular cylinder cavity of radius R/2 whose axis is parallel Proton C
to the conductor axis and displaced relative to it by a
y
distance R/2. A current of density j flows along the wire j
Smooth
whose top view and side view are shown in the figure. O C x
A point positive charge 'q' and a negative charged particle
(–q) of equal mass m are projected with velocity v ˆi and 0
(−)
Axis of conductors
8mv
u 0 ˆj respectively from point C. Given that  R 2 and all collisions are perfectly elastic.
 0 qj

Read the above passage carefully and answer the following question.
59. The trajectory of motion of positive particle lies in plane.
(A) x-y plane (B) y-z plane (C) x-z plane (D) none

60. The radius of curvature of motion of positive charge particle is given by :


2mv 0 4mv 0 8mv 0
(A) (B) (C) (D) None of these
q  0 JR q  0 JR q  0 JR

61. The time period motion of negative charged particle is given by :


4 8 m 16 m
(A) (B) (C) (D) None of these
q  0 JR q  0 JR q  0 JR

MATRIX MATCH TYPE


62. A beam consisting of four types of ions A, B, C and D enters a region that contains a uniform
magnetic field as shown. The field is perpendicular to the plane of the paper, but its precise
direction is not given.
1 3
ION MASS CHARGE
A 2m e
B 4m –e v
C 2m –e
D m +e 2 4
All ions in the beam travel with the same speed. The table below gives the masses and charges
of the ions. The ions fall at different positions 1, 2, 3 and 4, as shown. Correctly match the
ions with respective falling positions.
Table–I Table–II
(A) A (P) 1
(B) B (Q) 2
(C) C (R) 3
(D) D (S) 4

PHYSICS WALLAH 89
Magnetic Effect of Current

63. A charged particle with some initial velocity is projected in a region where uniform electric
and/or magnetic fields are present. In Column-I information about the existence of electric
and/or magnetic field and direction of initial velocity of charged particle are given, while in
column II the possible paths of charged particle is mentioned. Match the entries of Column I
with the entries of Column II.
Column–I Column–II

(A) E = 0, B  0 and initial velocity is (P) Straight line

at an unknown angle with B

(B) E  0, B = 0 and initial velocity is (Q) Parabola

at an unknown angle with E

(C) E  0, B  0, E || B and initial velocity (R) Circular

is perpendicular to E

(D) E  B, B  0, E perpendicular B and (S) Helical path with nonuniform pitch

initial velocity is perpendicular to (T) Helical path with uniform pitch

both E and B

PHYSICS WALLAH 90
Magnetic Effect of Current

EXERCISE #5 (JEE MAINS)

1. If in a circular coil A of radius R, current i is flowing and in another coil B of radius 2R a


current 2i is flowing, then the ratio of the magnetic fields, BA and BB produced at the centre by
them will be : [AIEEE 4/300 2002]
(A) 1 (B) 2 (C) 1/2 (D) 4

2. If an electron and a proton having same momenta enter perpendicularly to a magnetic field,
then : [AIEEE 4/300 2002]
(A) curved path of electron and proton will be same (ignoring the sense of revolution)
(B) they will move undeflected
(C) curved path of electron is more curved than that of proton
(D) path of proton is more curved

3. If a current is passed through a spring then the spring will : [AIEEE 4/300 2002]
(A) expand (B) compress (C) remain same (D) none of these

4. Wires 1 and 2 carrying currents i1 and i2 respectively are inclined at an angle  to each other.
What is the force on a small element dl of wire 2 at distance r from wire 1 (as shown in figure)
due to the magnetic field of wire 1? [AIEEE 4/300 2002]
0 0
(A) i1 i2 dl tan  (B) i1 i2 dl sin 
2 r 2 r
0 0
(C) i1 i2 dl (cos  + 1) (D) i1 i2 dl sin 
4 r 4 r

5. At a specific instant emission of radioactive compound is deflected in a magnetic field. The


compound can emit : [AIEEE 4/300 2002]
(i) electrons (ii) protons (iii) He2+ (iv) neutrons
The emission at the instant can be
(A) i, ii, iii (B) i, ii, iii, iv (C) iv (D) ii, iii

6. A magnetic needle lying parallel to a magnetic field requires W units of work to turn it through
60º. The torque needed to maintain the needle in this position will be :
(A) 3W (B) W (C) ( 3 / 2) W (D) 2 W

7. Magnetic lines of force inside a bar magnet : [AIEEE 4/300 2003]


(A) are from north-pole to south-pole of the magnet
(B) do not exist
(C) depend upon the area of cross-section of the bar magnet
(D) are from south-pole to north-pole of the magnet

PHYSICS WALLAH 91
Magnetic Effect of Current

8. A current i ampere flows along an infinitely long straight thin walled tube, then the magnetic
induction at any point inside the tube is :
 0 2i 2i
(A) infinite (B) zero (C) , tesla (D) tesla
4 r r

9. A long wire carries a steady current. It is bent into a circle of one turn and the magnetic field at
the centre of the coil is B. It is then bent into a circular loop of n turns. The magnetic field at
the centre of the coil will be: [AIEEE 4/300 2004]
(A) nB (B) n2B (C) 2nB (D) 2n2B

10. A current i ampere flows along an infinitely long straight thin walled tube, then the magnetic
induction at any point inside the tube is : [AIEEE 4/300 2004]
 0 2i 2i
(A) infinite (B) zero (C) , tesla (D) tesla
4 r r

11. A long wire carries a steady current. It is bent into a circle of one turn and the magnetic field at
the centre of the coil is B. It is then bent into a circular loop of n turns. The magnetic field at
the centre of the coil will be: [AIEEE 4/300 2004]
(A) nB (B) n2B (C) 2nB (D) 2n2B

12. The magnetic field due to a current carrying circular loop of radius 3 cm at a point on the axis
at a distance of 4 cm from the centre is 54 T. What will be its value at the centre of the loop ?
[AIEEE 4/300 2004]
(A) 250 T (B) 150 T (C) 125 T (D) 75 T

13. Two long conductors, separated by a distance d carry currents I1 and I2 in the same direction.
They exert a force F on each other. Now the current in one of them is increased to two times
and its direction is reversed. The distance is also increased to 3d. The new value of the force
between them is : [AIEEE 4/300 2004]
(A) –2 F (B) F/3 (C) –2F/3 (D) – F/3

14. The length of a magnet is large compared to its width and breadth. The time period of its
oscillation in a vibration magnetometer is 2s. The magnet is cut perpendicular to its length into
three equal parts and three parts are then placed on each other with their like poles together.
The time period of this combination will be : [AIEEE 4/300 2004]
(A) 2s (B) 2/3s (C) 2 3 s (D) 2 / 3 s

15. The materials suitable for making electromagnets should have : [AIEEE 4/300 2004]
(A) high retentivity and high coercivity
(B) low retentivity and low coercivity
(C) high retentivity and low coercivity
(D) low retentivity and high coercivity

PHYSICS WALLAH 92
Magnetic Effect of Current

16. Two thin, long, parallel wires, separated by a distance ‘d’ carry a current of ‘i’ A in the same
direction. They will : [AIEEE 4/300 2005]
 0i 2
(A) attract each other with a force of
(2  d)
 0i 2
(B) repel, each other with a force of
(2  d)
 0i 2
(C) attract each other with a force of
(2 d 2 )
 0i 2
(D) repel each other with a force of
(2 d 2 )

17. Two concentric coils each of radius equal to 2 cm are placed at right angles to each other. 3
ampere and 4 ampere are the currents flowing in each coil respectively. [AIEEE 4/300 2005]
The magnetic induction in weber/m2 at the centre of the coils will be (0 = 4 × 10–7 Wb/A.m):
(A) 12 × 10–5 (B) 10–5 (C) 5 × 10–5 (D) 7 × 10–5

18. A uniform electric field and a uniform magnetic field are acting along the same direction in a
certain region. If an electron is projected along the direction of the fields with a certain
velocity, then : [AIEEE 4/300 2005]
(A) its velocity will decrease
(B) its velocity will increase
(C) it will turn towards right of direction of motion
(D) it will turn towards left of direction of motion.

19. A charged particle of mass m and charge q travels on a circular path of radius r that is
perpendicular to a magnetic field B. The time taken by the particle to complete one revolution
is : [AIEEE 4/300 2005]
2 mq 2 q 2 B 2 qB 2 m
(A) (B) (C) (D)
B B m qB

20. A magnetic needle is kept in a non-uniform magnetic field. It experiences :


[AIEEE 4/300 2005]
(A) a torque but not a force (B) neither a force nor a torque
(C) a force and a torque (D) a force but not a torque

21. In a region, steady and uniform electric and magnetic fields are present. These two fields are
parallel to each other. A charged particle is released from rest in this region. The path of the
particle will be a : [AIEEE 1.5/180 2006]
(A) circle (B) helix (C) straight line (D) ellipse

22. Needles N1, N2 and N3 are made of a ferromagnetic, a paramagnetic and a diamagnetic substance
respectively. A magnet when brought close to them will : [AIEEE 1.5/180 2006]
(A) attract all three of them
(B) attract N1 and N2 strongly but repel N3
(C) attract N1 strongly, N2 weakly and repel N3 weakly
(D) attract N1 strongly, but repel N2 and N3 weakly
PHYSICS WALLAH 93
Magnetic Effect of Current

23. A long solenoid has 200 turns per cm and carries a current i. The magnetic field at its centre is
6.28×10–2 Weber/m2. Another long solenoid has 100 turns per cm and it carries a current i/3.
The value of the magnetic field at its centre is : [AIEEE 4.5/180 2006]
–4 –2 2
(A) 1.05 × 10 Weber/m2 (B) 1.05 × 10 Weber/m
–5
(C) 1.05 × 10 Weber/m 2
(D) 1.05 × 1010–3 Weber/m2

24. A long straight wire of radius a carries a steady current i. The current is uniformly distributed
a
across its cross-section. The ratio of the magnetic field at and 2a from axis is :
2
[AIEEE 3/120 2007]
(A) 1/4 (B) 4 (C) 1 (D) 1/2

25. A current I flows along the length of an infinitely long, straight, thin walled pipe. Then :
[AIEEE 3/120 2007]
(A) the magnetic field is zero only on the axis of the pipe
(B) the magnetic field is different at different points inside the pipe
(C) the magnetic field at any point inside the pipe is zero
(D) the magnetic field at all points inside the pipe is the same, but not zero

26. A charged particle with charge q enters a region of constant, uniform and mutually orthogonal
fields E and B with a velocity v perpendicular to both E and B , and comes out without any
change in magnitude or direction of v . Then : [AIEEE 3/120 2007]
(A) v = E  B / B 2 (B) v = E  E / B 2 (C) v = E  E / E 2 (D) v = B  E / E 2

27. A charged particle moves through a magnetic field perpendicular to its direction. Then :
[AIEEE 3/120 2007]
(A) the momentum changes but the kinetic energy is constant
(B) both momentum and kinetic energy of the particle are not constant
(C) both, momentum and kinetic energy of the particle are constant
(D) kinetic energy changes but the momentum is constant

28. Two identical conducting wires AOB and COD are placed at right angles to each other. The
wire AOB carries an electric current  and COD carries a current . The magnetic field on a
point lying at a distance d from O, in a direction perpendicular to the plane of the wires AOB
and COD, will be given by : [AIEEE 3/120 2007]

1/ 2
 0  1 +  2  0 0 0
( +  22 ) ( +  22 )
1/ 2
(A)   (B) 2
(C) (1 + 2) (D) 2

2  d  2 d 2 d 2 d
1 1

PHYSICS WALLAH 94
Magnetic Effect of Current

29. Relative permittivity and permeability of a material are r and r, respectively. Which of the
following values of these quantities are allowed for a diamagnetic material ? [AIEEE 3/105 2008]
(A) r = 1.5, r = 0.5 (B) r = 0.5, r = 0.5
(C) r = 1.5, r = 1.5 (D) r = 0.5, r = 1.5

30. A horizontal overhead powerline is at a height of 4 m from the ground and carries a current of
100 A from east to west. The magnetic field directly below it on the ground is
(0 = 4 × 10–7 T mA–1): [AIEEE 3/ 105 2008]
(A) 5 × 10–6 T northward (B) 5 × 10–6 T southward
(C) 2.5 × 10–7 T northward (D) 2.5 × 10–7 T southward

Comprehension : [AIEEE 2009]


Direction : Question numbers 29 and 30 are based on the following paragraph :
A current loop ABCD is held fixed on the plane of the paper as shown in the figure. The arcs
BC (radius = b) and DA (radius = a) of the loop are joined by two straight wires AB and. CD.
A steady current  is flowing in the loop. Angle made by AB and CD at the origin O is 30º.
Another straight thin wire with steady current  flowing out of the plane of the paper is kept at
the origin.

31. The magnitude of the magnetic field due to the loop ABCD at the origin (O) is:
[AIEEE 4/144 2009]
 0  (b − a) 0  b − a 
(A) (B)
24ab 4   ab 

0   
(C)  2(b − a) + (a + b)  (D) zero
4  3 

32. Due to the presence of the current 1 at the origin : [AIEEE 4/144 2009]
(A) The forces on AD and BC are zero.
 0 1    
(B) The magnitude of the net force on the loop is given by  2(b − a) + (a + b) 
4  3 
 0 1
(C) the magnitude of the net force on the loop is given by (b – a).
24ab
(D) the forces on AB and DC are zero.

PHYSICS WALLAH 95
Magnetic Effect of Current

33. Two long parallel wires are at a distance 2d apart. They carry steady equal currents flowing out
of the plane of the paper as shown. The variation of magnetic field B along the line XX´ is
given by [AIEEE 4/144 2010]

(A) (B)

(C) (D)

34. A current  flows in an infinitely long wire with cross–section in the form of a semicircular
ring of radius R. The magnitude of the magnetic induction on its axis is :
[AIEEE - 2011, 1-May, 4/120, –1]
0 0 0 0
(A) (B) (C) (D)
 R
2
2 R 2
2 R 4 R

35. An electric charge +q moves with velocity v = 3iˆ + 4ˆj + kˆ , in an electromagnetic field given by :
E = 3iˆ + ˆj + 2kˆ and B = ˆi + ˆj + 3kˆ . The y - component of the force experienced by + q is :
[AIEEE 2011, 11 May; 4, –1]
(A) 7 q (B) 5 q (C) 3 q (D) 2 q

36. A thin circular disk of radius R is uniformly charged with density  > 0 per unit area. The disk
rotates about its axis with a uniform angular speed . The magnetic moment of the disk is :
[AIEEE 2011, 11 May; 4, –1]
R 4 R 4
(A) R4  (B)  (C)  (D) 2R4 
2 4

37. A charge Q is uniformly distributed over the surface of non-condcting disc of radius R. The
disc rotates about an axis perpendicular to its plane and passing through its centre with an
angular velocity . As a result of this rotation a magnetic field of induction B is obtained at the
centre of the disc. if we keep both the amount of charge placed on the disc and its angular
velocity to be constant and vary the radius of the disc then the variation of the magnetic
induction at the centre of the disc will be represented by the figure : [AIEEE 2012 ; 4/120, –1]

(A) (B) (C) (D)

38. Two short bar magnets of length 1 cm each have magnetic moments 1.20 Am2 and 1.00 Am2
respectively. They are placed on a horizontal table parallel to each other with their N poles
poining towards the South. They have a common magnetic equator and are separated by a
distance of 20.0 cm. The value of the result and horizontal magnetic induction at the mid-point
O of the line joining their centres is close to (Horizontal component of earth’s magnetic
induction is 3.6× 10–5 Wb/m2) [JEE-MAIN 2013 ; 4/120, –1]
–5 2 –4
(A) 3.6 × 10 Wb/m (B) 2.56 × 10 Wb/m2
(C) 3.50 × 10–4 Wb/m2 (D) 5.80 × 10–4 Wb/m2
PHYSICS WALLAH 96
Magnetic Effect of Current

39. A conductor lies along the z-axis at –1.5  z < 1.5 m and carries a fixed current of 10.0 A in
– â z direction (see figure). For a field B = 3.0 × 10–4e–0.2 x â y T, find the power required to move
the conductor at constant speed to x = 2.0 m, y = 0 m in 5 × 10–3s. Assume parallel motion
along the x-axis [JEE-Main 2014, 4/120, –1]

(A) 1.57 W (B) 2.97 W (C) 14.85 W (D) 29.7 W

40. The Coercivity of a small magnet where the ferromagnet gets demagnetized is 3 × 103 A m–1. The
current required to be passed in a solenoid of length 10 cm and number of turns 100, so that the
magnet gets demagnetized when inside the solenoid, is : [JEE(Mains) - 2014]
(A) 3A (B) 6 A (C) 30 mA (D) 60 mA

41. Two long current carrying thin wires, both with current I, are held by the insulating threads of
length L and are in equilibrium as shown in the figure, with threads making an angle '' with
the vertical. If wires have mass  per unit length then the value of I is :-
(g = gravitational acceleration) [JEE (Mains) - 2015]

 L

I I

gL  gL  gL  gL
(A) 2 tan  (B) tan  (C) sin  (D) 2 sin 
0 0  0 cos   0 cos 

42. Two coaxial solenoids of different raddi carry current I in the same direction. Let F1 be the
magnetic force on the inner solenoid due to the outer one and F2 be the magnetic force on the
outer solenoid due to the inner one. then : [JEE Main - 2015]
(A) F1 is radially inwards and F2 = 0

(B) F1 is radially outward and F2 = 0

(C) F1 = F2 = 0

(D) F1 is radially inwards and F2 is radially outwards.


PHYSICS WALLAH 97
Magnetic Effect of Current

43. A rectangular loop of sides 10 cm and 5 cm carrying a current 1 of 12 A is placed in different


orientations as shown in the figures below: [JEE MAIN 2015]
z z
1
(a) B
(b) B
1 1
1 1
y y
1 1
x x 1
z z
1 B
(c) (d) B
1
1 y 1 1
y
1 1
x x 1
If there is a uniform magnetic field of 0.3 T in the positive z direction, in which orientations
the loop would be in (i) stable equilibrium and (ii) unstable equilibrium ?
(A) (a) and (c), respectively (B) (b) and (d), respectively
(C) (b) and (c), respectively (D) (a) and (b), respectively

44. Hysteresis loops for two magnetic materials A and B are given below :
B B

H H

(A) (B)

These materials are used to make magnets for electric generators, transformer core and electromagnet
core. Then it is proper to use ; [JEE Main-2016]
(A) B for electromagnets and transformers.
(B) A for electric generators and transformers.
(C) A for electromagnets and B for electric transformers.
(D) A for transformers and B for electric generators.

45. Two identical wires A and B, each of length 'l', carry the same current 1. Wire A is bent into a
circle of radius R and wire B is bent to form a square of side 'a'. If B A and BB are the values of
BA
magnetic field at the centres of the circle and square respectively, then the ratio is:
BB
[JEE MAIN 2016]
2 2 2 2
(A) (B) (C) (D)
16 2 16 8 2 8

46. A magnetic needle of magnetic moment 6.7 × 10–2 Am2 and moment of inertia 7.5 × 10–6 kg
m2 is performing simple harmonic oscillations in a magnetic field of 0.01 T. Time taken for 10
complete oscillations is : [JEE Main-2017]
(A) 6.98 s (B) 8.76 s (C) 6.65 s (D) 8.89 s
PHYSICS WALLAH 98
Magnetic Effect of Current

47. An electron, a proton and an alpha particle having the same kinetic energy are moving in
circular orbits of radii re, rp, r respectively in a uniform magnetic field B. The relation between
re, rp, r is [JEE Main-2018]
(A) re < r  rp (B) re > rp = r
(C) re < rp = r (D) re < rp < r

48. A moving coil galvanometer has a coil with 175 turns and area 1 cm 2 . It uses a torsion band of
torsion constant 10 −6 N − m / rad . The coil is placed in a magnetic field B parallel to its plane.
The coil deflects by 1° for a current of 1 mA . The value of B (in Tesla) is approximately:
[JEE Main 2019]
(A) 10 −4 (B) 10 −2 (C) 10 −1 (D) 10 −3

49. An electron, moving along the x -axis with an initial energy of 100eV , enters a region of
magnetic field B = (1.5  10 −3 T ) kˆ at S (see figure). The field extends between x = 0 and

x = 2 cm . The electron is detected at the point Q on a screen placed 8 cm away from the

point S . The distance d between P and Q (on the screen) is: (Electron's charge = 1.6  10 −19 C ,
mass of electron = 9.1 10 −31 kg ) [JEE Main 2019]

(A) 11.65 cm (B) 12.87 cm (C) 1.22 cm (D) 2.25 cm

50. A square loop is carrying a steady current I and the magnitude of its magnetic dipole moment
is m . If this square loop is changed to a circular loop and it carries the same current, the
magnitude of the magnetic dipole moment of circular loop will be: [JEE Main 2019]
m 3m 2m 4m
(A) (B) (C) (D)
   

51. The magnitude of the magnetic field at the center of an equilateral triangular loop of side 1 m
which is carrying a current of 10 A is: [Take  o = 4   10 −7 NA −2 ] [JEE Main 2019]
(A) 18 T (B) 9 T (C) 3 T (D) 1 T

PHYSICS WALLAH 99
Magnetic Effect of Current

52. Two wires A & B are carrying currents I1 and I 2 as shown in the figure. The separation between
them is d . A third wire C carrying a current I is to be kept parallel to them at a distance x
from A such that the net force acting on it is zero. The possible values of x are:
[JEE Main 2019]

 I1  I2
(A) x =   d and x = d
 I1 − I 2  ( I1 + I 2 )
 I2   I2 
(B) x =   d and x =   d

 ( I1 + I 2 )   ( I1 − I 2 ) 
 I1   I2 
(C) x =   d and x =   d

 ( I1 + I 2 )   ( I1 − I 2 ) 
I1d
(D) x = 
( I1 − I 2 )

53. A rigid square of loop of side 'a' and carrying current I 2 is lying on a horizontal surface near a
long current I1 carrying wire in the same plane as shown in figure. The net force on the loop
due to the wire will be: [JEE Main 2019]

 o I1I 2
(A) Repulsive and equal to
2
 o I1I 2
(B) Attractive and equal to
3
 o I1I 2
(C) Repulsive and equal to
4
(D) 7ern

54. A rectangular coil (Dimension 5 cm  2.5 cm ) with 100 turns, carrying a current of 3 A in the
clock-wise direction, is kept centered at the origin and in the X-Z plane. A magnetic field of
1 T is applied along X -axis. If the coil is tilted through 45° about Z-axis, then the torque on the
coil is: [JEE Main 2019]
(A) 0.38Nm (B) 0.55Nm (C) 0.42Nm (D) 0.27 Nm

PHYSICS WALLAH 100


Magnetic Effect of Current

55. Two very long, straight, and insulated wires are kept at 90 angle from each other in xy -plane
as shown in the figure. [JEE Main 2019]

These wires carry currents of equal magnitude I, whose directions are shown in the figure. The
net magnetic field at point P will be:
0I + 0 I 0I
(A) Zero (B) − ( xˆ + yˆ ) (C) ( zˆ ) (D) ( x̂ + ŷ )
2 d d 2 d

56. A circular coil having N turns and radius r carries a current I . It is held in the XZ plane in a
magnetic field B î . The torque on the coil due to the magnetic field is: [JEE Main 2019]
Br I 2
B r I2

(A) (B) Br 2 IN (C) (D) Zero


N N

57. In an experiment, electrons are accelerated, from rest, by applying a voltage of 500 V .
Calculate the radius of the path if a magnetic field 100mT is then applied. [JEE Main 2019]
[Charge of the electron = 1.6  10 −19 C Mass of the electron = 9.1 10 −31 kg ]
(A) 7.5  10 −3 m (B) 7.5  10 −2 m (C) 7.5 m (D) 7.5  10 −4 m

58. One of the two identical conducting wires of length L is bent in the form of a circular loop and
the other one into a circular coil of N identical turns. If the same current is passed in both, the
ratio of the magnetic field at the central of the loop ( B1 ) to that at the centre of the coil ( B C ) ,
BL
i.e, will be: [JEE Main 2019]
BC
1 1
(A) N (B) (C) N 2 (D)
N N2

59. A current loop, having two circular arcs joined by two radial lines is shown in the figure. It
carries a current of 10 A . The magnetic field at point O will be close to: [JEE Main 2019]

(A) 1.0  10 −7 T (B) 1.5  10 −7 T (C) 1.5  10 −5 T (D) 1.0  10 −5 T


PHYSICS WALLAH 101
Magnetic Effect of Current

60. A galvanometer coil has 500 turns and each turn has an average area of 3  10 −4 m 2 . If a torque
of 1.5Nm is required to keep this coil parallel to a magnetic field when a current of 0.5 A is
flowing through it, the strength of the field (in T) is ____. [JEE Main 2020]

61. A coil in the shape of an equilateral triangle of side 10 cm lies in a vertical plane between the
pole pieces of permanent magnet producing a horizontal magnetic field 20mT . The torque
acting on the coil when a current of 0.2 A is passed through it and its plane becomes parallel
to the magnetic field will be x  10 −5 Nm . The value of x is _____. [JEE Main 2021]

62. A uniform conducting wire of length is 24a , and resistance R is wound up as a current
carrying coil in the shape of an equilateral triangle of side 'a' and then in the form of a square
of side 'a'. The coil is connected to a voltage source V0 . The ratio of magnetic moment of the
coils in case of equilateral triangle to that for square is 1: y where y is ____. [JEE Main 2021]

63. A long solenoid with 1000 turns /m has a core material with relative permeability 500 and
volume 10 3 cm 3 . If the core material is replaced by another material having relative
permeability of 750 with same volume maintaining same current of 0.75 A in the solenoid, the
 x 
fractional change in the magnetic moment of the core would be approximately   . Find
 499 
the value of x. [JEE Main 2021]

64. Two 10 cm long, straight wires, each carrying a current of 5 A are kept parallel to each other.
If each wire experienced a force of 10 −5 N , then separation between the wires is _____ cm .
[JEE Main 2022]

65. A deuteron and a proton moving with equal kinetic energy enter into a uniform magnetic field
at right angle to the field. If rd and rp are the radii of their circular paths respectively, then the
rd
ratio will be x :1 where x is _____. [JEE Main 2022]
rp

66. A singly ionized magnesium atom (A = 24) ion is accelerated to kinetic energy 5keV , and is
projected perpendicularly into a magnetic field B of the magnitude 0.5 T . The radius of path
formed will be _____ cm. [JEE Main 2022]

67. A wire of length 314 cm carrying current of 14 A is bent to form a circle. The magnetic
moment of the coil iA A − m 2 is. [Given  = 3.14 ] [JEE Main 2022]

68. A closely wounded circular coil of radius 5 cm produces a magnetic field of 37.68  10 −4 T at
its center. The current through the coil is _____ A. [Given, number of turns in the coil is 100
and  = 3.14 ] [JEE Main 2022]

PHYSICS WALLAH 102


Magnetic Effect of Current

69. A single turn current loop in the shape of a right angle triangle with sides 5 cm,12 cm,13 cm is
carrying a current of 2 A . The loop is in a uniform magnetic field of magnitude 0.75 T whose
direction is parallel to the current in the 13 cm side of the loop. The magnitude of the magnetic
x
force on the 5 cm side will be N . The value of x is _____. [JEE Main 2023]
130

70. Two long parallel wires carrying currents 8 A and 15 A in opposite directions are placed at a
distance of 7 cm from each other. A point P is at equidistant from both the wires such that
the lines joining the point P to the wires are perpendicular to each other. The magnitude of
magnetic field at P is _____  10 −6 T . (Given : 2 = 1.4 ) [JEE Main 2023]

71. A charge particle of 2C accelerated by a potential difference of 100 V enters a region of
uniform magnetic field of magnitude 4mT at right angle to the direction of field. The charge
particle completes semicircle of radius 3 cm inside magnetic field. The mass of the charge
particle is ____ 10 −18 kg [JEE Main 2023]

72. Two identical circular wires of radius 20 cm and carrying current 2 A are placed in
perpendicular planes as shown in figure. The net magnetic field at the centre of the circular
wires is _____  10 −8 T . (Take  = 3.14) [JEE Main 2023]

73. A proton with a kinetic energy of 2.0 eV moves into a region of uniform magnetic field of

magnitude  10 −3 T . The angle between the direction of magnetic field and velocity of
2
proton is 60 . The pitch of the helical path taken by the proton is _____ cm.
(Take, mass of proton = 1.6  10 −27 kg and Charge on proton = 1.6  10 −19 C ). [JEE Main 2023]

74. The ratio of magnetic field at the centre of a current carrying coil of radius r to the magnetic
field at distance r from the centre of coil on its axis is x :1 . The value of x is _____.
[JEE Main 2023]

75. A straight wire carrying a current of 14 A is bent into a semi-circular arc of radius 2.2 cm as
shown in the figure. The magnetic field produced by the current at the centre ( O ) of the arc. is
___  10 −4 T [JEE Main 2023]

PHYSICS WALLAH 103


Magnetic Effect of Current

76. A straight wire AB of mass 40 g and length 50 cm is suspended by a pair of flexible leads in
uniform magnetic field of magnitude 0.40 T as shown in the figure. The magnitude of the
current required in the wire to remove the tension in the supporting leads is A.
(Take g = 10 ms −2 ). [JEE Main 2023]

77. A square loop of edge length 2 m carrying current of 2 A is placed with its edges parallel to
the x − y axis. A magnetic field is passing through the x − y plane and expressed as
B = B0 (1 + 4 x ) kˆ , where Bo = 5T . The net magnetic force experienced by the loop is _____ N .
[JEE Main 2024]

78. A straight magnetic strip has a magnetic moment of 44Am 2 . If the strip is bent in a
22
semicircular shape, its magnetic moment will be _____ Am 2 . (given  = )
7
[JEE Main 2024]

79. An electron with kinetic energy 5eV enters a region of uniform magnetic field of 3T
perpendicular to its direction. An electric field E is applied perpendicular to the direction of
velocity and magnetic field. The value of E , so that electron moves along the same path, is
_____ NC −1 . (Given, mass of electron = 9  10 −31 kg , electric charge = 1.6 10 −19 C)
[JEE Main 2024]

80. A square loop PQRS having 10 turns, area 3.6  10 −3 m 2 and resistance 100Ω is slowly and
uniformly being pulled out of a uniform magnetic field of magnitude B = 0.5 T as shown.
Work done in pulling the loop out of the field in 1.0 s is _____  10 −6 J . [JEE Main 2024]

PHYSICS WALLAH 104


Magnetic Effect of Current

EXERCISE #6 (JEE ADVANCED)

1. Mark correct option [JEE 2003 (Scr)]

(A) coil expands (B) coil contracts (C) coil moves left (D) coil moves right

2. Figure represents four positions of a current carrying coil is a magnetic field directed towards
right. represent the direction of area of vector of the coil. The correct order of potential energy
is : [JEE 2003 (Scr)]

B
n
n n
n
(A) I > III > II > IV (B) I < III < II < IV (C) IV < I < II < III (D) III > II > IV > I

3. A wheel of radius R having charge Q, uniformly distributed on the rim of the wheel is free to
rotate about a light horizontal rod. The rod is suspended by light inextensible stringe and a
magnetic field B is applied as shown in the figure. The initial tensions in the strings are T0. If
3T0
the breaking tension of the strings are , find the maximum angular velocity 0 with which
2
the wheel can be rotate.
d

T0 T0
0

4. For a positively charged particle moving in a x – y plane initially along the x–axis, there is a
sudden change in it path due to the presence of electric and/or magnetic field beyond P. The
curved path is shown in the x – y plane and is found to be non–circular. Which one of the
following combinations is possible? [IIT JEE-2003]
y

P x
O

(A) E = 0; B = bjˆ + ckˆ ˆ B = ckˆ + aiˆ


(B) E = ai;
(C) E = 0; B = cjˆ + bkˆ ˆ B = ckˆ + bjˆ
(D) E = ai;

5. A proton and an alpha particle, after being accelerated through same potential difference, enter
a uniform magnetic field the direction of which is perpendicular to their velocities. Find the
ratio of radii of the circular paths of the two particles. [IIT JEE-2004]

PHYSICS WALLAH 105


Magnetic Effect of Current

6. An electron moving with a speed u along the positive x–axis at y = 0 enters a region of uniform
magnetic field B = − B kˆ which exists to the right of y–axis. The electron exist from the region
0

after sometime with the speed v at co–ordinate y, then : [IIT JEE-2004]


y

e– u
x

(A) v > u, y < 0 (B) v = u, y > 0 (C) v > u, y > 0 (D) v = u, y < 0

7. A moving coil galvanometer experiences torque = ki where i is current. If N coils of area A


each and moment of inertia I is kept in magnetic field B. [IIT JEE-2005]
(i) Find k in terms of given parameters,

(ii) If for current i deflection is, find out tensional constant of spring.
2
(iii) If a charge Q is passed suddenly through the galvanometer. Find out maximum angle of
deflection.

8. An infinite current carrying wire passes through point O and in perpendicular to the plane
containing a current carrying loop ABCD as shown in the figure. Choose the correct option (s).

(A) Net force on the loop is zero.


(B) Net torque on the loop is zero.
(C) As seen from O, the loop rotates clockwise.
(D) As seen from O, the loop rotates anticlockwise [IIT JEE-2006]

9. A magnetic field B = B0 ˆj exists in the region a < x < 2a and B = –B0 ˆj , in the region 2a < x < 3a,
where B0 is a positive constant. A positive point charge moving with a velocity v = v0 î ,
where v0 is a positive constant, enters the magnetic field at x = a. The trajectory of the charge
in this region can be like, [IIT JEE-2007]

(A) (B) (C) (D)

PHYSICS WALLAH 106


Magnetic Effect of Current

10. Two wires each carrying a steady current I are shown in four configurations in Column I. Some of
the resulting effects are described in Column II. Match the statements in Column I with the
statements in Column II and indicate your answer by darkening appropriate bubbles in the 4 × 4
matrix given in the ORS. [IIT JEE-2007]
Column I Column II
(A) Point P is situated (P) The magnetic fields (B) at P due to the
midway between the wires currents in the wires are in the same
direction.

(B) Point P is situated at the (Q) The magnetic fields (B) at P due to the
mid-point of the line joining currents in the wires are in opposite
the centers of the circular directions
wires, which have same radii.

(C) Point P is situated at the (R) There is no magnetic field at P


mid-point of the line joining
the centers of the circular
wires, which have same radii.

(D) Point P is situated at the (S) The wires repel each other
common center of the wires

11. A particle of mass m and charge q, moving with velocity v enters region II normal to the
boundary as shown in the figure. Region II has a uniform magnetic field B perpendicular to the
plane of the paper. The length of the Region II is . Choose the correct choice (s).
[IIT JEE-2008]
Region I Region II Region III

0
V

(A) The particle enters Region III only if its velocity v > q B
m
q B
(B) The particle enters Region III only if its velocity v <
m
q B
(C) Path length of the particle in Region II is max. when velocity v=
m
(D) Time spent in Region II is same for any velocity v as long as the particle returns to Region I
PHYSICS WALLAH 107
Magnetic Effect of Current

12. Statement-1: The sensitivity of a moving coil galvanometer is increased by placing a suitable
magnetic material as a core inside the coil. and [IIT-JEE 2008]
Statement -2: Soft iron has a high magnetic permeability and cannot be easily magnetized or
demagnetized.
(A) Statement-1 is True, Statement-2 is True; Statement-2 is a correct explanation for
Statement-1
(B) Statement-1 is True, Statement-2 is True; Statement-2 is NOT a correct explanation for
Statement-1
(C) Statement-1 is True, Statement-2 is False
(D) Statement-1 is False, Statement-2 is True

13. Six point charges, each of the same magnitude q, are arranged in different manners as shown in
Column II. In each case, a point M and a line PQ passing through M are shown. Let E be the
electric field and V be the electric potential at M (potential at infinity is zero) due to the given
charge distribution when it is at rest. Now, the whole system is set into rotation with a constant
angular velocity a about the line PQ. Let B be the magnetic field at M and µ be the magnetic
moment of the system in this condition. Assume each rotating charge to be equivalent to a
steady current. [IIT JEE-2009]
Column I Column II
(A) E = 0 (P) Charges are at the corners of a regular hexagon. M is at the centre of the
hexagon. PQ is perpendicular to the plane of the hexagon.
+ –
Q
– +
M
P+ –

(B) V  0 (Q) Charges are on a line perpendicular to PQ at equal intervals. M is the


mid-point be tween the two innermost charges.
P
M
– + – + – +

(C) B = 0 (R) Charges are placed on two coplanar circles of radius a and 2a. M is the
at the centre of the circle PQ is perpendicular to the plane of the circles.
+ – +
Q
+
– M–
P
+
(D) µ  0 (S) Charges are placed at the corners of a rectangle of sides a and 2a and at
the mid points of the longer sides. M is at the centre of the rectangle. PQ
is parallel to the longer sides.
– + –
M
P Q
– + –
(T) Charges are placed on two coplanar, identical insulating rings at equal
intervals. M is the mid point between the centres of the rings. PQ is
perpendicular to the line joining the centres and coplanar to the rings
P
+ –

M
+ + – –

PHYSICS WALLAH 108


Magnetic Effect of Current

14. Column II shows five systems in which two objects are labelled as X and Y. Also in each case
a point P is shown. Column I gives some statements about X and/or Y. Match these statements
to the appropriate system(s) from Column II. [IIT JEE-2009]
Column-I Column-II
(A) The force exerted by X on Y has a (P) Block Y of mass M left on a fixed inclined
magnitude Mg. plane X, slides on it with a constant velocity.
Y

P
(B) The gravitational potential energy (Q) Two ring magnets Y and Z, each of mass M,
of X is continuously increasing are kept in frictionless vertical plastic stand
so that they repel each other. Y rests on the
base X and Z hangs in air in equilibrium. P
is the top most point of the stand on the
common axis of the two rings. The whole
system is in a lift that is going up with a
constant velocity.
P

Z
Y
X

(C) Mechanical energy of the system (R) A pulley Y of mass m0 is fixed to a


X + Y is continuously decreasing table through a clamp X. A block of mass M
hangs from a string that goes over the
pulley and is fixed at point P of the table.
The whole system is kept in a lift that is
going down with a constant velocity.

P Y
X

(D) The torque of the weight of Y about (S) A sphere Y of mass M is put in a
point P is zero. no viscous liquid X kept in a container
at rest. The sphere is released and it
moves down in the liquid.

X
P
(T) A sphere Y of mass M is falling with
its terminal velocity in a viscous liquid
X kept in a container.

X
P

PHYSICS WALLAH 109


Magnetic Effect of Current

15. A steady current I goes through a wire loop PQR having shape of a right angle triangle with
PQ = 3x, PR = 4x and QR = 5x. If the magnitude of the magnetic field at P due to this loop is
 µ I 
k  0  , find the value of k. [IIT JEE-2009]
 48 x 

16. A thin flexible wire of length L is connected to two adjacent fixed points and carries a current I
in the clockwise direction, as shown in the figure. When the system is put in a uniform
magnetic field of strength B going into the plane of the paper, the wire takes the shape of a
circle. The tension in the wire is : [IIT JEE-2010]

IBL IBL IBL


(A) IBL (B) (C) (D)
 2 4

Paragraph for Q.No. 17 to 18


Electrical resistance of certain materials, known as superconductors, changes abruptly from a
nonzero value to zero as their temperature is lowered below a critical temperature TC(0). An
interesting property of superconductors is that their critical temperature becomes smaller than
TC(0) if they are placed in a magnetic field, i.e., the critical temperature TC(B) is a function of the
magnetic field strength B. The dependence of TC (B) on B is shown in the figure. [IIT JEE-2010]

17. In the graphs below, the resistance R of a superconductor is shown as a function of its temperature
T for two different magnetic fields B1(solid line) and B2 (dashed line). If B2 is larger than B1,
which of the following graphs shows the correct variation of R with T in these fields?

(A) (B) (C) (D)

18. A superconductor has TC(0) = 100 K. When a magnetic field of 7.5 Tesla is applied, its TC
decreases to 75 K. For this material one can definitely say that when
(A) B = 5 Tesla, TC (B) = 80 K
(B) B = 5 Tesla, 75 K < TC(B) < 100 K
(C) B = 10 Tesla, 75 K < TC(B) < 100K
(D) B = 10 Tesla, TC(B) = 70 K

PHYSICS WALLAH 110


Magnetic Effect of Current

19. An electron and a proton are moving on straight parallel paths with same velocity. They enter a
semi-infinite region of uniform magnetic field perpendicular to the velocity. Which of the
following statement(s) is/are true? [IIT JEE - 2011]
(A) they will never come out of the magnetic field region
(B) they will come out travelling along parallel paths
(C) they will come out of the same time
(D) they will come out at different times

20. A long insulated copper wire is closely wound as a spiral of ‘N’ turns. The spiral has inner radius
‘a’ and outer radius ‘b’. The spiral lies in the X-Y plane and a steady current ‘I’ flows through the
wire. The Z-component of the magnetic field at the center of the spiral is [IIT JEE - 2011]
Y

I a X
b

 0 NI b  0 NI b  0 NI b+a   0 NI  b + a 


(A) n  (B) n  (C) n  (D) n 
2(b − a)  a  2b a 2(b − a)  b − a  2b  b−a 

21. Consider the motion of a positive point charge in a region where there are simultaneous uniform
electric and magnetic fields E = E ˆj and B = B ˆj . At time t = 0, this charge has velocity v in
0 0

the x-y plane, making an angle  with the x-axis. Which of the following option (s) is (are)
correct for time t > 0? [IIT JEE - 2012]
(A) If  = 0°, the charge moves in a circular path in the x-z plane.
(B) If  = 0°, the charge undergoes helical motion with constant pitch along the y-axis.
(C) If  = 10°, the charge undergoes helical motion with its pitch increasing with time, along the y-axis
(D) If  = 90°, the charge undergoes linear but accelerated motion along the y-axis

22. A cylindrical cavity of diameter a exists inside a cylinder of diameter 2a as shown in the figure.
Both the cylinder and the cavity are infinitely long. A uniform current density J flows along the
N
length. If the magnitude of the magnetic field at the point P is given by  0 aJ , then the value
12
of N is [IIT JEE- 2012]

a
P O

2a

23. An infinitely long hollow conducting cylinder with inner radius R/2 and outer radius R carries
a uniform current density along its length. The magnitude of the magnetic field, | B | as a
function of the radial distance r from the axis is best represented by [IIT JEE-2012]

(A) (B) (C) (D)

PHYSICS WALLAH 111


Magnetic Effect of Current

24. A particle of mass M and positive charge Q, moving with a constant velocity u 1 = 4iˆ ms −1 ,
enters a region of uniform static magnetic field normal to the x-y plane. The region of the
magnetic field extends from x = 0 to x = L from all values of y. After passing through this
region, the particle emerges on the other side after 10 milliseconds with a velocity u 2 = 2 ( 3iˆ + ˆj) ms −1 .
The correct statements(s) is (are)
(A) The direction of the magnetic field is –z direction. [IIT JEE- 2013]
(B) The direction of the magnetic field is +z direction.
50 M
(C) The magnitude of the magnetic field units.
3Q
100 M
(D) The magnitude of the magnetic field is units.
3Q

25. A steady current I flows along an infinitely long hollow cylindrical conductor of radius R. This
cylinder is placed coaxially inside an infinite solenoid of radius 2R. The solenoid has n turns
per unit length and carries a steady current I. Consider a point P at a distance r from the
common axis. The correct statement(s) is (are) : [IIT JEE- 2013]
(A) In the region 0 < r < R, the magnetic field is non-zero
(B) In the region R < r < 2R, the magnetic field is along the common axis
(C) In the region R< r < 2R, the magnetic field is tangential to the circle of radius r, centred on the axis
(D) In the region r > 2R, the magnetic field is non-zero

26. Two parallel wires in the plane of the paper are distance X0 apart. A point charge is moving with
speed u between the wires in the same plane at a distance X1 from one of the wires. When the
wires carry current of magnitude I in the same direction, the radius of curvature of the path of the
point charge is R1. In contrast, if the currents I in the two wires have directions opposite to each
x0 R1
other, the radius of curvature of the path is R2. If = 3 , the value of is [JEE Adv.-2014]
x1 R2

Paragraph for Questions 27 & 28


The figure shows a circular loop of radius a with two long parallel wires (numbered 1 and 2)
all in the plane of the paper. The distance of each wire from the centre of the loop is d. The
loop and the wires are carrying the same current I. The current in the loop is in the counter
clockwise direction if seen from above. [JEE Adv.-2014]

Q S
d d
Wire 1 Wire 2
a

P R

PHYSICS WALLAH 112


Magnetic Effect of Current

27. When d  a but wires are not touching the loop, it is found that the net magnetic field on the
axis of the loop is zero at a height h above the loop. In that case
(A) current in wire 1 and wire 2 is the direction PQ and RS, respectively and h  a
(B) current in wire 1 and wire 2 is the direction PQ and SR, respectively and h  a
(C) current in wire 1 and wire 2 is the direction PQ and SR, respectively and h  1.2a
(D) current in wire 1 and wire 2 is the direction PQ and RS, respectively and h  1.2a

28. Consider d >> a, and the loop is rotated about its diameter parallel to the wires by 30° from the
position shown in the figure. If the currents in the wires are in the opposite directions, the
torque on the loop at its new position will be (assume that the net field due to the wires is
constant over the loop)
 0I2a 2  0I2a 2 3 0 I2a 2 3 0 I2a 2
(A) (B) (C) (D)
d 2d d 2d

29. A conductor (shown in the figure) carrying constant current I is kept in the x-y plane in a
uniform magnetic field B . If F is the magnitude of the total magnetic force acting on the
conductor, then the correct statement(s) is(are) [JEE Adv.-2015]
y
R R
I /6 /4
L R R L x
(A) If B is along ẑ, F  (L + R) (B) If B is along x̂, F = 0
(C) If B is along ŷ, F  (L + R ) (D) If B is along ẑ, F = 0

Paragraph for Questions 30 & 31


In a thin rectangular metallic strip a constant current I flows along the positive x-direction, as
shown in the figure. The length, width and thickness of the strip are , w and d, respectively. A
uniform magnetic field B is applied on the strip along the positive y-direction. Due to this, the
charge carriers experience a net deflection along the z-direction. This results in accumulation
of charge carriers on the surface PQRS and appearance of equal and opposite charges on the
face opposite to PQRS. A potential difference along the z-direction is thus developed. Charge
accumulation continues until the magnetic force is balanced by the electric force. The current
is assumed to be uniformly distributed on the cross section of the strip and carried by electrons.
[JEE Adv.-2015(Paper-2)]
l y

I K I
W R x
S d M
z
P Q

30. Consider two different metallic strips (1 and 2) of the same material. Their lengths are the same, widths
are w1 and w2 and thicknesses are d1 and d2, respectively. Two points K and M are symmetrically
located on the opposite faces parallel to the x-y plane (see figure). V1 and V2 are the potential
differences between K and M in strips 1 and 2, respectively. Then, for a given current I flowing
through them in a given magnetic field strength B, the correct statement(s) is(are)
(A) If w1 = w2 and d1 = 2d2, then V2 = 2V1 (B) If w1 = w2 and d1 = 2d2, then V2 = V1
(C) If w1 = 2w2 and d1 = d2, then V2 = 2V1 (D) If w1 = 2w2 and d1 = d2, then V2 = V1
PHYSICS WALLAH 113
Magnetic Effect of Current

31. Consider two different metallic strips (1 and 2) of same dimensions (lengths, width w and
thickness d) with carrier densities n1 and n2, respectively. Strip 1 is placed in magnetic field B1
and strip 2 is placed in magnetic field B2, both along positive y-directions. Then V1 and V2 are
the potential differences developed between K and M in strips 1 and 2, respectively. Assuming
that the current I is the same for both the strips, the correct option(s) is(are)
(A) If B1 = B2 and n1 = 2n2, then V2 = 2V1
(B) If B1 = B2 and n1 = 2n2, then V2 = V1
(C) If B1 = 2B2 and n1 = n2, then V2 = 0.5V1
(D) If B1 = 2B2 and n1 = n2, then V2 = V1

32. A symmetric star shaped conducting wire loop is carrying a steady state current I as shown in
the figure. The distance between the diametrically opposite vertices of the star is 4a. The
magnitude of the magnetic field at the center of the loop is [JEE Adv.-2017]

4a

0I 0I
(A) 6  3 − 1 (B) 6  3 + 1
4 a   4 a  
 I 0I
(C) 0 3  3 − 1 (D) 3 2 − 3 
4 a   4 a  

3R
33. A uniform magnetic field B exists in the region between x = 0 and x = (region 2 in the
2
figure) pointing normally into the plane of the paper. A particle with charge +Q and
momentum p directed along x-axis enters region 2 from region 1 at point P1 (y = –R). Which of
the following option(s) is/are correct? [JEE Adv.-2017]
y
Region 1 Region 2 Region 3
× × ×
× ×B ×
× × ×
× × ×
× × ×
O
× × × P2 x
× × ×
+Q P1 × × ×
(y = –R) × × ×
× ×
3R/2
2 p
(A) For B  , the particle will re-enter region 1
3 QR
8 p
(B) For B = , the particle will enter region 3 through the point P2 on x-axis
13 QR
(C) When the particle re-enters region 1 through the longest possible path in region 2, the
magnitude of the change in its liner momentum between point P1 and the farthest point
from y-axis is p / 2
(D) For a fixed B, particle of same charge Q and same velocity v, the distance between the
point P1 and the point of re-entry into region 1 is inversely proportional to the mass of the particle
PHYSICS WALLAH 114
Magnetic Effect of Current

Answer Q.34, Q.35 and Q.36 by appropriately matching the information given in the
three columns of the following table.

A charged particle (electron or proton) is introduced at the origin (x = 0, y = 0, z = 0) with a


given initial velocity v . A uniform electric field E -and a uniform magnetic field B -exist everywhere.
The velocity v , electric field E -and magnetic field B are given in column 1, 2 and 3, respectively.
The quantities E0, B0 are positive in magnitude. [JEE Adv.-2017]
Column-1 Column-2 Column-3
E0
(I) Electron with v = 2 xˆ (i) E = E 0 zˆ (P) B = − B 0 xˆ
B0

E0
(II) Electron with v = yˆ (ii) E = − E 0 yˆ (Q) B = B 0 xˆ
B0

(III) Proton with v = 0 (iii) E = − E 0 xˆ (R) B = B 0 yˆ


E0
(IV) Proton with v = 2 xˆ (iv) E = E 0 xˆ (S) B = B 0 zˆ
B0

34. In which case will the particle move in a straight line with constant velocity?
(A) (II) (iii) (S) (B) (IV) (i) (S) (C) (III) (ii) (R) (D) (III) (iii) (P)

35. In which case will the particle describe a helical path with axis along the positive z direction?
(A) (II) (ii) (R) (B) (IV) (ii) (R) (C) (IV) (i) (S) (D) (III) (iii) (P)

36. In which case would the particle move in a straight line along the negative direction of y-axis
(i.e., move along − ŷ )?
(A) (IV) (ii) (S) (B) (III) (ii) (P) (C) (II) (iii) (Q) (D) (III) (ii) (R)

37. Two infinitely long straight wires lie in the xy-plane along the lines x = ±R The wire located at
x = +R. carries a constant current I1 and the wire located at x = – R carries a constant current I2.
A circular loop of radius R is suspended with its centre at (0,0, 3 R) and in a plane parallel to
the xy-plane. This loop carries a constant current I in the clockwise direction as seen from
above the loop. The current in the wire is taken to be positive if it is in the + ˆj direction. Which

of the following statements regarding the magnetic field B is (are) true? [JEE Adv.-2018]
(A) If I1 = I2, then B cannot be equal to zero at the origin (0,0,0)
(B) If I1 > 0 and I2 < 0, then B can be equal to zero at the origin (0,0,0)
(C) If I1 < 0 and I2 > 0, then B can be equal to zero at the origin (0,0,0)
 0I 
(D) If I1 = I2, then the z-component of the magnetic field at the centre of the loop is  − 
 2R 

PHYSICS WALLAH 115


Magnetic Effect of Current

PARAGRAPH “X” (For Question 38, 39)


In electromagnetic theory, the electric and magnetic phenomena are related to each other.
Therefore, the dimensions of electric and magnetic quantities must also be related to each
other. In the questions below, [E] and [B] stand for dimensions of electric and magnetic fields
respectively, while [0] and [0] stand for dimensions of the permittivity and permeability of
free space respectively. [L] and [T] are dimensions of length and time respectively. All the
quantities are given in SI units.
(There are two questions based on PARAGRAPH “X”, the question given below is one of
them) [JEE Adv.-2018]
38. The relation between [E] and [B] is
(A) [E] = [B] [L] [T] (B) [E] = [B] [L]–1 [T]
(C) [E] = [B] [L] [T]–1 (D) [E] = [B] [L]–1 [T]–1

39. The relation between [0] and [0] is


(A) [0] = [0] [L]2 [T]–2 (B) [0] = [0] [L]–2 [T]2
(C) [0] = [0]–1 [L]2 [T]–2 (D) [0] = [0]–1 [L]–2 [T]2

40. A moving coil galvanometer has 50 turns and each turn has an area 2 × 10 –4 m2. The magnetic
field produced by the magnet inside the galvanometer is 0.02 T. The torsional constant of the
suspension wire is 10–4 N m rad–1. When a current flows through the galvanometer, a full scale
deflection occurs if the coil rotates by 0.2 rad The resistance of the coil of the galvanometer is
50  This galvanometer is to be converted into an ammeter capable of measuring current in
the range 0–1.0A. For this purpose, a shunt resistance is to be added in parallel to the
galvanometer. The value of this shunt resistance, in ohm, is __________. [JEE Adv.-2018]

41. Two infinitely long straight wires lie in the xy-plane along the lines x = R. The wire located
at x = +R carries a constant current I1 and the wire located at x = −R carries a constant current
I2. A circular loop of radius 𝑅 is suspended with its centre at (0, 0, 3R ) and in a plane parallel
to the xy-plane. This loop carries a constant current I in the clockwise direction as seen from
above the loop. The current in the wire is taken to be positive if it is in the + ˆj direction. Which
of the following statements regarding the magnetic field B is (are) true?
[JEE Adv.-2018(Paper-1)]
(A) If I1 = I2, then B cannot be equal to zero at the origin (0, 0, 0)
(B) If I1 > 0 and I2 < 0, then B can be equal to zero at the origin (0, 0, 0)
(C) If I1 < 0 and I2 > 0, then B can be equal to zero at the origin (0, 0, 0)
 0I 
(D) If I1 = I2, then the 𝑧-component of the magnetic field at the centre of the loop is  − 
 2R 
PHYSICS WALLAH 116
Magnetic Effect of Current

42. In the xy-plane, the region y > 0 has a uniform magnetic field B1 kˆ and the region y < 0 has another

uniform magnetic field B 2 kˆ . A positively charged particle is projected from the origin along

the positive y-axis with speed v0 =  ms–1 at t = 0, as shown in the figure. Neglect gravity in
this problem. Let t = T be the time when the particle crosses the x-axis from below for the first
time. If B2 = 4B1, the average speed of the particle, in ms–1, along the x-axis in the time
interval T is __________. [JEE Adv.-2018(Paper-1)]

43. A conducting wire of parabolic shape, initially y = x 2, is moving with velocity V = V0 ˆi in a



 y 
non-uniform magnetic field B = B 0  1 +    k̂ , as shown in figure. If V0, B0, L and β are positive
 L 
 

constants and Δϕ is the potential difference developed between the ends of the wire, then the
correct statement(s) is/are: [JEE Adv.-2019(Paper-1)]

B


V = V0ˆi

1
(A) Δ = B 0 V0 L for β = 0
2

4
(B) Δ = B 0 V0 L for β = 2
3

(C) |Δϕ| remains the same if the parabolic wire is replaced by a straight wire, y = x initially, of
length 2L

(D) |Δϕ| is proportional to the length of the wire projected on the y-axis.

PHYSICS WALLAH 117


Magnetic Effect of Current

44. A light disc made of aluminium (a nonmagnetic material) is kept horizontally and is free to
rotate about its axis as shown in the figure. A strong magnet is held vertically at a point above
the disc away from its axis. On revolving the magnet about the axis of the disc, the disc will
(figure is schematic and not drawn to scale) [JEE Adv.-2020(Paper-1)]

(A) rotate in the direction opposite to the direction of magnet’s motion


(B) rotate in the same direction as the direction of magnet’s motion
(C) not rotate and its temperature will remain unchanged
(D) not rotate but its temperature will slowly rise

45. An -particle (mass 4 amu) and a singly charged sulfur ion (mass 32 amu) are initially at rest.
They are accelerated through a potential V and then allowed to pass into a region of uniform
magnetic field which is normal to the velocities of the particles. Within this region, the -particle
and the sulfur ion move in circular orbits of radii ra and rS, respectively. The ratio (rS/ra) is
________. [JEE Adv.-2021(Paper-1)]

46. A physical quantity S is defined as S = (E  B)/μ 0 , where E is electric field, B is magnetic


field and 0 is the permeability of free space. The dimensions of S are the same as the
dimensions of which of the following quantity(ies) ? [JEE Adv.-2022(Paper-1)]
Energy Force
(A) (B)
Charge  Current Length  Time
Energy Power
(C) (D)
Volume Area

47. Two concentric circular loops, one of radius R and the other of radius 2R, lie in the xy-plane
with the origin as their common center, as shown in the figure. The smaller loop carries current
I1 in the anti-clockwise direction and the larger loop carries current I2 in the clockwise
direction, with I2 > 2I1. B(x, y) denotes the magnetic field at a point (𝑥, 𝑦) in the xy-plane.
Which of the following statement(s) is(are) correct? [JEE Adv.-2022(Paper-1)]

(A) B(x, y) is perpendicular to the xy-plane at any point in the plane


(B) | B(x, y) | depends on x and y only through the radial distance r = x 2 + y2
(C) | B(x, y) | is non-zero at all points for r < R
(D) B(x, y) points normally outward from the xy-plane for all the points between the two
loops.
PHYSICS WALLAH 118
Magnetic Effect of Current

48. A bar of mass M = 1.00 kg and length L = 0.20 m is lying on a horizontal frictionless surface.
One end of the bar is pivoted at a point about which it is free to rotate. A small mass m = 0.10 kg is
moving on the same horizontal surface with 5.00 ms−1 speed on a path perpendicular to the bar.
It hits the bar at a distance L/2 from the pivoted end and returns back on the same path with
speed v. After this elastic collision, the bar rotates with an angular velocity . Which of the
following statement is correct? [JEE Adv.-2023(Paper-1)]
(A)  = 6.98 rad s−1 and v = 4.30 m s−1
(B)  = 3.75 rad s−1 and v = 4.30 m s−1
(C)  = 3.75 rad s−1 and v = 10.0 m s−1
(D)  = 6.80 rad s−1 and v = 4.10 m s−1

PHYSICS WALLAH 119


Magnetic Effect of Current

Answer Key

EXERCISE #1
1. (a) 1µT along the negative Z-axis
(b) 3µT along the Z-axis
(c) 1µT along the Z-axis
(d) 3µT along the negative Z-axis
2. 4.0 × 10–5 T, downwards in both the cases
3. 5 × 10–4 T = 1.6 × 10–3 T towards west
4. (b) in a small region of length 2d about the mid-point between the coils,
− 3/ 2 −3/ 2
 2
  R 2
 
 0 IR 2 N   R  2   2
B=    + d  + R  +  − d  + R  
2   2    2
   

 
− 3/ 2 − 3/ 2 − 3/ 2
 0 IR 2 N  5R 2   4d   4d  
    1 +  +  1 −  
2  4    5R   5R  
−3/ 2
 0 IR 2 N  4   6d 6d 
   1 − +1+
2R 3 5  5R 5R 

where in the second and third steps abvoe, terms containing d2/R2 and higher powers of d/R are
d
neglected since  1 . The terms linear in d/R cancel giving a uniform field B in a small
R
region:
3/ 2
4  0 IN  0 IN
B=  0.72
5 R R

1
5.  10 −9 kˆ
2

4r
6. At a distance of from the centre in such a way that the direction of the current in it is

opposite to that in the nearest part of the circular wire.

7. 10–4T, perpendicular to paper outwards

8. (a) 0; (b) 1.41×10–6 T, 45° in xz-plane, (c) 5 × 10–6 T, +x-direction


5
9. 4   10 −5 T
2 2
10. 0 weber. m–1
3 0 i
11. (a) zero; (b)
8 r

PHYSICS WALLAH 120


Magnetic Effect of Current

 0 j(b 2 − a 2 )
12. (a) zero; (b)
2b
N
13. Hint: B for a toroid is given by n = . The field is non-zero only inside the core surrounded
2 r
by the windings. (a) Zero, (b) 3.0 × 10–2T, (c) zero, Note, the field varies slightly across the
cross-section of the toroid as r varies from the inner to outer radius. Answer (b) corresponds to
the mean radius r = 25.5 cm.
14. Magnetic field strength, B = 100G = 100 × 10–4 T
number of turns per unit length, n = 1000 turns m–1
Current flowing in the coil, I = 15A
Permeability of free space, = 4 × 10–7 T mA–1
Magnetic field is given by the relation,
B = µ0nI
B 100  10 −4
 nI = = = 7957.74
µ0 4   10 −7

 8000 A/m
If the length of the coil is taken as 50 cm, radius 4 cm, number of turns 400, and current 10A,
then these values are not unique for the given purpose. There is always a possibility of some
adjustments with limits.
2mv 0
15. 16. 1.2 × 10–2 m, 4.37 × 10–2 m
qB

17. (a) Circular trajectory of radius 1.0 mm normal to B.


(b) Helical trajectory of radius 0.5 mm with velocity component 2.3 × 107 ms–1 along B.

18. 4.2 cm

19. (a) Initial v is either parallel or anti-parallel to B.


(b) Yes, because magnetic force can change the direction of v, not its magnitude.
(c) B should be in a vertically downward direction.

20. Deuterium ions or deutreons ; the answer is not unique because only the ratio of charge to mass
is determined. Other possible answers are He++, Li+++, etc.

21. 18 MHz

22. (a) A horizontal magnetic field of magnitude 0.26 T normal to the conductor in such a
direction that Fleming a left-hand rule gives a magnetic force upward.
(b) 1.176 N.

23. (a) 2.1 N vertically downwards


(b) 2.1 N vertically downwards (true for any angle between current direction and B since l sin
 remains fixed, equal to 20 cm)
(c) 1.68 N vertically downwards

PHYSICS WALLAH 121


Magnetic Effect of Current

24. (a) 3.1 Nm, (b) No, the answer is unchanged because the formula  = N I A × B is true for a
planar loop of any shape.
25. Use  = IA × B and F = I1 × B
(a) 1.8 × 10–2 N m along –y direction (b) same as in (a)
(c) 1.8 × 10–2 N m along –x direction
(d) 1.8 × 10–2 N m at an angle of 240° with the +x direction
(e) zero (f) zero
Force is zero in each case. Case (e) corresponds to stable, and case (f) corresponds to unstable
equilibrium

26. (a) Zero, (b) zero, (c) force on each electron is evB = IB/(nA) = 5 × 10–25N.
Note : Answer (c) denotes only the magnetic force.
27. (a) 1.4, (b) 1

EXERCISE #2
0I 2  0 br12  0 bR 3
1. 2. 2 3. B1 = , B2 = 4. B min = 4.7  10 −3 T
42 R 3 3r2
2
 3qB 0 d 
5. 15 C 6. (6.4 m, 0,0) (6.4m, 0, 2m) 7. v −  v0 −
2
0 
 2m 
 0 I1 I 2
8. i1 = 0.1110 A, i2 = 0.096 A 9. l n (3) along –ve z direction
4
0I
10. (i) − qv 0 kˆ
4R
(ii) F1 = 2 I R B, F2 = 2IRB, Net force = F1 + F2 = 4IRB î

3mv 2 3mv 3
11. (a) 6.6 × 10–5T, (b) 0, 0, 8 × 10–6Nt 12. (a) , (b) , (c) zero
4qa 4a
 0 iqv mEI
13. 14. 5 15. 16. 0.62 N < F < 0.88 N
2 a Be
Q
17. 8 18. 4 19. 4 20. 20 21. h 2 tan 2 
4
m
22. (a) 20 min. (b) 5.94 × 10–2 Nm 23. T0 = 2  = 0.57 s
6IB

d I 0
24. z = 0, x = ± , (ii)
3 2d 

25. ˆ , (c) I = mg
(a) current in loop PQRS is clockwise from P to QRS., (b) F = BI 0 b(3kˆ − 4i)
6b B 0

 0 I2   L2 + a 2  ˆ
26. F = a2i ˆj 27. 4 28. F=   n   ( − k) , zero 29. 525
 2 
2
 a 

PHYSICS WALLAH 122


Magnetic Effect of Current

EXERCISE #3
1. A 2. B 3. C 4. D 5. A 6. B 7. C
8. C 9. C 10. C 11. D 12. D 13. A 14. B
15. C 16. C 17. D 18. D 19. B 20. A 21. C
22. B 23. C 24. A 25. B 26. A 27. B 28. B
29. C 30. D 31. C 32. A 33. A 34. B 35. C
36. D 37. A 38. B 39. D 40. C 41. C 42. C
43. B 44. C 45. A 46. D 47. D 48. ABC 49. ABC
50. ABC 51. ACD 52. AC 53. D 54. C 55. BD 56. AD
57. AB 58. A
59. (A)→ QS; (B)→PS; (C)→QS; (D)→ P
60. (A)→S; (B)→S; (C)→Q; (D)→R; (E)→ P

EXERCISE #4
1. C 2. B 3. A 4. C 5. B 6. C 7. A
8. A 9. A 10. B 11. C 12. A 13. C 14. C
15. D 16. C 17. C 18. B 19. A 20. D 21. A
22. D 23. B 24. A 25. A 26. B 27. C 28. C
29. A 30. D 31. B 32. C 33. B 34. C 35. B
36. C 37. A 38. B 39. A 40. A 41. B 42. ABC
43. ABD 44. ABC 45. ABC 46. BCD 47. C 48. A 49. A
50. D 51. C 52. B 53. D 54. A 55. A 56. B
57. C 58. A 59. C 60. B 61. D
62. (A)→R; (B)→S; (C)→Q; (D)→P 63. (A)→PRT; (B)→PQ; (C)→S; (D)→P;

EXERCISE #5 (JEE MAINS)


1. (A) 2. (A) 3. (B) 4. (C) 5. (A) 6. (A) 7. (D)
8. (B) 9. (B) 10. (B) 11. (B) 12. (A) 13. (C) 14. (B)
15. (C) 16. (A) 17. (C) 18. (A) 19. (D) 20. (C) 21. (C)
22. (C) 23. (B) 24. (C) 25. (C) 26. (A) 27. (A) 28. (B)
29. (A) 30. (B) 31. (A) 32. (A) 33. (A) 34. (A) 35. (A)
36. (C) 37. (A) 38. (B) 39. (B) 40. (A) 41. (D) 42. (B)
43. (B) 44. (A) 45. (C) 46. (C) 47. (C) 48. (D) 49. (B)
50. (D) 51. (A) 52. (D) 53. (C) 54. (D) 55. (A) 56. (B)

PHYSICS WALLAH 123


Magnetic Effect of Current

57. (D) 58. (D) 59. (D) 60. (20) 61. (3) 62. (3) 63. (250)
64. (5) 65. (2) 66. (10) 67. (11) 68. (3) 69. (9) 70. (68)
71. (144) 72. (628) 73. (40) 74. (8) 75. (2) 76. (2) 77. (160)
78. (28) 79. (4) 80. (3)

EXERCISE #6 (JEE ADVANCED)

T0 d
1. A 2. A 3.  disk =
qBR 2
1
4. B 5. 6. D
2

2I 0 NAB NAB
7. (a) k = NAB; (b) C = ; (c)  = Q
 2I 0 I
8. AC
9. A
10. (A)→QR; (B)→P; (C)→QR; (D)→QS; OR (A)→QR; (B)→P; (C)→QR; (D)→Q;
11. ACD 12. (C)
13. (A)→PRS; (B)→RS; (C)→PQT; (D)→RS;
14. (A)→PT; (B)→QST; (C)→PRT; (D)→Q;
15. 7 16. C 17. A 18. B 19. BC, BD, BCD 20. A
21. CD 22. 5 23. D 24. AC 25. AD 26. 3 27. C
28. B 29. ABC 30. AD 31. AC 32. A 33. AB 34. A
35. C 36. D 37. ABD 38. C 39. D 40. 5.55 / 5.56
41. ABC 42. 2 43. BCD 44. B 45. 4 46. D 47. AB
48. A

PW Web/App - https://smart.link/7wwosivoicgd4
Library- https://smart.link/sdfez8ejd80if

PHYSICS WALLAH 124

You might also like